ORTHOPEDIC MCQS ONLINE QUESTION BANK H2C
ORTHOPEDIC MCQS ONLINE QUESTION BANK H2C
2379. (3838) Q5-7621:
An 18-month-old patient is suspected of X-linked hypophosphatemic rickets. The clinical findings are most likely to include all of the following except:
1) Below average stature
3) Dental abscesses or caries
2) Exaggerated genu varus
5) Alopecia
4) Delayed motor milestones
Patients with X-linked hypophosphatemic rickets may present with below average stature, exaggerated genu varus, dental abscesses or caries, and delayed motor milestones. Alopecia is not common in patients with this condition but is characteristic of the rare condition of end-organ insensitivity to vitamin D (vitamin D-resistant rickets).
Correct Answer: Alopecia
2380. (3839) Q5-7622:
A newborn is brought in for consultation due to a foot deformity. The left foot is dorsiflexed at the ankle, and the dorsum of the foot is able to be placed against the tibia. Neurologic and vascular examination is normal. Recommended treatment includes:
1) Magnetic resonance imaging for completion of diagnosis
3) Observation
2) Serial casting
5) Lengthening of the anterior tibialis tendon
4) Ankle-foot orthosis
The newborn is presenting with a calcaneovalgus foot. This condition resolves spontaneously, therefore, no additional work-up is needed.
Correct Answer: Observation
2381. (3840) Q5-7623:
In a newborn, how much angulation is acceptable in a fracture of the humeral diaphysis:
1) 10°
3) 30°
2) 20°
5) 50°
4) 40°
In the humeral diaphysis of a newborn, one can accept virtually any alignment because of the tremendous remodeling capacity of the child. Complete remodeling of 66° within 1 year has been documented.
Correct Answer: 50°
2382. (3841) Q5-7624:
Which of the following radiographic parameters remains essentially fixed throughout life:
1) Pelvic tilt
3) Pelvic incidence
2) Sacral slope
5) Lumbar lordosis
4) T1 tilt
Pelvic incidence is a measure of the relationship between the superior sacral endplate and the hips. It does not change significantly during life unless there is a major pelvic fracture. Pelvic incidence is defined as the angle between a line connecting the hip center and the midpoint of the S1 endplate, and a perpendicular to the midpoint of the S1 endplate. This angle is fixed in bone. By contrast, all of the other angles vary with posture and age.
Correct Answer: Pelvic incidence
2383. (3842) Q5-7625:
A 6-year-old boy presents with a linear area of swelling in the region of the posterior paraspinous muscles. He has no history of weight loss or fevers. He also has bilateral great toe deformities consisting of valgus and shortening. The most likely diagnosis is:
1) Multiple hereditary exostoses
3) Osteosarcoma
2) Ewingâs sarcoma
5) Synovial sarcoma
4) Fibrodysplasia ossificans progressiva
Fibrodysplasia ossificans progressiva is characterized by linear swelling and progressive ossification from a posterior-central origin. Patients also have a characteristic shortened and valgus great toe. The natural history of this disorder is one of progressive ossification, and it eventually impairs nutrition.
Correct Answer: Fibrodysplasia ossificans progressiva 2384. (3843) Q5-7626:
Slide 1 Slide 2
Based on the presented radiographs (Slide 1, Slide 2), what is the proper diagnosis of this 12-month-old female patient:
1) Normal study
3) Developmental coxa vara of the right femur
2) Avascular necrosis of the femoral head
5) Osteogenesis imperfecta
4) Physeal fracture
This patient had a physeal fracture of the right proximal femur, which was the result of a non-accidental injury. Notice the displacement of the right femoral metaphysic anteriorly on the frog view despite comparable degrees of rotation. A metaphyseal calcification is apparent posterior to the femoral neck. Treatment involved spica cast application.
Correct Answer: Physeal fracture
Slide
A 23-month-old patient is evaluated for intoeing. Based on his radiograph (Slide), what is the most likely diagnosis:
1) Osteogenesis imperfecta
3) Physiologic bowing
2) Tibial torsion
5) Fibrous dysplasia
4) Rickets
This patient has rickets. Note the diffuse osteopenia, âgrainyâ nature, thinning of the tibial and femoral diaphyseal cortex, varus of both the proximal and distal tibial metaphyses, and widened and irregular physes at all levels.
Correct Answer: Rickets 2386. (3845) Q5-7628:
Slide 1 Slide 2 Slide 3
A 13-year-old boy has been experiencing hip pain for 1 month. The pain is worse with activity. The patient has no history of fever or weight loss, and both of his hips flex to 90°. Based on his radiograph (Slide 1) and magnetic resonance image (Slide 2), which of the following is the most likely diagnosis:
1) Leukemia
3) Psoas tendonitis
2) Pincer-type impingement
5) Avascular necrosis
4) Slipped epiphysis
This patient has a subtle grade 1 slipped capital femoral epiphysis. He is a male Risser 0 and is overweight. He has decreased internal rotation of the involved hip in flexion. The plain pelvis radiograph shows slightly less epiphyseal height on the left side, slight posterior metaphyseal overlap, and increased irregularity of the physis. MRI confirms a lucent plane in the physis and surrounding edema. A plain film (Slide 3) confirms the diagnosis. In situ fixation was performed on this patient.
Correct Answer: Slipped epiphysis
Slide 1 Slide 2 Slide3
A 13-year-old boy has been experiencing hip pain for 1 month. The pain is worse with activity. The patient has no history of fever or weight loss, and both of his hips flex to 90°. Based on his radiograph (Slide 1) and magnetic resonance image (Slide 2), which of the following is the most likely diagnosis:
1) Leukemia
3) Psoas tendonitis
2) Pincer-type impingement
5) Avascular necrosis
-
Slipped epiphysis
This patient has a subtle grade 1 slipped capital femoral epiphysis. He is a male Risser 0 and is overweight. He has decreased internal rotation of the involved hip in flexion. The plain pelvis radiograph shows slightly less epiphyseal height on the left side, slight posterior metaphyseal overlap, and increased irregularity of the physis. MRI confirms a lucent plane in the physis and surrounding edema. A plain film (Slide 3) confirms the diagnosis. In situ fixation was performed on this patient.
Correct Answer: Slipped epiphysis 2388. (3847) Q5-7630:
Slide
A 9-year-old girl sustains an injury (Slide) as a result of a fall. What is her risk of avascular necrosis:
1) 95%
3) 50%
2) 75%
-
5%
4) 25%
This patient has a displaced basicervical fracture of the femur that is classified as Delbet type III. The risk of avascular necrosis is approximately 20% to 25%. She should be monitored closely for the first 12 to 18 months after injury for this complication.
Correct Answer: 25%
Slide 1
A 9-year-old girl sustains an injury (Slide 1) as a result of a fall. What is the recommended treatment:
-
Traction
3) Closed reduction and Steinmann pin fixation
2) Closed reduction and spica cast application
5) Open reduction and fixation with hip screw
4) Open reduction and Steinmann pin fixation
This patient has a basicervical fracture, which is prone to development of varus. Traction, spica cast, and Steinman pin fixation do not provide adequate control of this process. Steinmann pins do not allow compression across the fracture site. Closed or open reduction and internal fixation with a compression screw fixation, preferably a hip screw, provide the best control of the fracture. A hip screw provides the best protection against the development of varus. If the fracture is not opened, then aspiration of the hematoma is recommended.
Correct Answer: Open reduction and fixation with hip screw 2390. (3849) Q5-7632:
Slide
A 6-year-old boy presents with a 3-month history of medial midfoot pain. Based on his radiograph (Slide), which of the following is the correct diagnosis:
1) Kohlerâs disease
3) Eosinophilic granuloma
2) Pannerâs disease
5) Freibergâs disorder
4) Accessory navicular
This child has avascular necrosis of the tarsal navicular. This is an idiopathic phenomenon more common in boys of this age than girls. The condition responds to rest and orthotics or cast immobilization. The navicular revascularizes over the ensuing year.
Correct Answer: Kohlerâs disease
Which of the following syndromes has the highest incidence of scoliosis as a percentage of affected patients:
1) Marfan syndrome
3) Down syndrome
2) Loeys-Dietz syndrome
5) Prader-Willi syndrome
4) Rett syndrome
Marfan syndrome, due to a mutation in fibrillin-1, causes spinal deformity in two-thirds of patients. Rett syndrome, caused by a mutation in MECP-2, results in neurological balance disorder and may cause scoliosis in up to half of affected patients. Scoliosis is seen in smaller numbers of patients with Prader-Willi, Down, and Loeys-Dietz syndromes.
Correct Answer: Marfan syndrome
2392. (254) Q6-367:
Acute pain that presents in the lateral arm and shoulder is suggestive of:
1) Cervical myelopathy
3) Radial tunnel syndrome
2) Thoracic outlet syndrome
5) Diabetic neuropathy
4) Cervical disk herniation
Acute radiculopathies of the upper extremity are suggestive of soft cervical disk herniations. Three types of soft disk herniations have been described:
Intraforaminal is the most common and is often evidenced by radicular symptoms in a dermatomal distribution. Posterolateral herniation results in predominantly motor symptoms.
Midline disk herniations may result in myelopathy.
Correct Answer: Cervical disk herniation
2393. (255) Q6-368:
A cervical disk herniation that causes weakness in the wrist extensors will likely produce sensory changes in the:
1) Lateral arm
3) Radial forearm
2) Ulnar forearm
5) Hypothenar eminance
4) Long finger
The wrist extensors are innervated by C 6. Weakness would likely be the result of a C 5-C 6 cervical disk herniation, thus causing a C 6 radiculopathy. The dermatome of C 6 includes the radial side of the forearm and radial digits.Correct Answer: Radial forearm
2394. (256) Q6-369:
In order to diagnose a disk herniation, the preferred test is:
1) Electromyogram
3) Computerized tomography scan
2) Flexion-extension roentgenograms
5) MRI without gadolinium
4) Magnetic resonance imaging (MRI) with gadolinium
Magnetic resonance imaging (MRI) has become the modality of choice for diagnosis of disk herniation. It is readily available, noninvasive, and has proven as accurate as post-myelogram computerized tomography scans in the evaluation of cervical radiculopathy. An MRI study should include a T1- and T2-weighted image sequence with both sagittal and axial images.Correct Answer: MRI without gadolinium
1) Nonsteroidal anti-inflammatory drugs
3) Physical therapy
2) Cervical immobilization
5) Home traction
4) Anterior cervical discectomy and fusion
Initial management of cervical radiculopathy is nonoperative. This includes: Nonsteroidal anti-inflammatory drugs
Occasional narcotic analgesic medications Immobilization in a soft collar for 10 to 14 days Physical therapy with emphasis on isometric exercises Home traction
Surgical intervention is indicated for patients who fail nonoperative treatment. Motor deficits intertractable pain may serve as indication for early surgical therapy. In addition, patients with intertractable pain or motor deficits may be recommended for early surgery.Correct Answer: Anterior cervical discectomy and fusion
2396. (258) Q6-371:
Figure 1
This patient has a wide-based gait and hyper-reflexia in his lower extremities. His magnetic resonance image is presented. Other physical findings in this patient would likely include:
1) Negative Hoffman sign
3) Carpal tunnel syndrome
2) Negative Babinski sign
5) Thoracic outlet syndrome
4) Hand intrinsic wasting
This patientâs clinical presentation is suggestive of cervical spondylotic myelopathy. The T2 sagittal magnetic resonance image shows multiple areas of spinal cord compression due to spondylosis. In this situation, one sees evidence of upper motor neuron dysfunction in the lower extremites such as hyper-reflexia, Hoffman sign, clonus, wide-based gait, and sexual or bladder dysfunction. The upper extremities may demonstrate upper and lower motor neuron findings, hypo-reflexia or hyper-reflexia, crossed radial reflex, Hoffman sign, and hand intrinsic wasting.Correct Answer: Hand intrinsic wasting
2397. (259) Q6-372:
A useful test to differentiate cervical radiculopathy from diabetic peripheral neuropathy is:
1) Hemoglobin A1C
3) Computerized tomography-myelogram
2) Magnetic resonance imaging
5) Cervical flexion/extension roentgenograms
4) Electrodiagnostic testing
An electromyogram detects motor changes as a result of nerve compression. It can be used to differentiate cervical radiculopathy from peripheral neuropathy.Correct Answer: Electrodiagnostic testing
1) Recurrent laryngeal nerve
3) Hypoglossal nerve
2) Superior laryngeal nerve
5) Esophagus
4) Thoracic duct
On the left side of the neck, the thoracic duct ascends lateral to the carotid sheath and is at potential risk for laceration with resulting chylothorax. This potential risk must be avoided by remaining medial to the carotid sheath during the left-sided anterior cervical spine approach.Correct Answer: Thoracic duct
2399. (261) Q6-374:
With a right-sided anterior cervical spine approach, the structure at increased risk is the:
1) Recurrent laryngeal nerve
3) Hypoglossal nerve
2) Superior laryngeal nerve
5) Esophagus
4) Thoracic duct
The left recurrent laryngeal nerve enters the thorax within the carotid sheath before looping around the aortic arch and ascending into the neck between the trachea and the esophagus. On the right side, the nerve exits the carotid sheath at a higher level, making the nerve susceptible to injury during the surgical dissection.Correct Answer: Recurrent laryngeal nerve
2400. (262) Q6-375:
The structure most at risk during lateral mass screw placement is the:
1) Vertebral artery
3) Spinal cord
2) Nerve root
5) Thoracic duct
4) Recurrent laryngeal nerve
Lateral mass screws placed in the cervical spine for plate fixation are directed 30° lateral and 15° cephalad from a point 1 mm medial to the mid-portion of the particular pillar. The nerve root exits at the anterolateral portion of the facet joint and is at risk of injury. The vertebral artery may be injured in screws placed too medially. The spinal cord is essentially free of danger with accepted techniques.Correct Answer: Nerve root
2401. (263) Q6-376:
After undergoing an anterior cervical discectomy and fusion through a left-sided neck incision, the patient is noted to have a drooping eyelid and a right pinpoint pupil. This is likely due to:
1) Spinal cord injury
3) Retractor placement
2) Nerve root injury
5) Vertebral artery injury
4) Carotid sheath compression
This patient has postoperative Horner syndrome. The sympathetic chain lies lateral to the longus colli muscles. Retractors must be placed deep into these muscles. Retractors placed ventrally to the longus colli muscles can cause injury to the sympathetic chain, esophagus medially, and carotid sheath contents laterally.Correct Answer: Retractor placement
After an anterior cervical discectomy and fusion with autogenous iliac crest bone graft, the patient reports numbness in the lateral thigh. This is due to:
1) Nerve root injury
3) Lateral femoral cutaneous nerve injury
2) Intraoperative positioning
5) Sciatic nerve injury
4) Genitofemoral nerve injury
The lateral femoral cutaneous nerve emerges from the lateral border of the psoas major muscle and crosses the ilium as it runs toward the anterior superior iliac spine. The course of the nerve is variable. The nerve is at risk of injury, with resulting meralgia paresthetica, in approximately 10% of patients.Correct Answer: Lateral femoral cutaneous nerve injury
2403. (265) Q6-378:
One advantage of posterior laminoforaminotomy in the treatment of cervical radiculopathy is:
1) Obviates the need for fusion
3) Easier access to midline disk herniations
2) Improves nerve root decompression
5) Improves postoperative alignment
4) Improves wound healing
The posterior cervical foraminotomy has a surgical success rate similar to an anterior cervical discectomy and fusion. Proponents argue that the posterior procedure obviates the need for fusion, therefore, postoperative immobilization is unnecessary. The posterior approach cannot address segmental kyphosis or recreate disk space height.Correct Answer: Obviates the need for fusion
2404. (266) Q6-379:
The most frequently involved spinal segment in rheumatoid arthritis is:
1) C 1-C 2
3) C 5-C 6
2) C 7-T1
5) L5-S1
4) T12-L1
C 1-C 2 is the most frequently involved spinal segment in rheumatoid arthritis (RA), and it is also the most clinically significant. These articulations are exclusively synovial and the primary target of RA.Correct Answer: C 1-C 2
2405. (267) Q6-380:
The most useful measurement for predicting neurological deficit in rheumatoid arthritis involvement of the cervical spine is:
1) Flexion angle
3) Anterior atlantodens interval (AADI)
2) Posterior atlantodens interval (PADI)
5) C 2-C 3 disk height
4) Lordosis angle
The posterior atlantodens interval (PADI) is the distance between the posterior surface of the dens and the anterior edge of the posterior ring of C 1, as seen on a lateral radiograph. A PADI < 14 mm was 97% sensitive in predicting the presence of neurological deficit. Patients with a PADI >14 mm had a 94% chance of being neurologically intact.Correct Answer: Posterior atlantodens interval (PADI)
A relative contraindication to cervical laminectomy for the treatment of cervical spondylosis is:
1) Positive Babinski sign
3) Positive Hoffman sign
2) Cervical lordosis
5) Cervical kyphosis
4) Wrist extensor weakness
Cervical laminectomy for spondylosis is performed to allow the spinal cord to migrate posteriorly in order to decompress the spine. This can be accomplished only if the spine is lordotic. If kyphosis exists, then the cord may remain draped over anterior osteophytes and continued compression may exist. In cases of cervical kyphosis, anterior decompression is preferred.Correct Answer: Cervical kyphosis
2407. (269) Q6-383:
The major risk factor for nonunion in a type 2 odontoid fracture is:
1) Age >35 years old
3) Anterior displacement >5 mm
2) Posterior displacement >5 mm
5) Flexion 30°
4) Smoking history
A posterior displacement >5 mm has the greatest risk of nonunion. However, age >65 years old is a next risk factor. Type 2 odontoid fractures have the highest rate of nonunion of the 3 types and there has been reported to be >60% nonunion when not treated with a halo immobilization. Halo traction is a viable alternative for a patient who has minimal displacement and/or is not a candidate for surgery.Correct Answer: Posterior displacement >5 mm
2408. (270) Q6-384:
A 48-year-old man presents with a closed head injury requiring intubation and isolated bilateral facet dislocation. The next appropriate step is:
1) Closed reduction with axial traction with Gardner-Wells tongs
3) Emergent magnetic resonage imaging
2) Posterior open reduction and posterior cervical platting
5) Administration of Decadron (Merck & Co., West Point, Pa.) 10 mg/hr intravenously
4) Neurostabilization until the patientâs neurologic status improves
The patient has a severe closed head injury and is unable to tolerate a close reduction maneuvers with Gardner-Wells tongs. Emergent magnetic resonance imaging should be obtained to evaluate the potential presence of a disk herniation at the dislocation. Should a disk herniation be present, anterior approach and diskectomy should be performed prior to reduction maneuvers.Correct Answer: Emergent magnetic resonage imaging
2409. (271) Q6-385:
A major indication for surgical decompression of an L1 burst fracture is:
1) Loss of anterior body height of 60%
3) Kyphosis of 15°
2) Retropulsion of canal fragments to 50% of canal size
5) Presence of a posterior lamina fracture
4) Post-void residual of 450 mL
Generalized treatment algorithms for burst fractures involving upper lumbar spine have relative indications for surgery that include 50% loss of height, 25% of kyphosis, and 50% canal compromise. Absolute indications for decompression include neurological deficits including a potential conus injury. Post-void residual of > 450 mL is suggestive of sacral root injury at the level of conus. Bradford suggests that anterior decompression of this injury has favorable outcome with frequent resolution or improvement of symptoms.Correct Answer: Post-void residual of 450 mL
2410. (272) Q6-386:
An injury associated with a type 1 fracture of the odontoid is:
1) Concomitant fracture of the body at C 2
3) Atlanto-occipital dislocation
2) Burst fracture of the lumbar spine
5) Associated Jefferson fracture of the ring of C 1
4) Rupture of the transverse ligament
Type 1 fractures are a rare entity. They are frequently treated with immobilization with a hard collar if isolated. There have been numerous reports in the literature of a type 1 fracture of the odontoid being associated with an atlanto-occipital dislocation, and this injury must be suspected. The potential for missing atlanto-occipital dislocation may lead to a fatal outcome.Correct Answer: Atlanto-occipital dislocation
2411. (273) Q6-387:
A type 3 traumatic spondylolisthesis of the axis, as classified by Levine and Edwards, is best treated with which of the following:
1) Soft collar immobilization
3) Halo vest immobilization
2) Hard Philadelphia cervical orthosis
5) Gardner-Wells tongs application and awake reduction, then posterior stabilization
4) Open reduction and operative posterior stabilization
The Levine classification of traumatic spondylolisthesis or Hangman fractures involving C 2 in the type 3 injury has a combined bilateral facet dislocation at C 2-C 3 as well as the traumatic spondylolisthesis of the axis. Closed reduction could not be performed secondary to the traumatic spondylolisthesis at the C 2 isthmus.Correct Answer: Open reduction and operative posterior stabilization
2412. (274) Q6-389:
A type 3 Anderson and Montensano fracture of the occipitocondyle is best described as:
-
An impacted comminuted fracture of the occipital condyle
-
An occipital condyle fracture associated with a basilar skull fracture
5) A crush injury to the occipital condyle in the face of underlined osteoarthritis
4) An occipital condyle avulsion fracture from the alar ligament
) An occipital condyle-axial dislocation
A type 3 fracture of the occipital condyle is an avulsion fracture from traction of the alar ligament. This usually results from a rotation, lateral bending moment, or combined injury. If the injury is severe, both alar ligaments may be involved and occipitocervical instability may occur. Treatment for a type 3 injury would include a rigid or collar orthosis for 8 to 12 weeks, possible halo mobilization, and, if ligamentous instability has occurred, posterior cervical fusion.Correct Answer: An occipital condyle avulsion fracture from the alar ligament
An 8-year-old boy has had torticollis for approximately 5 weeks. He has undergone immobilization with a cervical collar without success. The patient has not undergone traction and now has atlantoaxial rotatory subluxation. The best treatment for this patient should be:
1) Continued treatment in hard cervical orthosis
3) Hospitalization with cervical traction
2) Manipulation and closed reduction of his deformity
5) Application of halo vest and reduction
4) Posterior C 1-C 2 fusion
The treatment protocol for atlantoaxial rotatory subluxation is based on the onset and length of time of deformity. Soft collar treatment for this deformity is best if treated within 1 week of onset. For deformities lasting up to 1 month, in-house hospitalization with traction is warranted. However, the success of this treatment declines markedly after 1 month, at which time posterior C 1-C 2 fusion is warranted.Correct Answer: Posterior C 1-C 2 fusion
2414. (276) Q6-391:
During posterior cervical plating, several techniques can be employed. The recommended lateral mass screw position is:
1) 10° laterally, 90° perpendicular to the lateral mass
3) 30° laterally and 15° cephalad
2) 50° cephalad and 30° laterally
5) 60° laterally and 30° medially
4) 15° laterally and 30° cephalad
Surgical technique for cervical lateral mass fixation as described by An and colleagues is 30° of Lateral angulation and 15° of angulation cephalad to the facet joint.. This has been described as the safest recommended technique for lateral mass screw placement.Correct Answer: 30° laterally and 15° cephalad
2415. (277) Q6-392:
A 35-year-old man presents 3 years after a motor vehicular trauma. It is now 3 years following operative stabilization of the spine at C 7. He complained of mild weakness in his right upper extremity at the biceps level and has corresponding parasthesias in the right thumb. The next step in the evaluation of this patient is:
1) Anteroposterior lateral flexion extension radiographs of the cervical spine
3) Magnetic resonance imaging of the cervical spine
2) Computerized tomography scan of the cervical spine
5) Anti-inflammatory medication for presumed tendonitis
4) Physical therapy with range of motion and strengthening exercises of both upper extremities
The patient is a 35-year old man has been stable since his injury. The most important evaluation for this individual would be magnetic resonance imaging to rule out potential cervical cord syrinx that has occurred given new onset weakness and sensory changes proximal to his injury.Correct Answer: Magnetic resonance imaging of the cervical spine
A 55-year-old man with ankylosing spondylitis has a minor fall and is suffering with neck pain. Anteroposterior and lateral radiographs are negative with no evidence of fracture. He has no neurologic loss and has normal strength with the exception of severe restricted motion. Twelve hours following injury, he is found to have bilateral bicep and tricep weakness. The appropriate management and the work up of this individual is:
1) Computerized tomography (CT) anteroposterior lateral radiographs of the cervical spine
3) Magnetic resonance imaging (MRI) of the cervical spine
2) CT scan of the cervical spine
5) Electromyogram to better delineate all the nerve neuropathy
4) Bone scan of the MRI
The patient is within 12 hours of having normal cervical spine films. Approximately one third of patients with ankylosing spondylitis incur occult injuries to the cervical spine that are not identified by plain films prior to kyphotic progression. A bone scan would delineate a fracture after 72 hours. However, the presence of progressive weakness should raise suspicion of a potential epidural hematoma. For this reason, magnetic resonance imaging would better delineate epidural hematoma.Correct Answer: Magnetic resonance imaging (MRI) of the cervical spine
2417. (279) Q6-394:
A 2-year-old boy with a congenital heart anomaly has a 40° thoracolumbar curvature. Standing posteroanterior and lateral radiographs reveal vertebral anomalies indicative of congenital scoliosis. Which of the following patterns of congenital scoliosis has the worst prognosis for progression?
1) Block vertebrae
3) Fully segmented hemivertebra
2) Unilateral unsegmented bar
5) Nonsegmented hemivertebra
4) Unilateral unsegmented bar with a contralateral fully segmented hemivertebra
Congenital spinal deformity is caused by structural abnormalities in the vertebrae that can result in asymmetric growth, such as scoliosis or kyphosis. It has been classified in 2 types. Type I involves defects of formation and type II involves defects of segmentation. However, in many instances, deformities can be a mixture of both. Defects of formation include segmented or unsegmented hemivertebrae and wedge vertebrae. Defects of segmentation include block vertebrae, unilateral bars, or unilateral bars with hemivertebrae.
The potential for progression is dependent on the growth potential of the anomalies. The presence of healthy-appearing disks between the hemivertebra and its normal counterparts indicates good growth potential and risk for progression. A unilateral bar on the opposite side of a segmented hemivertebra acts as a tether on the concave side of the curve and has the most likelihood for progression.
Children with congenital scoliosis also have a significant incidence of associated anomalies, both intraspinal and other organ systems. About 30% have a spinal dysraphism such as diastematomyelia, meningocele or lipoma. Other associated anomalies include Klippel-Feil syndrome (25%), genitourinary tract abnormalities (30%), cardiac defects (12%), and Sprengelâs deformity (10%).
Correct Answer: Unilateral unsegmented bar with a contralateral fully segmented hemivertebra
Six months ago, an 11-year-old premenarchal girl with adolescent idiopathic scoliosis had a right thoracic curve from T5 to T12 measuring 20°. Her physical examination was normal. She returned to the office and a standing posteroanterior radiograph demonstrates a 28° right thoracic curve from T5 to T12; she is Risser stage 0. A lateral radiograph shows a thoracic kyphosis of 10°. At this time, you recommend:
1) Repeat radiograph in 6 months
3) Full-time use of a thoracolumbosacral orthosis
2) Thoracic flexibility exercises
5) Posterior spinal fusion with instrumentation
4) Electrical stimulation
In skeletally immature patients with adolescent idiopathic scoliosis and curves approaching 30° with documented progression, bracing may be effective at preventing further progression of the curve. Risk of progression in adolescent idiopathic scoliosis is related to curve magnitude and remaining growth potential. The risk of further progression in this patient is 68%, and bracing is indicated. Electrical stimulation and physical therapy have not been shown to affect the natural history of scoliosis. Surgery may be indicated in patients with more severe curves. In the sagittal plane, hypokyphosis is usually present in adolescent idiopathic scoliosis.Correct Answer: Full-time use of a thoracolumbosacral orthosis
2419. (281) Q6-396:
A 15-year-old boy with adolescent idiopathic scoliosis has a right thoracic curve from T5 to T11 measuring 45° and a left thoracolumbar curve from L1 to L4 measuring 32°. He is Risser stage 2 and has a hypokyphotic thoracic spine. Bending films demonstrate moderate flexibility in the lumbar curve. He was prescribed a thoracolumbosacral orthosis since age 14, but his scoliosis has progressed. His physical exam reveals a prominent right rib hump and mild right shoulder elevation. His head is centered above his pelvis. His neurological examination is normal. You recommend:
1) Continued full-time use of the orthosis until skeletal maturity
3) Repeat evaluation in 6 months
2) Discontinuation of the orthosis due to failure
5) Posterior spinal fusion of the thoracic curve only with instrumentation
4) Posterior spinal fusion T5-L4 with instrumentation
This patient has a right thoracic curve with a compensatory left lumbar curve pattern of adolescent idiopathic scoliosis. There has been documented progression into the surgical range despite bracing, and he still has some growth remaining. Surgical intervention is indicated. This curve pattern (King II, Lenke D) can be approached posteriorly with thoracic fusion alone to the neutral and stable vertebra and instrumentation to obtain and maintain correction. The unfused lumbar curve will spontaneously correct to balance the fused thoracic curve. Care must be taken to avoid fusion into the lower lumbar spine and preserve motion segments.Correct Answer: Posterior spinal fusion of the thoracic curve only with instrumentation
A 1-year-old male infant is referred by his pediatrician for evaluation of possible scoliosis. Otherwise, he is healthy. His physical exam reveals normal neurologic function, plagiocephaly and a flexible thoracic curve. Radiographs reveal a left thoracic curve with a Cobb angle of 36° and no vertebral anomalies. The apical ribs are in Phase I, and the rib-vertebral angle difference is 18°. At this time, management should include:
1) Observation
3) Full-time use of a thoracolumbosacral orthosis
2) Serial body casting to obtain correction
5) Spinal instrumentation without fusion
4) Posterior spinal fusion
Infantile idiopathic scoliosis is rare in this country and not well understood. It is more common in Europe, occurs more frequently in boys, and left thoracic curves predominate. Plagiocephaly, or a flattening of the posterior skull on the convex side of the spinal curvature, is frequently found in these patients, suggesting a postural cause of both.
There are 2 types of infantile idiopathic scoliosis: resolving and progressive. Distinguishing between the 2 types has obvious consequences regarding prognosis and treatment. Prior to Mehtaâs work, identification of the type of infantile idiopathic scoliosis was difficult, because it was not related to curve magnitude, age at onset, rate of progression, or degree of rotation.
Mehta showed that the 2 groups were distinguishable by the relationship of the ribs to the apical vertebral body on the posteroanterior radiograph. Ribs that do not overlap the vertebral body are in Phase I, and ribs that do overlap the vertebral body on the convexity of the curve are in Phase II. The rib-vertebral angle is constructed by the intersection of a line perpendicular to the apical vertebral endplate with a line drawn along the long axis of the corresponding rib. The rib-vertebral angle (RVA) difference is the difference of the RVA of the concave and convex ribs of the apical vertebra. In scoliosis, the convex ribs form a more acute angle than the concave ribs, so this difference is >0. Mehta concluded that curves in which the ribs are in Phase I and the RVA difference is < 20° have a better prognosis (resolving type) and require just observation. Treatment for progressive curves includes serial body casts, orthoses, or surgery for severe curves. Various surgical approaches include posterior spinal fusion, instrumentation without fusion to allow spinal growth, or anterior convex hemiepiphysiodesis with posterior hemiarthrodesis.Correct Answer: Observation
2421. (283) Q6-398:
The most appropriate indication, after scoliosis curve progression, for a posterior spinal fusion with segmental instrumentation to the pelvis in a severely involved spastic quadriplegic child with cerebral palsy is:
1) Pelvic obliquity
3) Poor nutritional status
2) Deterioration in function
5) Non-ambulatory status
4) Normal pulmonary function
Patients with a spastic quadriplegic pattern of cerebral palsy have higher than 25% incidence of scoliosis. This neuromuscular scoliosis differs from that of idiopathic scoliosis in that it is usually a long C-shaped thoracolumbar curve that may involve the pelvis. Frequently, posterior spinal fusion from T1 to the sacrum is required with rigid segmental instrumentation with stabilization to the pelvis (a unit rod).
Indications for fusion in these patients include curve progression and loss of function. This can include loss of sitting ability, poor pulmonary function due to poor pulmonary toiletting, and recurrent infection such as decubitus ulcers. These children are most often non-ambulators and are dependent on wheelchair sitting supports for postural control.
Correct Answer: Deterioration in function
A 12-year-old boy with Duchenne muscular dystrophy has a 25° curve in the thoracolumbar spine with moderate pelvic obliquity. His pulmonary function tests are 70% of predicted function. He uses a wheelchair for ambulation, but is able to stand for transfers. Management should include:
1) Observation, with repeat radiograph and pulmonary function tests in 6 months
3) Wheelchair seat pressure mapping and lateral trunk support modifications
2) Thoracolumbosacral orthosis
5) Anterior spinal release and posterior spinal fusion with instrumentation
4) Posterior spinal fusion with instrumentation
Scoliosis in patients with Duchenne muscular dystrophy typically becomes progressive when ambulation ceases. For curves >20Â
°, posterior spinal fusion with instrumentation is indicated. Fixation to the pelvis is necessary to improve sitting if pelvic obliquity is present. Severe, collapsing scoliosis can result without operative intervention and can result in diminishing pulmonary function and loss of sitting ability. Surgery can be safely undertaken if pulmonary function remains >40% of predicted function, but anterior surgery causes morbidity on an already compromised pulmonary system. Nonoperative treatment such as orthoses or trunk supports offer little in the way of controlling progression and are generally not well tolerated by these patients.Correct Answer: Posterior spinal fusion with instrumentation
2423. (285) Q6-400:
A 6-year-old girl with a lumbar level paraplegia secondary to myelomenigocele presents with a rapidly progressive thoracolumbar scoliosis. The most accurate test to determine the etiology of the spinal deformity is:
1) Bone scan
3) Magnetic resonance imaging of the brain and spinal cord
2) Lumbar puncture and cerebrospinal fluid analysis
5) Computerized tomography scan of the spine at the lumbosacral junction
4) Lateral flexion/extension radiographs of the cervical spine
Children with myelodysplasia are at risk for scoliosis, but a rapidly progressive curve should alert the physician to aggressively investigate the etiology. Causes include:
Tethered cord Syringomyelia Shunt failure
Progressive hydromyelia Arnold-Chiari malformation
All of these conditions can be readily diagnosed by a magnetic resonance imaging scan of the brain and spinal cord. Radiologic imaging such as radiographs, computerized tomography scans, or bone scans for bony lesions is rarely helpful. Cerebrospinal fluid analysis is unlikely to reveal an answer regarding scoliosis.
In the presence of a working ventriculoperitoneal shunt, the most likely etiology in this child is a tethered cord. Increased lumbar lordosis, back pain, or an increase in lower root level spasticity should alert the clinician to the possible presence of a tethered cord.Correct Answer: Magnetic resonance imaging of the brain and spinal cord
An 11-year-old girl presents with low back pain for 2 monthsâ duration. She is an elite gymnast and has missed 2 meets because of the pain. Physical exam reveals pain with hyperextension of the lumbar spine. Her neurological exam is normal. Radiographs of the lumbar spine, including oblique views, are normal. The recommendation is:
1) Rest, with slow return to training in 4 weeks
3) Magnetic resonance imaging of the spinal cord
2) Custom lumbosacral orthosis
5) Bone scan with single photon emission computed tomography imaging
4) Physical therapy exercises
Athletes involved in sports requiring repetitive hyperextension or rotation of the lumbar spine are susceptible to stress fractures of the pars interarticularis or spondylolysis. Two months of insidious back pain warrants a diagnostic work up, and radiographs may be nondiagnostic in the early period. A bone scan with single photon emission computed tomography will confirm the diagnosis in a patient with a history and physical findings of spondylolysis. Magnetic resonance imaging is rarely helpful in the diagnosis of this bony lesion, but it may be the next diagnostic modality if the bone scan was negative and the pain continued. Rest with immobilization is usually the first line of treatment for spondylolysis. In cases of refractory pain, controversy exists in the surgical management of this condition. Some authors favor repair of the lytic defect and others prefer a posterolateral fusion.Correct Answer: Bone scan with single photon emission computed tomography imaging
2425. (287) Q6-402:
An 8-year-old girl presents with back pain and an abnormal gait. She walks with externally rotated feet and limited hip flexion. She has a palpable step-off at the lumbosacral junction and hamstring tightness. Radiographs of the lumbosacral spine demonstrate a dysplastic spondylolisthesis with a slip angle of 55° and slippage of 60% of L5 on S1. The recommended course of treatment is:
1) Posterolateral spinal fusion
3) Physical therapy
2) Epidural steroid injection
5) Observation with repeat radiographs in 6 months
4) Lumbosacral orthosis
Spondylolisthesis is the forward slipping of 1 vertebra on the next caudal vertebra. Spondylolisthesis in children can be classified into 2 types: isthmic and dysplastic. Isthmic spondylolisthesis is an entity in which there is a lesion in the pars interarticularis that permits forward slippage; the articular facets are normal. Dysplastic or congenital spondylolisthesis implies that there is a congenital deficiency in the L5-S1 facet that allows forward slipping. There is no defect or elongation in the pars.
Growing children, particularly females, with dysplastic spondylolisthesis are at risk for further progression. This patientâs abnormal gait is due to hamstring tightness, probably due to the lumbosacral instability and nerve root irritation. With a grade III slip, back pain, and an abnormal gait, this patient is a candidate for an in situ posterolateral spinal fusion. Use of instrumentation is controversial, especially because the long-term results of in situ noninstrumented fusions are superior.Correct Answer: Posterolateral spinal fusion
The following can be found in the examination and radiographs of a child with Scheuermann disease:
1) Schmorl nodes
3) Anterior wedging 3 or more vertebrae
2) Back pain
5) All of the above
4) Thoracic kyphosis
Scheuermann disease is increased thoracic kyphosis, usually rigid, occurring in adolescent males. The etiology is unknown, but has included theories dealing with avascular necrosis of the ring apophysis, growth plate abnormalities, biologic and mechanical causes. The classic definition is increased thoracic kyphosis (>45°) with 5° or more of anterior wedging at 3 sequential vertebrae. Other radiographic abnormalities include:
Endplate irregularities Spondylolysis
Compensatory lumbar hyperlordosis Schmorlâs nodes
Hamstring tightness and rigid thoracic kyphosis is noted on physical examination, and neurological function is normal.
Treatment consists of bracing in skeletally immature patients with a thoracolumbosacral orthosis, but many adolescent male patients are noncompliant with bracing. In the skeletally mature patient with pain and severe deformity (>65° of kyphosis), posterior spinal fusion with instrumentation is indicated. Occasionally, anterior diskectomy and interbody fusion with posterior fusion and instrumentation are required for severe deformity correction.
Postural kyphosis is also common in adolescent males, but the vertebral changes are not present, and the deformities are usually more supple. Treatment is hyperextension exercises.
Correct Answer: All of the above
2427. (289) Q6-404:
The natural history of which of the following spinal deformities in children carries with it the highest risk of paraplegia?
1) Congenital lordosis
3) Neuromuscular scoliosis
2) Congenital kyphosis
5) Postlaminectomy kyphosis
4) Idiopathic scoliosis
Congenital kyphosis, if left alone, is the most likely cause of paraplegia of all noninfectious spinal deformities. Defects of formation are more progressive than defects of segmentation, and paraplegia is common with defects that have an apex at T4-T9, the watershed area of spinal cord blood flow.
Treatment is usually surgical. There is no evidence of successful nonoperative treatment for congenital kyphosis. An early, limited posterior fusion, coupled with anterior growth, may result in a slow correction of the kyphosis. For kyphosis >55° in children older than 5 years of age, anterior and posterior spinal fusions are necessary. The tethering structures anteriorly must be released (anterior longitudinal ligament, annulus fibrosus) and distraction anteriorly is maintained by autogenous strut grafts.
Posteriorly, compression instrumentation is required with fusion. If neurological compromise exists preoperatively, magnetic resonance imaging is necessary to delineate the area of compression so that an anterior cord decompression may be performed successfully.Correct Answer: Congenital kyphosis
A 12-year-old girl presents to the clinic with scoliosis detected by school screening. Her past medical history includes ophthalmologic observation for Lisch nodules of the iris. She has just started her menstrual periods. On physical exam, she has axillary freckles and normal neurological function. Standing radiographs of the spine illustrate a 32° right thoracic curve from T4 to T10 and rib pencilling. In the sagittal plane, she has a thoracic kyphosis of 30°. The most likely diagnosis is:
1) Adolescent idiopathic scoliosis
3) Neurofibromatosis-1 (NF-1)
2) Congenital kyphoscoliosis
5) Stickler disease
4) Neurofibromatosis-2 (NF-2)
Neurofibromatosis (von Recklinghausen disease) is an autosomal dominant disorder that affects connective tissue. The most common type is NF-1, and is associated with primary skeletal disorders such as scoliosis, cortical thinning and pseudarthrosis of the tibia. It is the result of an abnormality on chromosome 17, and is also associated with:
Café au lait spots Neurofibromas
Axillary or inguinal freckling
Iris hamartomata (Lisch nodules)
Scoliosis in NF-1 can occur in 2 patterns. The first is similar to idiopathic scoliosis. The second, or dystrophic type is marked by short, sharper deformities, scalloping of the vertebral bodies, rib pencilling, enlarged foramina and severe apical vertebral body rotation. Some authors have demonstrated that curves characterized as idiopathic in childhood can take on dystrophic characteristics later in life and progress rapidly. Treatment is usually surgical.Correct Answer: Neurofibromatosis-1 (NF-1)
2429. (291) Q6-406:
A 3-year-old boy is referred by the pediatrician for neck stiffness. He has a mild hearing loss, but is otherwise healthy. On examination, his neck is rather short, and he has limitation of lateral rotation and bending, but flexion and extension are normal. There are no palpable bands in his neck. The anteroposterior and lateral cervical spine films ordered by the pediatrician show a congenital fusion of cervical vertebrae. The most likely diagnosis is:
1) Klippel-Feil syndrome
3) Congenital muscular torticollis
2) Fixed atlantoaxial rotatory subluxation
5) Axial neck pain
4) Arnold-Chiari malformation
The classic findings of Klippel-Feil syndrome include a short neck, low posterior hairline, and decreased neck range of motion, but
<50% of patients have all 3 elements of the triad. The neck motion is limited due to congenital fusion of cervical vertebrae, and the severity of cervical spine involvement usually heralds associated manifestations. Facial asymmetry, cranial nerve palsy, deafness, cardiac anomalies, and synkinesia may be detected in the involved child.
It is important to differentiate congenital muscular torticollis from Klippel-Feil syndrome, because releasing the sternocleidomastoid muscle will not correct a bony deformity. Static lateral radiographs of the cervical spine may appear normal in young children, as ossification of abnormal levels has not yet occurred. Flexion/extension lateral radiographs are useful to define congenital fusions, and magnetic resonance imaging may further delineate the anatomy. It is also important to test neck motion in all planes, because flexion/extension may be normal if movement occurs through just a few spared levels.
Children with Klippel-Feil syndrome rarely develop neurological symptoms as a result of the congenital cervical fusion. Later in life, they may develop neurological impairment as a result of instability or degenerative disk disease.
Correct Answer: Klippel-Feil syndrome
A 10-year-old boy with Down syndrome presents with his parents who have noticed that his endurance for walking seems to have decreased, and he seems clumsier. Your physical examination reveals generalized ligamentous laxity, but no other musculoskeletal abnormalities. His neurological examination is normal. His flexion/extension cervical spine radiographs are abnormal. The most likely pathophysiology is:
1) Os odontoideum
3) Klippel-Feil syndrome
2) Arnold-Chiari malformation
5) Transverse atlantal ligament insufficiency
4) Hypothyroidism
Children with Down syndrome (trisomy 21) have a higher incidence of hypothroidism, congenital heart disease, leukemia, and slipped capital femoral epiphysis. About 20% of children with Down syndrome develop atlantoaxial instability due to incompetence of the transverse atlantal ligament, and fortunately, most are asymptomatic.
Patients with Down syndrome should be screened for atlantoaxial instability with routine flexion/extension lateral cervical radiographs, especially prior to athletic participation. An atlanto-dens interval (ADI) of >5 mm should be treated with activity restriction in the absence of myelopathy. With symptoms of cervical myelopathy or an ADI >7 mm, an atlantoaxial arthrodesis is indicated.
Correct Answer: Transverse atlantal ligament insufficiency
2431. (293) Q6-408:
A 4-week-old female infant has congenital muscular torticollis. Which of the following is not associated with this condition?
1) Plagiocephaly
3) Developmental dysplasia of the hips
2) Cervical spine anomalies
5) Difficult delivery
4) Sternocleidomastoid muscle fibrosis
Congenital muscular torticollis is the most common cause of torticollis in the infant and young child. Usually, the children have a history of a breech or difficult delivery or primiparous birth. The exact etiology is unknown, but theories center around a compartment syndrome of the sternocleidomastoid muscle as a result of compression of soft tissues around the neck at the time of delivery. This results in fibrosis of the sternocleidomastoid muscle, tilting of the head to the ipsilateral side, and rotation of the head to the opposite side.
Congenital muscular torticollis is associated with developmental dysplasia of the hips in up to 20% of children, so a careful examination of hip stability is mandatory, with dynamic ultrasound, if necessary. Plagiocephaly or facial and skull deformities occur in progressive torticollis within the first year of life. The association of metatarsus adductus with congenital muscular torticollis is variable in the literature. Plain radiographs of the cervical spines of children with congenital muscular torticollis are always normal, with the exception of the head tilt and rotation.
Treatment initially includes stretching exercises and physical therapy early in life. Surgery (release of the muscle) is recommended if the torticollis persists after 1 year of age.Correct Answer: Cervical spine anomalies
A 13-year old boy presents to the emergency department with back pain of 5 daysâ duration. The pain is exacerbated by sitting or standing. He has a low-grade fever. He has pain on percussion of the lumbar spine. He has no tension signs. White blood cell count is 8000/mm3 and the erythrocyte sedimentation rate is 40 mm/hr. Plain radiographs of the spine demonstrate a narrowed intervertebral space at L3-L4. The most likely diagnosis is:
1) Epidural abscess
3) Vertebral osteomyelitis
2) Diskitis
5) Iliopsoas abscess
4) Pott disease
The symptoms of diskitis are often vague and insidious. This hematogenous infection of the disk space acts differently than other musculoskeletal infections. The presentation is often that of a patient with low back pain or refusal to ambulate. Fever is usually low or absent. The white blood cell count is usually normal, but the erythrocyte sedimentation rate or C-reactive protein levels may be elevated. Blood cultures are frequently negative. Radiographs of the spine may be normal initially, but may show intervertebral disk space narrowing or end plate irregularities. Bone scan and magnetic resonance imaging are also helpful in the diagnosis.
Treatment is usually conservative, and outcomes are aided by the fact that this condition is usually self-limiting. Rest and immobilization provide symptomatic relief, and many authors favor intravenous antibiotics. After an initial response in 72 hours or less, the patient can be switched to oral antibiotics for 3 to 5 weeks. A biopsy is indicated if the patient does not improve quickly, or if a tumor or abscess formation is suspected.
Patients with vertebral osteomyelitis or abscesses are typically more ill-appearing, have high fevers and white blood cell counts, and a markedly elevated erythrocyte sedimentation rate. Furthermore, a patient with an epidural abscess may have neurological symptoms or a positive straight leg raising test, due to nerve root irritation or spinal cord compression.
Correct Answer: Diskitis
2433. (295) Q6-410:
An 11-year-old boy sustains a fall while jumping on a trampoline. He has moderate back pain, an L-5 radiculopathy, and weakness of the right extensor hallucis longus. Radiographs and a computerized tomography scan of the lumbar spine demonstrate a slipped vertebral apophysis. The recommended treatment is:
1) Laminectomy and excision of annulus and vertebral bony margin
3) Thoracolumbosacral orthosis
2) Bed rest
5) Spinal traction
4) Physical therapy
This patient has a slipped vertebral apophysis as a result of trauma. This is analagous to a Salter-Harris type II fracture. A portion of the apophysis and annulus slip posteriorly and may impinge on the exiting nerve root. These usually do not resolve spontaneously or improve with conservative therapy, and excision is indicated. The disk fragments and retropulsed bone must be removed from the canal with a laminectomy for exposure.Correct Answer: Laminectomy and excision of annulus and vertebral bony margin
2434. (296) Q6-411:
Appropriate treatment of a nondisplaced Jefferson fracture is:
1) Hard cervical orthosis
3) Soft collar
2) Halo vest
5) Nerve treatment necessary
4) Posterior surgical stabilization
Fractures involving the C 1 or atlas are generally caused by axial compression with either a flexion or extension force. Generally, fractures involving the C 1 consist of multiple fragments. The classical Jefferson fracture is a 4-part fracture of the atlas and can be unstable. However, in this situation, a nondisplaced fracture represents a relatively stable injury. An open-mouth odontoid anteroposterior radiograph is frequently useful to evaluate unstable patterns. An unstable fracture typically has displacement of the lateral masses greater than 8 mm. If displacement of this amount occurs, generally, the transverse ligament has been disrupted and should be treated by halo vest immobilization. In this nondisplaced situation, a hard Philadelphia collar is the most appropriate form of treatment.Correct Answer: Hard cervical orthosis
2435. (297) Q6-412:
The American Spinal Injury Association has developed a classification of spinal cord injuries. Using this classification system, an Asia C injury is best described as:
1) Complete motor loss with incomplete sensation
3) Incomplete motor loss with some preservation of motor function with groups with less then grade 3 strength
2) Complete motor loss with complete sensation loss
5) Incomplete motor loss with 4+ strength and patchy sensation
4) Incomplete motor loss with normal bladder function
Asia C is an incomplete spinal cord injury with reservation of motor function with < grade 3 motor strength.Correct Answer: Incomplete motor loss with some preservation of motor function with groups with less then grade 3 strength
2436. (298) Q6-413:
Which of the following incomplete spinal cord injury syndromes has the most potential for recovery:
1) Anterior cord syndrome
3) Brown-Sequard syndrome
2) Central cord syndrome
5) Pyramidal syndrome
4) Posterior cord syndrome
Brown-Sequard syndrome is described as ipsilateral loss of motor function and contralateral loss of pain and temperature sensation. This syndrome is caused by penetrating injuries. Generally < 90% of patient who have this injury will recover ambulation.Correct Answer: Brown-Sequard syndrome
A 6-year-old boy has neck pain and stiffness following an upper respiratory tract infection. He presented with his head tilted to the right and turned to the left 3 weeks ago, but a soft cervical collar has not been beneficial. There is no known history of trauma. A computerized tomography scan shows rotatory subluxation of C 1 on C 2. The next step in the treatment of this child is:
1) Observation
3) In situ C 1-C 2 fusion posteriorly
2) Open reduction and C 1-C 2 fusion through an anterior approach
5) Hard cervical collar
4) Cervical traction
This child has torticollis as sequelae of an upper respiratory infection (Grisel syndrome) and rotatory subluxation (fixation) of C 1 on C 2. Other causes of torticollis include congenital muscular torticollis, neurogenic causes, Sandifer syndrome, Klippel-Feil syndrome, juvenile rheumatoid arthritis, and trauma. The common thread is that all of the etiologies appear to weaken, through inflammation or force, the supporting soft tissue structures of the atlantoaxial articulation. The diagnosis is made by dynamic CT scan.
Fielding classified atlantoaxial rotatory subluxation into 4 types:
Type I is a simple rotatory displacement without an anterior shift, and is the most common type in children.
Type II is rotatory fixation with anterior displacement >3 to 5 mm, and is associated with a deficiency of the transverse ligament and unilateral displacement of one lateral mass of the atlas.
Type III rotatory fixation there is anterior displacement >5 mm with bilateral displacement of the lateral mass with one side displaced more than the other. This is caused by a deficiency of both the transverse ligament and secondary ligament. Type IV is rotatory fixation with posterior displacement where the dens allows posterior shift of one or both of the lateral masses, and one shifting more than the other.
Types III and IV are rare but have potential for catastrophe and should be recognized to promptly initiate treatment.
Children with rotatory fixation of <1 week can be treated with a soft cervical collar and rest for 1 week. Most cases resolve, but close follow-up is necessary. If spontaneous reduction does not occur after 1-2 weeks, aggressive treatment is necessary.
Inpatient halter traction with judicious use of muscle relaxants and analgesics is recommended. Halo traction is necessary for reduction of longer standing (2-4 weeks) subluxation. Surgery is indicated in cases of neurological compromise, failure to achieve closed reduction, long-standing deformity (3 months or more), or recurrence following closed treatment. A Gallie-type fusion posteriorly is favored.
Correct Answer: Cervical traction
2438. (673) Q6-928:
A 40-year-old victim of a car accident was complaining of anterior chest pain. An x-ray of the chest showed no widening of the mediastinum and absence of pneumothorax. Lateral CXR revealed a fractured sternum with the proximal part of the fracture displaced posteriorly. Which of the following is the next step in the management of this patient?
1) Arterial blood gas
3) Chest bandage for the fractured sternum
2) Aortogram
5) Lateral x-ray of the thoracic spine with the patient supine
4) Computed Tomography of the chest
N/ACorrect Answer: Lateral x-ray of the thoracic spine with the patient supine
Figure 1 Figure 2
A 42-year-old male has a history of 6 months of pain in the lower thoracic region. Recently, the patient developed weakness in the right lower extremity, bladder and bowel movement. Plain x-rays were normal, but an magnetic resonance imaging (MRI) showed a posterolateral thoracic disk herniation at the level of T10-T11 (Slides 1 and 2). Which of the following is the best suggested treatment?
1) Bed rest
3) Laminectomy and decompression
2) Thoraco-lumbar orthosis
5) Thoracotomy, vertebractomy, strut graft and internal fixation
4) Diskectomy through thoracotomy or costotransverectomy
Conservative treatment should be considered for patients without major neurologic deficits. Posterior laminectomy and decompression provides inadequate exposure of the herniated Disk. Vertebractomy, strut bone graft and instrumentation are not necessary. Thoracotomy and costotransversectomy are commonly used for disk herniations at the levels of T4-T12.Correct Answer: Diskectomy through thoracotomy or costotransverectomy
2440. (675) Q6-931:
The patient's clinical diagnosis is degenerative spondylolithesis. In what patient population is this condition most commonly symptomatic?
1) Pre-teen males
3) Males over 70-years-old
2) Females 40- to 70-years-old
4) Females 20- to 30-years-old
Degenerative spondylolithesis is most frequently symptomatic in the 40- to 70-year-old age range and is six times more common in females than in males. This population appears to have enough disc degeneration and motion to become symptomatic, whereas the older population tend to have acquired enough ankylosis at the level to prevent instability symptoms.Correct Answer: Females 40- to 70-years-old
2441. (676) Q6-932:
The patient was diagnosed with spinal stenosis of the lumbosacral spine. In addition to educating the patient about his condition, the most appropriate initial treatment is:
1) Walking program
3) Lumbar traction
2) Nonsteroidal anti-inflammatory drugs
4) Spinal decompression and fusion
Initial treatment begins with patient education, a physical therapy regime (gentle conditioning exercises), judicious activity change, and sometimes spinal support with a corset or light-weight brace. Anti-inflammatory nonsteroidal drugs provide some relief of symptoms for many patients.Correct Answer: Nonsteroidal anti-inflammatory drugs
The biggest contribution to lumbar lordosis:
1) Occurs mostly within the disk spaces
3) Is normally 70°
2) Occurs mostly within the vertebral bodies
5) Can be visualized best on an anteroposterior plain radiograph
4) Decreases with increasing age
Most of the lumbar lordosis occurs within the disk spaces and not within the vertebral bodies. Normal lumbar lordosis is between 30°to 50°, increases with age, and is best visualized on a lateral plain radiograph.Correct Answer: Occurs mostly within the disk spaces
2443. (957) Q6-1258:
The superior aspect of the iliac crest often bisects this midline spinal structure:
1) L2/L3 disk space
3) L3/L4 disk space
2) L3 vertebral body
5) L5/S1 disk space
4) L4/L5 disk space
The L4/L5 intervertebral disk space is located by placing your fingers at the top of a patient's iliac crests, while allowing your thumbs to meet at the midline of the spine between the palpable L4 and L5 spinous processes.Correct Answer: L4/L5 disk space
2444. (958) Q6-1259:
Certain physical examination maneuvers attempt to elicit tension signs. When used in the supine position, these maneuvers are designed to apply stretch or tension on the sciatic nerve and any inflamed nerve root against a herniated lumbar disk. Which of the following physical examination tests is not a tension sign maneuver:
1) Lasegue sign
3) Bowstring sign
2) McMurray sign
5) Contralateral straight-leg raising test
4) The sitting room test
McMurray sign is used to detect a torn meniscus in the knee and will have minimal effect on the sciatic nerve.
Lasegue sign is the classic straight-leg raising test.
The bowstring sign is a variation of the straight-leg raising test performed with the knee in a flexed position. Digital pressure is then applied over the popliteal space in an attempt to reproduce the tension sign.
The sitting room test is performed with the patient in a sitting position. The hip remains flexed at 90° while the examiner extends the ipsilateral knee.
The contralateral straight-leg raising test is performed in the same manner as the straight-leg raising test except the contralateral, or nonpainful, leg is raised.
Correct Answer: McMurray sign
When palpating the sacral triangle in the posterior aspect of a patient's lower back, if gaps are present between the spinous processes or no lumbar or sacral bony prominences are detected, this is suggestive of:
1) Spina bifida
3) Scoliosis
2) Gibbus deformity
4) Becs de perroquet
When palpating the lumbosacral area (sacral triangle), if palpable gaps are present between the spinous processes or there is an absence of lumbar and/or sacral bony prominences, this is suggestive of spina bifida.
A Gibbus deformity is characterized by a sharp kyphosis and is often found in the thoracic spine. Scoliosis is identified by a lateral curvature of the spine
A palpable "step-off" of one spinous process relative to the next would be suggestive of a spondylolisthesis.
Becs de perroquet is a radiographic feature associated with tuberculosis of the lumbar spine in which bony bridges form across the sides of two adjacent vertebrae.
Correct Answer: Spina bifida
2446. (960) Q6-1261:
Which of the following is not a routinely used imaging technique for the evaluation of lumbar disk disease:
1) Myelography
3) Magnetic resonance imaging (MRI)
2) Computer tomography
5) Bone scan
4) Positron emission tomography (PET)
Positron emission tomography (PET) is a technique that measures brain activity through positron emission from radiolabled glucose.
Myelography is an invasive procedure with radio-opaque dye placed into subarachnoid space. It aids in the detection of neural compressive lesions.
Computer tomography alone offers better visualization of bony lesions, foraminal spinal stenosis, and lateral disk herniations when compared to plain myelography. Computer tomography is often combined with myelography.
Magnetic resonance imaging (MRI) has an advantage over CAT because it detects soft tissue pathologies, including improved spinal cord imaging in the detection of intraspinal tumors. MRI also examines the entire spine.
Bone scanning is a nonspecific but sensitive test. It is useful in detecting neoplastic, infectious, traumatic, and/or arthritic problems in the spine.
Correct Answer: Positron emission tomography (PET)
2447. (961) Q6-1262:
A 28-year-old woman complains of pain and numbness in her lower legs bilaterally for approximately 2 months following strenuous moving of furniture. She now states that she has not voided in the past 48 hours and that her abdomen area is markedly distended. Which is the most likely causative lesion of the patient's symptoms:
1) Cauda equina syndrome
3) Posterolateral disk herniation
2) Far-lateral disk herniation
5) Muscle spasms of the lower back
4) Spinal stenosis
This patient's symptoms are most consistent with cauda equina syndrome. This surgical emergency can present with bowel or bladder dysfunction, and bilateral lower extremity symptoms are also often present.Correct Answer: Cauda equina syndrome
A 28-year-old woman complains of pain and numbness in her lower legs bilaterally for approximately 2 months following strenuous moving of furniture. She now states that she has not voided in the past 48 hours and that her abdomen area is markedly distended. What diagnostic test must be performed in order to support the suspected diagnosis:
1) Voiding cystourethrogram
3) Posteroanterior and lateral plain radiographs of the lumbar spine
2) Magnetic resonance imaging of the lumbosacral spine
5) Electromyography of the lower extremities
4) Spinal puncture to rule out infection
Based on history and physical examination, the suspected diagnosis is cauda equina syndrome. This potential surgical emergency requires immediate spinal imaging. A magnetic resonance imaging of the lumbosacral spine is the most appropriate test.Correct Answer: Magnetic resonance imaging of the lumbosacral spine
2449. (963) Q6-1264:
On physical examination, the umbilicus is a superficial landmark for the bifurcation of the aorta into the common iliac arteries and overlies this disk space:
1) L1/L2
3) L3/L4
2) L2/L3
5) L5/S1
4) L4/L5
The umbilicus is a superficial landmark that often lies over the anterior L3/L4 disk space, which is the location of the aortic bifurcation into the common iliac arteries. Below this arterial division, in lean individuals, one can palpate the anterior bodies of L4, L5, and S1.Correct Answer: L3/L4
2450. (964) Q6-1266:
Slide 1
A 35-year-old woman has been complaining of severe unrelenting mid to low back pain for the past 5 months. Conservative management, consisting of bed rest and nonsteroidal anti-inflammatory drugs (NSAIDs), has not decreased the intensity of her symptoms. She immigrated to the United States from Vietnam 6 months ago. Based on the sagittal magnetic resonance image below, the next step in her management is:
1) Antibiotics with gram-positive coverage
3) Biopsy of the lesion to obtain a specimen for pathology
2) Surgical decompression and reconstruction
5) Physical therapy for low back strengthening
4) Continued conservative management and observation
It is prudent to determine the underlying etiology of this lesion. Tuberculous spondylitis is increasing in frequency and must be suspected in people who emigrate from countries where tuberculosis is endemic. A biopsy of the region must be obtained in order to make the diagnosis of tuberculosis accurately or any other infectious and noninfectious causative agent in order to determine proper management.Correct Answer: Biopsy of the lesion to obtain a specimen for pathology
What is the most common sequence of steps performed during a midline open disectomy in the treatment and excision of a herniated posterolateral lumbar disk:
1) The paraspinal musculature is stripped from the lamina of the vertebra, the ligamentum flavum is excised, portions of the superior and inferior lamina are removed, the nerve root and dural sac is retracted, and the disk herniation is excised.
3) The ligamentum flavum is excised, the paraspinal musculature is stripped from the lamina of the vertebra, the nerve root and dural sac is retracted, portions of the superior and inferior lamina are removed, and the disk herniation is excised.
2) The ligamentum flavum is excised, the paraspinal musculature is stripped from the lamina of the vertebra, portions of the superior and inferior lamina are removed, the nerve root and dural sac is retracted, and the disk herniation is excised.
5) Ligamentum flavum is excised, the disk herniation is excised the paraspinal musculature is stripped from the lamina of the vertebra, the nerve root and dural sac is retracted, and portions of the superior and inferior lamina are removed.
4) The paraspinal musculature is stripped from the lamina of the vertebra, the disk herniation is excised, the ligamentum flavum is excised, the nerve root and dural sac is retracted, and portions of the superior and inferior lamina are removed.
The traditional surgery for the excision of a herniated posterolateral lumbar disk is by means of a midline incision. This procedure is then performed in a stepwise fashion: The paraspinal musculature is stripped from the lamina of the vertebra; the ligamentum flavum is then excised; portions of the superior and inferior lamina are removed; and the nerve root and dural sac are identified and carefully retracted. This is followed by excision of the herniated disk material and wound closure.Correct Answer: The paraspinal musculature is stripped from the lamina of the vertebra, the ligamentum flavum is excised, portions of the superior and inferior lamina are removed, the nerve root and dural sac is retracted, and the disk herniation is excised.
2452. (966) Q6-1269:
On physical examination, a patient with a weak extensor hallucis longus muscle might be expected to have a far-lateral disk herniation at what level in the spine:
1) L2/L3
3) L4/L5
2) L3/L4
5) S1/S2
4) L5/S1
A far-lateral disk herniation in the lumbar spine often compromises the more proximal, or exiting, nerve root and not the more distal, or traversing, nerve root most typically affected in a posterolateral disk herniation. Therefore, a far-lateral disk herniation at the L5 - S1 level could irritate the L5 nerve root, which would affect the extensor hallucis longus muscle.Correct Answer: L5/S1
2453. (968) Q6-1271:
One traditional surgery performed for the treatment of a symptomatic posterolateral lumbar disk herniation is a partial laminectomy and lumbar disk excision by means of a midline incision. What is the long-term (>2 years) success rate for relief of both leg and low back pain, respectively:
1) 80% and 93%
3) 93% for both
2) 93% and 80%
5) 55% and 65%
4) 80% for both
The success rate following a partial laminectomy and lumbar disk excision for a posterolateral herniated disk for relief of both leg and low back pain is predictable at 93% and 80%, respectively.Correct Answer: 93% and 80%
A 38-year-old man injured his neck and spinal cord 6 months ago as a result of a motorcycle accident. He is now a C 6 quadriplegic. He wants to know how his sexual function will be affected by his condition. The patient should be informed that:
1) He should be able to obtain normal erections and ejaculation function.
3) With external stimulation, a reflex erection is possible but normal ejaculation is not.
2) No erections or ejaculation function will exist.
5) He has a low probability of being sterile because precise control of thermal regulation of the testes is usually retained.
4) Normal erections and ejaculations with low sperm counts are possible.
With this type of spinal cord injury, it is possible to have an erection with external stimulation; however, ejaculation would have to be facilitated by electrical stimulation or vibratory means.Correct Answer: With external stimulation, a reflex erection is possible but normal ejaculation is not.
2455. (970) Q6-1273:
When testing the range of motion in the lumbar spine, which maneuver involves stretching of the interspinous and supraspinous ligaments, ligamentum flavum, and posterior longitudinal ligament while relaxing the anterior longitudinal ligament:
1) Flexion
3) Lateral bending
2) Extension
5) None of the above
4) Lateral bending
Flexion of the lumbar spine creates stretching or tension of the posterior spinal structures including the interspinous and supraspinous ligaments, ligamentum flavum, and posterior longitudinal ligament. Flexion also allows relaxation of the anterior longitudinal ligament. This is often elicited by having a patient bend forward in attempt to touch the floor with the fingertips. If the patient cannot touch the floor, the distance from the floor to fingertips is measured in fingerbreadths or inches.Correct Answer: Flexion
2456. (971) Q6-1275:
Slide 1
A 22-year-old woman sustained an injury to her low back 1 year ago while playing rugby. She now complains of excruciating low back pain with numbness and tingling into her left buttock. This pain is affecting her daily living activities. The patient underwent 6 months of conservative management consisting of restriction of activities, physical therapy, and anti-inflammatory medication with little relief. Based on the image below, the next appropriate step in the management of this patient is:
1) Continued conservative management
3) Laminectomy at the L2/L3 level
2) Posterolateral fusion at the L5/S1 level with bone graft
5) Diskectomy at the L3/L4 level
4) Laminectomy at the L3/L4 level
The patient has an L5/S1 spondylysis with a grade 1 spondylolisthesis. This patient has undergone a sufficient attempt at conservative management with continued unrelenting low back pain. The next most appropriate step in the management of this condition is a posterolateral fusion at the L5/S1 level with autologous bone graft.Correct Answer: Posterolateral fusion at the L5/S1 level with bone graft
When performing a neurological examination, if a surgeon has a patient resist thigh adduction against resistance, the surgeon is testing which nerve(s):
1) Segmental nerves from T1-L1
3) Obturator nerve
2) Femoral nerve
5) Segmental nerves from L5-S1
4) Sciatic nerve
The obturator nerve innervates most of the hip adductor group, which consists of neurologic levels L2, L3, and L4.Correct Answer: Obturator nerve
2458. (973) Q6-1277:
A 72-year-old man comes to the office complaining of low back and thigh pain that has been progressively getting worse over the years. The pain now bothers him during weather changes. The patient tells you that his pain is worse with standing and walking for long periods. However, leaning forward and sitting alleviates his discomfort. The patient denies any bladder or bowel complaints. This patient most likely has:
1) A herniated lumbar disk
3) Lumbar spinal stenosis
2) Cauda equina syndrome
5) Vascular insufficiency
4) Spinal cord tumor
This patient is presenting with the hallmark symptoms of lumbar spinal stenosis, which consists of increased back and thigh pain with walking or standing with relief of the symptoms by leaning forward or sitting down. This patient denies any bladder or bowel dysfunction that makes cauda equina syndrome unlikely. Although urinary dysfunction is uncommon in spinal stenosis, it can occur in approximately 3% to 4% of cases.Correct Answer: Lumbar spinal stenosis
2459. (974) Q6-1278:
A positive straight-leg raise sign:
1) Occurs when the obturator nerve is stretched
3) Is important in predicting a successful outcome if surgery is performed to excise the herniated fragment
2) Is always associated with a history of trauma or injury
5) Is not sensitive in detecting a lumbar disk herniation
4) Is a specific test to detect a lumbar disk herniation
A positive straight-leg raise sign occurs when the sciatic nerve is placed under tension or is stretched reproducing a radiculopathy. It is also suggestive of inflammation around the nerve root, hence it is a good predictor of the successful relief of symptoms from surgical decompression. Additionally, a positive straight-leg raise sign is a relatively sensitive test for detecting a lumbar disk herniation, but it is not specific. Because the sign is not specific, it does not necessarily correspond to trauma or injury.Correct Answer: Is important in predicting a successful outcome if surgery is performed to excise the herniated fragment
2460. (975) Q6-1279:
What physical examination maneuvers listed below check the status of the L4 neurologic level:
1) Sensation on the lateral side of the ankle, the patellar tendon reflex, and plantar eversion
3) Sensation on the lateral side of the ankle, the Achilles tendon reflex, and plantar inversion
2) Sensation on the lateral side of the ankle, the patellar tendon reflex, and plantar inversion
5) Sensation on the medial side of the ankle, the patellar tendon reflex, and plantar inversion
4) Sensation on the medial side of the ankle, the Achilles tendon reflex, and plantar inversion
Sensation on the medial side of the ankle, the patellar tendon reflex, and plantar inversion are associated with the L4 neurologic level. Sensation on the lateral side of the ankle and the Achilles tendon reflex are associated with the S1 neurologic level.Correct Answer: Sensation on the medial side of the ankle, the patellar tendon reflex, and plantar inversion
2461. (976) Q6-1280:
In describing idiopathic scoliosis, there are several terms given to curve patterns on radiographs to describe the specific type of spinal deformity present. Which term best describes an area of the spine with a lateral curve that lacks normal flexibility noted radiographically by its failure to demonstrate segmental mobility or correction on supine side-bending radiographs:
1) Primary curve
3) Major curve
2) Compensatory curve
5) Structural curve
4) Minor curve
The above question is describing a structural (nonflexible) curve.
A primary curve is the earliest of several curves that may eventually develop. The term major curve is used to designate the largest structural curve.
The minor curve refers to the smallest structural curve and is usually more flexible than the major curve.
The compensatory curve is located above or below a major curve to attempt to maintain normal body alignment.
Correct Answer: Structural curve
2462. (977) Q6-1281:
When performing a physical examination, if running the pointed edge of a reflex hammer along the crest of a patient's tibia causes extension of the great toe while the remaining toes splay or plantarflex, this finding would indicate:
1) A Babinski reflex is present
3) A Positive Oppenheim test
2) A Babinski reflex is absent
5) None of the above
4) A Negative Oppenheim test
An Oppenheim test is considered positive when running a pointed object along a patient's tibial crest elicits splaying or plantarflexion of the smaller toes with great toe extension. An Oppenheim test is considered negative when this reaction is not present.
A Babinski reflex is present when running a pointed object across the plantar surface of a patient's foot elicits splaying or plantarflexion of the toes with extension of the great toe. A Babinski reflex is considered absent when the toes either do not move or all five toes flex and/or bunch up in response to the stimulus.
Correct Answer: A Positive Oppenheim test
2463. (978) Q6-1282:
When trying to distinguish hamstring tightness/discomfort from sciatic pain/radiculopathy, surgeons can perform a straight-leg raise test on the affected side until the point at which the patient develops discomfort. This is followed by slightly lowering the affected extremity. While holding the patient's leg in this position, what maneuver could be performed in order to help reproduce true sciatic pain:
1) Rotate the patient's foot medially 10°
3) Plantarflex the patient's foot
2) Rotate the patient's foot laterally 10°
5) Perform a Babinski test
4) Dorsiflex the patient's foot
Dorsiflexion of the foot, known as Braggard test, adds additional tension or stretch to the sciatic nerve and may help reproduce the sciatic pain/radiculopathy.Correct Answer: Dorsiflex the patient's foot
Beevor sign is a physical examination maneuver that tests the integrity of the rectus abdominus muscles that are segmentally innervated by the anterior primary divisions of the T5-T12/L1 nerve roots. When performing this test, the patient is asked to perform a partial sit-up. A positive Beevor sign is indicated by:
1) Pain in the abdominal musculature
3) The absence of any deviation of the umbilicus
2) Simultaneous raising both lower extremities
5) Pain radiating down both lower extremities
4) The presence of umbilical deviation
When performing a sit-up, umbilical deviation due to abnormal contraction of the rectus musculature indicates either a partial or complete loss of segmental innervation to a portion of the rectus abdominus and/or paraspinal musculature. It is frequently seen in patients with certain neurological disorders such as meningomyelocele and poliomyelitis.Correct Answer: The presence of umbilical deviation
2465. (980) Q6-1284:
Which anatomic structure(s) may be the cause of referred pain to the lumbar spine region:
1) The hip
3) The pelvis
2) The rectum
5) The hip, the pelvis, and the rectum
4) Both the hip and the pelvis
The hip, rectum, and pelvis may refer pain to the lumbar spine region. In order to perform a complete physical examination, a rectal exam is recommended on all patients with pain in the lumbar spine region.Correct Answer: The hip, the pelvis, and the rectum
2466. (981) Q6-1285:
The following nonoperative treatments have not been proven effective in the early acute stage (2 weeks to 3 months) of low back pain:
1) Nonsteroidal anti-inflammatory drugs
3) Anesthetic/corticosteriod injections into the epidural space
2) Bed rest
5) Intraspinal anesthetic/corticosteriod injections
4) Intrathecal anesthetic/corticosteriod injections
Nonsteroidal anti-inflammatory drugs have been shown effective and are frequently used during the acute phase of low back pain. Their main effect is to alleviate soft tissue inflammation that is often present in the early phase.
Patient questionnaires have identified bed rest as among the most frequently prescribed treatments for lower back pain. It has been shown that bed rest results in reduced intradiskal pressure that occurs in the supine position.
Anesthetic/corticosteriod injections are widely advocated for the treatment of low back pain and can be administered along nerve roots, into the sacroiliac joints, intervertebral disks, paraspinal soft tissues, and the epidural space or intrathecally for many conditions. However, there is no evidence that intraspinal steroids have an effective role in the acute management of low back pain.
Correct Answer: Intraspinal anesthetic/corticosteriod injections
A 32-year-old woman is diagnosed on magnetic resonance imaging with a far-lateral disk herniation at the L3/L4 level causing radiating right lower extremity discomfort across the anterior aspect of her knee with no motor or reflex abnormalities. Which of the following nerve roots is most likely affected:
1) L1
3) L3
2) L2
5) L5
4) L4
A far-lateral or foraminal disk herniation often affects the exiting or more proximal nerve root. The traversing or more distal nerve root is typically affected from a posterolateral disk herniation. In this case, the patient has a far-lateral disk herniation at the L3/L4 level resulting in L3 nerve root symptomatology.Correct Answer: L3
2468. (983) Q6-1287:
An 18-year-old man comes to the office with a grade IV isthmic spondylolisthesis and severe left lower extremity discomfort. Which imaging study would best help identify the site of potential nerve root compression:
1) Computed tomography scan
3) Plain myelography
2) Magnetic resonance imaging (MRI) scan
5) Positron emission tomography (PET)
4) Bone scan
An MRI scan would be the imaging study of choice to best identify the site of nerve root compression because it provides parasagittal views that can help determine the degree of narrowing of the neural foramina.
Plain myelography may not accurately identify lateral recess compression.
A CT scan would produce images perpendicular to the plane of nerve root compression. Bone scan and PET scan would be of little or no help in identifying nerve root compression.
Correct Answer: Magnetic resonance imaging (MRI) scan
2469. (984) Q6-1288:
Some of the more common risk factors that could predispose a person to developing low back pain are listed below. Which risk factor has not been implicated:
1) Occupations requiring the use of jackhammers
3) Cigarette or other tobacco consumption
2) Being above normal bodyweight
5) Occupations requiring the frequent operation of motor vehicles
-
Female gender
Being female has not been implicated as a risk factor in the development of low back pain. Occupations that require heavy lifting, the use of jackhammers, and operating motor vehicles, as well as the usage of tobacco products and being overweight have all been associated with a higher incidence of developing low back pain.Correct Answer: Female gender
When considering surgical intervention in the management of low back pain, it is crucial to try and identify the possible offending agent or pain generator. Based on awake anatomical stimulation studies, what percentage of patients should report significant discomfort when a nerve root is either compressed or stretched in an attempt to elicit pain:
1) 99%
3) 40%
2) 60%
-
1%
4) 20%
Studies have reported on diskectomies in awake patients performed under local anesthesia. Anatomic spinal structures were stimulated prior to additional local anesthesia placed into these deeper areas and patients were asked to report any pain.
Compression or stretching of nerve roots caused significant pain 100% of the time. Stimulation of the posterior dura caused significant pain only 1% of the time.Correct Answer: 99%
2471. (986) Q6-1291:
Which disorder does not represent a rheumatologic/inflammatory condition associated with causing low back pain:
-
Rheumatoid arthritis
3) Psoriatic arthritis
2) Reiter syndrome
5) Osteoarthritis
-
Ankylosing spondylitis
Although osteoarthritis is the most common arthritic disorder associated with low back pain, it is not a rheumatologic condition. Rheumatoid arthritis, Reiter syndrome, psoriatic arthritis, and ankylosing spondylitis are all rheumatologic or inflammatory conditions associated with the development of low back pain.Correct Answer: Osteoarthritis
2472. (987) Q6-1292:
A 57-year-old man with known lung cancer and metastatic disease complains of increasing low back pain. How often is the lumbar spine involved when a patient has known spinal metastasis:
1) 5%
3) 50%
2) 15%
-
99%
4) 85%
In 85 % of patients with metastatic disease to the spine, the lumbar region vertebral body is involved. If operable, the vertebral body lesion can be managed via an anterior approach with a corpectomy, as opposed to a posterior approach with laminectomy and removal of all involved posterior elements (if they are involved).Correct Answer: 85%
2473. (988) Q6-1293:
A 32-year-old man develops left lower extremity discomfort following a basketball game. A magnetic resonance image of the lumbar spine reveals a posterolateral disk herniation at the level of L4/L5. All of the following conditions could be associated with this except:
-
Decreased strength in the hip abductors
3) Numbness in the lateral aspect of the leg
2) Decreased strength in the extensor hallucis longus muscle
5) Pain in the dorsum of the foot
4) Decreased strength in plantarflexion of the foot
Decreased strength in the hip abductors and in the extensor hallucis longus muscle along with numbness in the lateral aspect of the leg and pain in the dorsum of the foot can all be associated with a herniation at the L4/L5 level affecting the L5 nerve root. Decreased strength in plantarflexion of the foot is present when the S1 nerve root is involved, such as with a disk herniation at the L5/S1 level.Correct Answer: Decreased strength in plantarflexion of the foot
2474. (989) Q6-1294:
A 43-year-old man develops pain radiating down his left leg from a far-lateral disk herniation at the L5/S1 level. Which is the most likely nerve root contributing to his discomfort:
1) L2
3) L4
2) L3
5) S1
4) L5
Typically, for a posterolateral disk herniation, the traversing or more distal nerve root is often involved. With a far-lateral disk herniation (as in this case), the exiting nerve root is usually involved. In this case, it would be the L5 nerve root.Correct Answer: L5
2475. (990) Q6-1296:
Which orthopedic test, designed to apply tension to the spinal cord producing pain, involves having the patient lie supine while the examiner flexes the patient's head to his chest:
1) Kernig sign
3) Milgram test
2) Hoover test
5) Gaenslen test
4) Naffziger test
The Kernig test involves the patient lying supine while the examiner forcibly flexes the head to the chest applying tension to the spinal cord. The test is positive if pain is elicited and indicates meningeal irritation in conditions such as meningitis.
The Hoover test can help identify a patient who is malingering. This test involves having the patient lie supine with the examiner's hands placed under both of the patient's heels. The patient is asked to lift the affected leg. If a true effort is made, the examiner should feel downward pressure in the patient's opposite foot as he attempts to lift the affected leg. If no downward pressure is felt, the patient purposely may not be trying and might be malingering.
The Milgram test may be used in conditions with suspected intrathecal pathology. While lying supine, the patient is asked to perform bilateral straight leg lifts. If the patient can sustain his feet 2 inches off the ground for more 30 seconds, intrathecal pathology is less likely.
The Naffziger test is designed to increase intrathecal pressure and thus pain by compressing the jugular veins. The Gaenslen test is used to determine sacroiliac joint pathology.
Correct Answer: Kernig sign
2476. (991) Q6-1297:
A 52-year-old man sustained an L1 burst fracture after falling from a ladder 3 weeks ago. He was found neurologically intact after initial examination. He has been treated with conservative management since the accident, but he now complains of an inability to void along with numbness and tingling in both of his legs. Computed tomography scan shows worsening conus compression. On physical examination, you discover that he has weak anal sphincter tone. Your next step in the treatment of this patient should be:
1) Anterior decompression and fusion with grafting with or without instrumentation
3) Laminectomy
2) High-dose steroids
5) Continue conservative management with follow-up in 1 month
4) Posterior fusion with instrumentation
This patient has deteriorating neurological findings involving the cauda equina; therefore, surgical decompression is indicated. An anterior approach will directly decompress the neural structures.
Performing a laminectomy alone is actually contraindicated as it may increase potential spinal instability.
A posterior approach to achieve a fusion with instrumentation may not fully decompress the neural structures, although this is somewhat controversial.
Correct Answer: Anterior decompression and fusion with grafting with or without instrumentation
A 27-year-old man comes in for examination. He complains of worsening pain in his lower back. He states that the pain started 4 days ago after lifting a heavy box. The patient's neurological exam is completely nonfocal. The initial management of this patient should include:
1) Computed tomography scan of the lumbar spine with contrast
3) Magnetic resonance image of the lumber spine
2) Epidural steroid injection
5) Computed tomography scan of the lumbar spine without contrast
4) Bed rest of short duration along with nonsteroidal anti-inflammatory medications
This patient's neurological exam is normal and his injury was recent; this is most likely a soft tissue injury to his low back. Due to the patient's age and the fact that this is most likely not a serious injury, an initial conservative approach would be most appropriate. Epidural steroids or a selective nerve root block would be indicated if this patient developed worsening low back complaints or radicular pain in association with his low back pain.Correct Answer: Bed rest of short duration along with nonsteroidal anti-inflammatory medications
2478. (993) Q6-1299:
When trying to distinguish sciatic radicular pain from pain following a hamstring strain, it is important to know that pain from a hamstring strain usually only involves the posterior aspect of thigh. Sciatic nerve pain may also be associated with:
1) Pain radiating down the leg and into the foot
3) Pain in the opposite leg
2) Low back pain
5) Pain radiating down the leg and into the foot, pain in the opposite leg, and low back pain
4) Pain radiating down the leg and into the foot, and pain in the opposite leg
Sciatic pain can involve all of the above complaints including radicular pain extending down the leg, low back pain, and pain into the opposite leg.Correct Answer: Pain radiating down the leg and into the foot, pain in the opposite leg, and low back pain
2479. (994) Q6-1300:
A 42-year-old man sustained a twisting injury to his low back 5 months ago. Since the injury, he has persistent low back pain that radiates into his right thigh and down to his posterior calf. The patient underwent a magnetic resonance imaging of his lumbar spine revealing a small posterolateral lumbar disk herniation at the L4L5 level. Over the past month, the patient states that his leg pain has been getting progressively better and has almost disappeared over the past week with the use of nonsteroidal anti-inflammatory medications and occasional bed rest. The next step in the management of this patient should be:
1) A lumbar computed tomography scan
3) Surgical excision of the herniated disk
2) Repeat magnetic resonance image
5) Epidural steroid injection
4) Continued conservative management
The patient has shown continued improvement of his symptoms including the leg pain with conservative treatment. Epidural steroids would be indicated if this patient had continued or worsening leg pain and/or low back pain.Correct Answer: Continued conservative management
Slide 1
This radiograph shows a grade I spondylolisthesis of L5 on S1. This is due to a defect in what anatomical area:
1) Superior articular process
3) Pars interarticularis
2) Inferior articular process
5) Lamina
4) Pedicle
The anatomical region involved in a spondylolisthesis is the pars interarticularis that is located between the superior and inferior articular processes and is a high stress area of relatively thinner bone.Correct Answer: Pars interarticularis
2481. (996) Q6-1302:
Slide 1
A 16-year-old football lineman develops unrelenting low back pain for the past 3 months. Based on the magnetic resonance image shown, the next step in the management of this patient is:
1) Fusion in situ
3) Excision of a herniated disk
2) Epidural injection therapy
5) Observation
4) Restriction of the exacerbating activity
This patient has a grade I-II spondylolisthesis of L5 on S1. The initial management should include restriction of physical activity. Once the symptoms abate, the athlete can return to the sport. If symptoms return, other interventions are indicated including possible brace wear.Correct Answer: Restriction of the exacerbating activity
Slide 1
A 50-year-old woman with 3 months of low back pain recently discovers a hard, painless lump in her breast. Due to the back discomfort, she undergoes plain radiography and subsequently a computed tomography scan (below). The most likely diagnosis is:
1) Osteomyelitis
3) Fracture
2) Osteoid osteoma
5) Metastatic disease
4) Herniated nucleus pulposis
The computed tomography scan reveals a destructive lesion involving the vertebral body extending into the pedicle in a patient with a suspected breast malignancy. This is a metastatic lesion until proven otherwise. This patient needs a thorough evaluation of her breast lesion, as well as her spine lesion, including biopsies. An osteoid osteoma is seen in a younger population and is seen on a computed tomography scan as a sclerotic round lesion.Correct Answer: Metastatic disease
2483. (998) Q6-1305:
Slide 1
A 45-year-old construction worker with long standing low back pain now notices bilateral thigh and lower extremity discomfort for the past 6 months. He has undergone conservative treatment with little success including injection therapy. He cannot perform his work duties. Based on the lateral radiograph shown below, the next step in the management of the patient should consist of:
1) Laminectomy
3) Posterolateral fusion in situ without instrumentation
2) Continued conservative treatment
5) Decompression and posterolateral fusion in situ with instrumentation.
4) Diskectomy
This patient has an isthmic L4L5 spondylolisthesis with accompanied neuroforaminal stenosis. He already failed conservative management and would benefit from operative intervention. This should consist of a decompressive procedure to alleviate his lower extremity symptomatology along with a fusion procedure of the L4-L5 level due to the listhesis. If he undergoes a laminectomy alone, he may develop increased low back pain in the future.Correct Answer: Decompression and posterolateral fusion in situ with instrumentation.
A 35-year-old woman presents with severe back pain. Radiographic evaluation reveals a thoracic curve of 70° and a loss of thoracic kyphosis. Surgery is recommended to correct the deformity. Which of the following tests must be ordered as part of the preoperative evaluation:
1) Electrocardiogram (ECG)
3) Electromyelogram (EMG)
2) Pulmonary function tests
5) Somatosensory evoked potentials (SSEP)
4) Chest radiograph
Thoracic curves greater than 65° may affect pulmonary function, especially when they are combined with thoracic lordosis. This patient displays a thoracic curve of 70° and a loss of the normal thoracic kyphosis; therefore, pulmonary function tests are part of the routine evaluation.Correct Answer: Pulmonary function tests
2485. (1203) Q6-1569:
A calcified thoracic disk in the spinal canal is pathognomonic for:
1) Scheuermann disease
3) Thoracic disk herniation
2) Ankylosing spondylitis
5) Infection
4) Vertebral wedging
Plain radiographs of the spine are helpful in the diagnosis of disk herniation only if they demonstrate disk calcification. However, the calcified disk is not always the disk that is herniated, but it suggests the diagnosis. Detection of a calcified disk on radiograph in the canal is pathognomonic of herniation.Correct Answer: Thoracic disk herniation
2486. (1204) Q6-1570:
Surgical treatment of thoracic disk herniation by a laminectomy is contraindicated because this procedure is associated with which of the following:
1) Incomplete relief of symptoms
3) Destabilization of the spine
2) High incidence of neurologic damage
5) High incidence of post-laminectomy kyphosis
4) High incidence of recurrence
There is a high incidence of spinal cord injury associated with thoracic disks removed by laminectomy. The advent of alternative procedures, such as costotransversectomy and transthoracic decompression, has led to a decrease in spinal cord injury admissions. Also, patients who do not improve after laminectomy are less likely to be helped by later anterior decompression.Correct Answer: High incidence of neurologic damage
2487. (1205) Q6-1571:
The most common presenting symptom of a patient with a thoracic disk herniation is:
1) Anterior band-like chest pain
3) Epigastric pain
2) Intrascapular pain
5) Lower extremity weakness
4) Lower extremity pain
Brown et al reported on a series of 55 patients initially treated with conservative management. Anterior band-like chest pain occurred in 67% of his patients. Lower extremity complaints accounted for 20% and ranged from weakness (16%) to parasthesias (4%).Correct Answer: Anterior band-like chest pain
The natural history of an asymptomatic thoracic disk herniation is:
1) Rapid progression to a symptomatic thoracic disk herniation
3) To remain asymptomatic
2) A slow progression to a symptomatic thoracic disk herniation
5) To completely resorb and progress to a degenerative disk
4) To completely resorb and remain asymptomatic
The natural history of an asymptomatic thoracic disk herniation is to remain asymptomatic and exhibit little change in size. In a series of 48 asymptomatic thoracic disk herniations, Wood found that all disks remained asymptomatic at follow-up with little fluctuation in size of the disk.Correct Answer: To remain asymptomatic
2489. (1207) Q6-1573:
A 48-year-old man presents with acute onset of unilateral, anterior band-like chest pain after lifting heavy machinery at work. The history and physical examination and the magnetic resonance image confirm a T9-T10 thoracic disk herniation. The best initial treatment for this patient is:
1) Bed rest and traction for 6 weeks
3) Activity modification and physical therapy
2) Costotransversectomy to remove the T9-T10 disk herniation
5) Laminectomy and decompression of the disk
4) Transthoracic decompression of the disk
Brown et al retrospectively reviewed the natural history of symptomatic thoracic disk herniations and found 77% of patients did well with nonsurgical management. The patients returned to their previous level of activity following activity modification and physical therapy.Correct Answer: Activity modification and physical therapy
2490. (1208) Q6-1574:
The most common site of a thoracic disk herniation requiring surgery is from levels:
1) T1-T4
3) T8-T11
2) T4-T7
5) T12-L1
4) T11-T12
T8-T11 is the most common site of disk herniation that requires surgery. A review of 71 patients with 82 thoracic disk herniations undergoing surgery found that 66% of disks were between T8-T11. The most common disk level was T9-T10, which represented 26% of the herniations.Correct Answer: T8-T11
2491. (1209) Q6-1575:
The most common location for a thoracic disk herniation is:
1) Central
3) Centrolateral
2) Lateral
5) Mediolateral
4) Medial
The most common locations for a thoracic disk herniation are centrolateral (94%) and lateral (6%). Disks classified as centrolateral have the bulk of the disk herniation medial to the lateral margin of the thecal sac.Correct Answer: Centrolateral
A 38-year-old construction worker falls from a scaffolding and sustains a pure flexion-compression injury to T12. In this type of injury, which portion of the vertebral body fails first:
1) End plate
3) Posterior elements
2) Subcortical cancellous bone
5) Lamina
4) Middle column
Failure occurs first at the end plate. The intact intervertebral disk has limited compressibility. Therefore, when the compressive forces exceed the disk compressibility, the load is transmitted to the contiguous bone. The end plate will rupture first followed by the subcortical cancellous vertebral bone.Correct Answer: End plate
2493. (1211) Q6-1578:
An absolute indication for surgical management of thoracolumbar burst fractures is:
1) Canal compromise greater than 10%
3) Kyphotic deformity greater than 10%
2) Canal compromise greater than 30%
5) Progressive neurologic deficit
4) Kyphotic deformity greater than 30%
Patients with a neurologic deficit or a progressive neurologic deficit should undergo operative decompression. Controversy exists as to the amount of kyphosis and canal compression that is considered acceptable. Support can be found in the literature for both operative and nonoperative management of neurologically intact burst fractures. Each patient must be evaluated on a case by case basis and followed closely after injury.Correct Answer: Progressive neurologic deficit
2494. (1212) Q6-1579:
A 12-year-old girl presents with back pain of 3 monthsâ duration. She is a Risser stage 2. She displays a left thoracic curve of 27° on radiographs. The next study obtained in the work-up should be:
1) Lateral bending films
3) Head computerized tomography
2) Computerized tomography scan of the spine
5) Ultrasound of the kidneys
4) Magnetic resonance image of the thoracic spine
Left thoracic curves are unusual in idiopathic scoliosis. A magnetic resonance image of the thoracic spine is mandatory in the work-up to rule out diastematomyelia, tethered spinal cord, spinal tumor, or other type of congenital anomaly.Correct Answer: Magnetic resonance image of the thoracic spine
2495. (1213) Q6-1580:
The most common organism responsible for vertebral column infection is:
1) Pseudomonas aeruginosa
3) Staphylococcus aureus
2) Staphylococcus epidermidis
5) Mycobacterium tuberculosis
4) Escherechia coli
Staphylococcus aureus accounts for more than 50% of spinal infections and often results from hematogenous dissemination. Gram-negative organisms are more common following genitourinary procedures or urinary tract infections. Staphylococcus epidermidis can complicate spinal surgical wounds, and polymicrobial infection is more common in these circumstances.Correct Answer: Staphylococcus aureus
Symptoms of spinal infection may include all of the following except:
1) Activity-related back pain
3) Neurological deficit
2) Fever
5) Decreased spinal range of motion
4) Torticollis
Neck or back pain associated with spinal infection is relentless and constant. The pain is not usually associated with activity. There may be night pain as well. Other symptoms and signs are variable, requiring a high degree of suspicion. Fever occurs less than 50% of the time and neurological deficit less than 10% of the time. Paraspinal muscle spasms may result in decreased range of motion or torticollis.Correct Answer: Activity-related back pain
2497. (1215) Q6-1582:
Which test is most specific for diagnosing spinal column infection:
1) White blood count
3) Carbon-reactive protein
2) Erythrocyte sedimentation rate
5) Biopsy
4) Blood culture
Vertebral biopsy, either via open or computed tomography-guided means, is most specific even though false-negative rates for closed and open biopsies are 30% and 14%, respectively.
A patientâs white blood count may be normal even in acute spinal infection.
Although often elevated, erythrocyte sedimentation rate and carbon-reactive protein are nonspecific tests. Blood cultures are negative in more than 75% of patients.
Correct Answer: Biopsy
2498. (1216) Q6-1583:
Which of the following describes the magnetic resonance image (MRI) appearance of vertebral osteomyelitis:
1) Increased signal onT1 images, decreased on T2 images
3) Decreased signal onT1 images, increased on T2 images
2) Decreased signal onT1 images, decreased on T2 images
5) MRI is usually unable to detect vertebral osteomyelitis
4) Increased signal onT1 images, increased on T2 images
Magnetic resonance image (MRI) carries a 95% accuracy rate. Infected disk and vertebral bone appear on MRI with decreased signal onT1 images and increased signal on T2 images. Gadoliniun enhancement is useful in differentiating spinal infection or abscess from epidural scar in the postoperative setting.Correct Answer: Decreased signal onT1 images, increased on T2 images
2499. (1217) Q6-1584:
Appropriate treatment for spinal infection may include all the following except:
1) Antibiotics
3) Brace immobilization
2) Surgical decompression
5) Removal of spinal hardware in the chronic infection
4) Removal of spinal hardware in the acute postoperative setting
Spinal stability appears to improve healing of spinal infection. Chronic, persistent infections may require removal of hardware. Antibiotics and immobilization are the mainstays of treatment. Neurological deficit from epidural abscess or kyphotic collapse may require operative decompression.Correct Answer: Removal of spinal hardware in the acute postoperative setting
Which of the following is not a surgical indication in the treatment of spinal column infection:
1) Persistent back pain and elevated c-reactive protein despite 8 weeks of intravenous antibiotics and bracing
3) Progressive kyphotic collapse
2) Progressive neurological deficit and magnetic resonance image evidence of epidural abscess
5) Extension of infection into the disk space
4) Development of sepsis
Uncomplicated spinal osteomyelitis and diskitis are treated nonoperatively. Operative debridement, decompression, and stabilization may be useful in cases of abscess, sepsis, neurological deficit, and progressive deformity.Correct Answer: Extension of infection into the disk space
2501. (1219) Q6-1586:
Which of the following is more characteristic of tuberculoid rather than pyogenic spinal infection:
1) Bony destruction on plain radiography
3) Prolonged onset of mild back pain despite extensive destruction seen on radiograph
2) Elevated erythrocyte sedimentation rate
5) Predilection for the cervical spine
4) High fevers, weight loss, and night pain
Spinal tuberculosis typically follows an indolent course early on despite radioqraphic findings out of proportion to the exam. Pyogenic and tuberculoid spinal infections involve the thoracic spine more commonly than the cervical spine. Both spinal infections may result in bony destruction, elevated erythrocyte sedimentation rates, and may or may not present with constitutional symptoms.Correct Answer: Prolonged onset of mild back pain despite extensive destruction seen on radiograph
2502. (1220) Q6-1587:
Which of the following is a risk factor for neurological deficit associated with tuberculoid spinal infection:
1) Age
3) Erythrocyte sedimentation rate higher than 90
2) Pulmonary involvement
5) History of hypertension
4) History of smoking
Tuberculosis in the cervical spine of children younger than 10 years of age carries a significantly lower risk of paralysis than in older patients (17% vs 81%).Correct Answer: Age
2503. (1221) Q6-1588:
All of the following organisms may cause granulomatous opportunistic spinal infection in immunocompromised patients except:
1) Mycobacteria
3) Actinomyces
2) Nocardia
5) Brucella
4) Staphylococcus
Staphylococcal infection is typically pyogenic, not granulomatous.Correct Answer: Staphylococcus
Antibiotic treatment for spinal tuberculosis includes all of the following except:
1) Isoniazid
3) Pyrazinamide
2) Ethambutol
5) Cefotaxime
-
Rifampin
A four-drug regimen against spinal tuberculosis is recommended because of the high prevalence of organism resistance. Cefotaxime is a cephalosporin not active against mycobacterial infection.Correct Answer: Cefotaxime
2505. (1223) Q6-1590:
What percentage of spinal infections have concurrent positive blood cultures:
1) 5%
3) 55%
2) 25%
-
95%
4) 75%
Even though the majority of spinal infections are considered hematogenous in origin, only 25% of infections occur with positive blood cultures.Correct Answer: 25%
2506. (1224) Q6-1591:
The treatment of choice for spinal epidural abscess is:
-
Four weeks of antibiotics
3) Surgical drainage plus a prolonged course of antibiotics
2) Parenteral antibiotics until the erythrocyte sedimentation rate falls to half of its pretreatment value
5) Bracing and analgesia
4) Spinal fusion
It is generally believed that pockets of pus, whether they are epidural, paravertebral, or psoas abscesses, must be drained in addition to antimicrobial therapy.Correct Answer: Surgical drainage plus a prolonged course of antibiotics
2507. (1225) Q6-1592:
Which of the following antibiotics would not be useful in staphylococcal vertebral osteomyelitis:
1) Cefuroxime
3) Cefazolin
2) Nafcillin
5) Tobramycin
-
Ciprofloxicin
Aminoglycosides, such as tobramycin, are active against gram-negative organisms. First- and second-generation cephalosporins are alternatives to semisynthetic penicillins that may be useful if the organism is not resistant. Ciprofloxicin has also been considered a possible alternative to penicillins against gram-positive vertebral osteomyelitis.Correct Answer: Tobramycin
2508. (1226) Q6-1593:
How common are spinal infections following penetrating injury to the spine:
1) 2%
3) 60%
2) 20%
-
95%
4) 80%
One study found that 5 of 239 patients with gunshot or stab wounds developed meningitis, paravertebral abscess, vertebral osteomyelitis, or epidural abscess.Correct Answer: 2%
2509. (1227) Q6-1595:
Risk factors implicated in postoperative wound infection following lumbar spine surgery include all of the following except:
-
Use of instrumentation
3) History of smoking
2) Presence of spina bifida occulta
5) Obesity
4) Longer operative duration
Instrumented cases, preoperative history of smoking or obesity, and longer operating room duration have all been identified as possible risk factors for surgical site infection. Additionally, patient age may be a risk factor or may be associated with a risk factor like medical comorbidity or nutritional depletion.Correct Answer: Presence of spina bifida occulta
2510. (1228) Q6-1596:
In the face of vertebral infection and progressive deformity, surgical reconstruction should:
1) Never use instrumentation
3) Always be approached posteriorly
2) Never use allograft bone
5) Always follow extensive antibiotic treatment until the infection is eradicated
4) Always involve an aggressive debridement
Surgical reconstruction in the face of spinal infection may be indicated should progressive neurological deficit or deformity occur. Such reconstruction may be successful if an aggressive debridement of all infectious foci is done, even if instrumentation or allograft is used. The optimal approach is dictated by the location of the infection and the type and degree of deformity (and is often anterior or anterior-posterior).Correct Answer: Always involve an aggressive debridement
2511. (1229) Q6-1598:
The spinal surgical procedure associated with the highest rate of surgical site infection is:
1) Neuromuscular scoliosis fusion
3) Reduction and fusion of traumatic cervical facet fracture-dislocation
2) Lumbar spondylolisthesis fusion
5) Cervical laminectomy, foraminotomy, and arthrodesis
4) Lumbar stenosis decompression and fusion
Postoperative infection rates reach 11% for neuromuscular disease indications. For muscular dystrophy scoliosis surgery, the rate may be as high as 23%, for cerebral palsy 18%, and for myelomeningocele 11%.Correct Answer: Neuromuscular scoliosis fusion
2512. (1230) Q6-1599:
Which of the following comprises the middle column in the Denis three-column model of the thoracolumbar spine:
1) Posterior longitudinal ligament, spinal canal, pedicles, and facet joints
3) Vertebral body, posterior longitudinal ligament, and disk
2) Facet joints, intertransverse membrane, and ligamentum flavum
5) Interspinous ligament, supraspinous ligament, and ligamentum flavum
4) Posterior half of the vertebral body, posterior half of the disk, and posterior longitudinal ligament
The middle column is composed of the posterior half of the vertebral body, posterior half of the disk, and posterior longitudinal ligament. The middle column, according to Denis, is important to determine the stability of a thoracolumbar fracture. There is the potential for instability when the middle column is disrupted.Correct Answer: Posterior half of the vertebral body, posterior half of the disk, and posterior longitudinal ligament
2513. (1231) Q6-1600:
Slide 1
Which type of biomechanical force(s) acts on the anterior portion of the thoracolumbar junction (T12-L2) at rest in a standing position:
1) Compression
3) Compression and tension
2) Compression and shear
5) Shear and torsion
4) Tension
The thoracolumbar junction is normally a straight portion of the spine (no lordosis or kyphosis) and the vertebral bodies are subject to compressive forces at rest when the patient is in a standing position. The posterior osteoligamentous structures are subject to tension along with the paraspinous muscles that help to maintain an upright posture.Correct Answer: Compression
2514. (1232) Q6-1601:
Which of the following statements is true regarding the bulbocavernosus reflex:
1) This reflex is a sign of a spinal cord injury.
3) This reflex may be elicited by pulling on an indwelling catheter that causes a contraction of the cremaster muscle.
2) This reflex is mediated by the S3 and S4 segments of the spinal cord.
5) This reflex is mediated by the S3 and S4 segments of the spinal cord, and this reflex often means that a spinal cord injury is complete.
4) This reflex often means that a spinal cord injury is complete.
The bulbocavernosus reflex is mediated by the S3 and S4 regions of the spinal cord. This reflex is elicited by pulling on an indwelling catheter or squeezing the glans penis or clitoris and observing contraction of the anal sphincter. The bulbocavernosus reflex may be absent soon after a spinal cord injury due to spinal shock, but it often returns in 24 to 48 hours and indicates the end of spinal shock. A better sense of prognosis of a spinal cord injury is possible after spinal shock has ended.Correct Answer: This reflex is mediated by the S3 and S4 segments of the spinal cord.
2515. (1233) Q6-1602:
According to the Frankel grading scale for a neurologic injury, what is meant by Frankel C:
1) There is an antigravity motor function in some distal muscles below the level of the spinal cord injury.
3) There is muscle function, but not with adequate power to overcome gravity in some muscles below the level of the spinal cord injury.
2) There is an antigravity muscle function in the muscles 1 or 2 root levels below the spinal cord injury.
5) The patient has a chance for further neurologic recovery.
4) There is muscle function, but not with adequate power to overcome gravity in the muscles 1 or 2 root levels below the spinal cord injury.
The Frankel grading scale is used to communicate the extent of neurologic injury in the setting of a spinal cord injury.
The A category indicates that there is no motor or sensory function below the level of the injury. The B category indicates that there is only sensory function below the level of the injury.
The C category indicates that there is muscle function, but not with adequate power to overcome gravity in some muscles below the level of the spinal cord injury.
The D category indicates that there is motor function with at least antigravity power below the level of the injury.
The E category indicates that the muscle function below the level of the injury is normal in power. One can see some motor function for 1-2 root levels below the level of a spinal cord injury that is due to âroot escapeâ and should not be confused with distal motor sparing.
Correct Answer: There is muscle function, but not with adequate power to overcome gravity in some muscles below the level of the spinal cord injury.
2516. (1234) Q6-1603:
Which type of thoracolumbar fracture is associated with the highest incidence of intra-abdominal visceral trauma:
1) Burst fracture
3) Flexion-distraction injury
2) Compression fracture
5) Pathologic fracture
4) Fracture dislocation
The flexion-distraction injury was originally termed the âseatbelt injuryâ or in the case of a fracture proceeding through bone, a âChance fracture.â This injury is usually the result of a severe flexion force to the lumbar spine with flexion moment anterior to the spine (e.g., at a lap belt). Due to the severe energy dissipation at the level of the flexion moment, there is a high incidence of intra-abdominal visceral trauma.Correct Answer: Flexion-distraction injury
2517. (1235) Q6-1604:
Which type of treatment would be most appropriate for a young, healthy patient with an incomplete spinal cord injury (ASIA C) 5 days following a T12 burst fracture with 30% canal compromise:
1) Bed rest, followed by hyperextension casting
3) Posterior fusion in situ
2) Posterior distractive instrumentation and fusion
5) Anterior T12 corpectomy, strut grafting, and instrumentation
4) Anterior T12 corpectomy and strut grafting
Surgery is indicated in patients with an incomplete spinal cord injury with spinal cord compression. Although some indirect decompression may be achieved early following the injury using posterior distractive instrumentation, the level of decompression is often better using an anterior approach (especially several days following the fracture). Following anterior decompression, either anterior instrumentation or posterior instrumentation is indicated to stabilize the construct and allow early mobilization.Correct Answer: Anterior T12 corpectomy, strut grafting, and instrumentation
2518. (1236) Q6-1605:
Which of the following is the best indication for a laminectomy in a patient who has sustained a thoracolumbar burst fracture with a neurologic deficit:
1) Spinal cord compression
3) A small epidural hematoma is present on a magnetic resonance image
2) A lamina fracture is present on a computerized tomography scan
5) Greater than 50% canal compromise on a computerized tomography scan
4) Greater than 30° of kyphosis on a lateral radiograph
A laminectomy is never indicated as the sole method of treatment for a thoracolumbar burst fracture. Laminectomy creates additional instability at the level of the fracture and does not effectively decompress the spinal cord, which is compressed anteriorly from the retropulsed bony fragment. When lamina fractures are present on a computerized tomography scan, there is a significant incidence of dural tears and entrapped nerve tissue within the lamina fracture. Surgeons should consider performing a laminectomy in addition to other methods of achieving anterior decompression and stabilization of a burst fracture with a lamina fracture.Correct Answer: A lamina fracture is present on a computerized tomography scan
1) Treat all patients with a spinal cord injury with methylprednisolone 30 mg/kg over 1 hr followed by a maintenance rate of 5.4 mg/kg/hr for 23 hours.
3) Treat all patients with a spinal cord injury with decadron 10 mg/kg bolus followed by 1 mg/kg/hr for 23 hours.
2) Treat only patients who present within the first 8 hours of a spinal cord injury with methylprednisolone 30 mg/kg over 1 hr followed by a maintenance rate of 5.4 mg/kg/hr for 23 hours.
5) Treat only patients who present with complete spinal cord injuries within the first 8 hours of a spinal cord injury with methylprednisolone 30 mg/kg over 1 hr followed by a maintenance rate of 5.4 mg/kg/hr for 23 hours.
4) Treat only patients who present within the first 8 hours of a spinal cord injury with decadron 10 mg/kg bolus followed by 1 mg/kg/hr for 23 hours.
The NASCIS-II recommendations are to treat patients who present with an incomplete spinal cord injury within 8 hours of the injury with methylprednisolone 30 mg/kg over 1 hour followed by a maintenance rate of 5.4 mg/kg/hr for 23 hours. Because it is difficult to tell which patients have a complete or incomplete spinal cord injury in this time frame due to spinal shock, it has generally been accepted to treat all patients with spinal cord injuries with this treatment protocol as long as they present within the first 8 hours of the injury.Correct Answer: Treat only patients who present within the first 8 hours of a spinal cord injury with methylprednisolone 30 mg/kg over 1 hr followed by a maintenance rate of 5.4 mg/kg/hr for 23 hours.
2520. (1238) Q6-1607:
Which of the following statements is true regarding neurogenic shock:
1) Neurogenic shock is due to severe blood loss associated with a spinal cord injury.
3) Neurogenic shock is due to increased parasympathetic tone.
2) Neurogenic shock can be diagnosed when there is hypotension and tachycardia.
5) Neurogenic shock is a sign of an incomplete spinal cord injury.
4) Neurogenic shock is best treated with judicious use of fluids and vasopressors.
Neurogenic shock is present when there is a spinal cord injury interrupting sympathetic tone to the heart and blood vessels, and it is heralded by bradycardia and hypotension. It is important to maintain a reasonable blood pressure to prevent further damage to the spinal cord due to ischemia. In the absence of significant blood loss from another source, neurogenic shock must be treated with vasopressor medication and atropine. Severe neurogenic shock may require cardiac pacing. Fluids must be used carefully as overzealous use of fluid resuscitation can result in pulmonary edema.Correct Answer: Neurogenic shock is best treated with judicious use of fluids and vasopressors.
2521. (1239) Q6-1608:
Which is the best indication for surgical treatment of a patient with a thoracolumbar burst fracture:
1) 60% canal compromise by a retropulsed bony fragment
3) A fracture of the lamina present on a computerized tomography scan
2) 25° of kyphosis on the lateral radiograph
5) A polytrauma patient
4) An incomplete neurologic deficit
The exact indications for surgery vs nonoperative management of thoracolumbar burst fractures remains controversial. The best indication is an incomplete neurologic deficit with spinal cord compression. Other considerations include the degree of deformity (greater than 30° is generally considered appropriate to consider surgery) and the other injuries. Although much has been written about canal compromise, in the absence of a neurologic deficit it is not clear that surgery is always indicated due to canal compromise alone. Large canal fragments have been shown to resorb with conservative treatment.Correct Answer: An incomplete neurologic deficit
1) Football lineman
3) Eskimo
2) Gymnast
5) Weight lifter
4) Nonambulatory patient
Isthmic spondylolisthesis is most common in white men and least common in black women. It is thought to arise from repetitive hyperextension of the lumbar spine causing a stress fracture of the pars intra-articularis. Sports such as weight lifting, gymnastics, football, and javelin throwing have a particularly high incidence of this condition. Isthmic spondylolisthesis is never present at birth and is rare in nonambulatory patients.Correct Answer: Nonambulatory patient
2523. (1241) Q6-1610:
Which patient is at the lowest risk for progression of spondylolisthesis:
1) A 45-year-old man with grade II isthmic spondylolisthesis at L5-S1.
3) A 25-year-old man with a grade I isthmic spondylolisthesis at L4-5.
2) A 5-year-old girl with grade I dysplastic spondylolisthesis at L5-S1.
5) A 12-year-old girl with Grade II isthmic spondylolisthesis at L5-S1.
4) A 16-year-old boy with Grade III isthmic spondylolisthesis at L5-S1.
Young age, dysplastic spondylolisthesis, and spondylolisthesis above L5-S1 are all risk factors for progression. Adults with isthmic spondylolisthesis at L5-S1 (85% of cases) are at a low risk for progression of the slippage.Correct Answer: A 45-year-old man with grade II isthmic spondylolisthesis at L5-S1.
2524. (1242) Q6-1612:
What is the most common source of neurologic compression in a patient with lumbar spinal stenosis due to degenerative changes in the lumbar spine:
1) Disk
3) Superior articular process
2) Inferior articular process
5) Lamina
4) Pars intrarticularis
Degenerative spinal stenosis is the most common variety of spinal stenosis and usually manifests compression of the thecal sac in the lateral recess of the canal (defined as the area of the spinal canal between the facet joints and the intervertebral disk). The primary cause of stenosis is hypertrophy of the facet joint with compression from the superior articular process. This must be relieved for a patient to achieve an adequate decompression.Correct Answer: Superior articular process
2525. (1940) Q6-2350:
Which of the following cervical spine nerve roots may cause paralysis of the diaphragm if injured during an anterior approach:
1) C 3
3) C 5
2) C 4
5) C 7
4) C 6
The C 4 cervical spine nerve root provides the primary innervation of the diaphragm.Correct Answer: C 4
1) C 4
3) C 6
2) C 5
5) C 8
4) C 7
The deltoid muscle is almost entirely innervated by the C 5 cervical spine nerve root.Correct Answer: C 5
2527. (1942) Q6-2352:
The biceps reflex is diminished by compression of which of the following cervical spine nerve roots:
1) C 4
3) C 6
2) C 5
5) C 8
4) C 7
Although there is a small contribution from the C 6 cervical spine nerve root, the biceps reflex is primarily derived from the C 5 cervical spine nerve root.Correct Answer: C 5
2528. (1943) Q6-2353:
If the C 5 cervical spine nerve root is injured during a decompression of the cervical spine, then sensation is lost over which of the following areas:
1) The lateral aspect of the arm from the shoulder to the elbow
3) The lateral border of the forearm including the thumb
2) The medial aspect of the arm from the shoulder to the elbow
5) The medial border of the forearm including the little finger
4) The middle finger
The C 5 cervical spine nerve root supplies sensation from the lateral aspect of the arm from the shoulder to the elbow.
C 5 â Lateral aspect of the arm from the shoulder to the elbow C 6 â Lateral border of the forearm including the thumb
C 7 â Middle finger
C 8 â Medial border of the forearm including the little finger T1 â Medial aspect of the arm from the shoulder to the elbow
Correct Answer: The lateral aspect of the arm from the shoulder to the elbow
2529. (1944) Q6-2354:
If the C 6 cervical spine nerve root is injured during a posterior decompression of the cervical spine, then sensation is lost in which of the following areas:
1) The lateral aspect of the arm from the shoulder to the elbow
3) The lateral border of the forearm including the thumb
2) The medial aspect of the arm from the shoulder to the elbow
5) The medial border of the forearm including the little finger
4) The middle finger
If the C 6 cervical spine nerve root is injured during a posterior decompression of the cervical spine, then sensation is lost in which of the following areas:
C 5 â Lateral aspect of the arm from the shoulder to the elbow C 6 â Lateral border of the forearm including the thumb
C 7 â Middle finger
C 8 â Medial border of the forearm including the little finger T1 â Medial aspect of the arm from the shoulder to the elbow
Correct Answer: The lateral border of the forearm including the thumb
2530. (1945) Q6-2355:
If the C 7 cervical spine nerve root is injured during a posterior decompression of the cervical spine, then sensation is lost in which of the following areas:
1) The lateral aspect of the arm from the shoulder to the elbow
3) The lateral border of the forearm including the thumb
2) The medial aspect of the arm from the shoulder to the elbow
5) The medial border of the forearm including the little finger
4) The middle finger
The C 7 cervical spine nerve root supplies sensation to the skin over the volar aspect of the middle finger.
C 5 â Lateral aspect of the arm from the shoulder to the elbow C 6 â Lateral border of the forearm including the thumb
C 7 â Middle finger
C 8 â Medial border of the forearm including the little finger T1 â Medial aspect of the arm from the shoulder to the elbow
Correct Answer: The middle finger
2531. (1946) Q6-2356:
If the C 8 cervical spine nerve root is injured during a posterior spinal decompression, then sensation is lost over which of the following areas:
1) The lateral aspect of the arm from the shoulder to the elbow
3) The lateral border of the forearm including the thumb
2) The medial aspect of the arm from the shoulder to the elbow
5) The medial border of the forearm including the little finger
4) The middle finger
The C 8 cervical spine nerve root supplies sensation to the medial border of the forearm including the little finger.
C 5 â Lateral aspect of the arm from the shoulder to the elbow C 6 â Lateral border of the forearm including the thumb
C 7 â Middle finger
C 8 â Medial border of the forearm including the little finger T1 â Medial aspect of the arm from the shoulder to the elbow
Correct Answer: The medial border of the forearm including the little finger
2532. (1947) Q6-2357:
If the brachioradialis reflex is diminished after a posterior spinal decompression, then which of the following nerve roots is injured:
1) C 5
3) C 7
2) C 6
5) T1
4) C 8
The brachioradialis reflex is mediated by the C 6 cervical spine nerve root.
C 5 â Biceps
C 6 â Brachioradialis C 7 â Triceps
Correct Answer: C 6
1) C 5
3) C 7
2) C 6
5) T1
4) C 8
The triceps muscle extends the elbow and is innervated by the C 7 cervical spine nerve root. Motor innervations include:
Shoulder abduction (deltoid) - - C 5 Elbow flexion - - C 5
Wrist extension - - C 6, C 7 Wrist flexion - - C 7
Finger extension - - C 7 Finger flexion - - C 8
Finger abduction/adduction - - T1 Correct Answer: C 7
2534. (1949) Q6-2359:
If the flexor carpi radialis is weak after a spinal decompression, then which of the following nerve roots is injured:
1) C 5
3) C 7
2) C 6
5) T1
4) C 8
The flexor carpi radialis is the most powerful wrist flexor and is innervated by the C 7 cervical spine nerve root. The flexor carpi ulnaris, which is weaker than the flexor carpi radialis, is innervated by the C 8 cervical spine nerve root.Correct Answer: C 7
2535. (1950) Q6-2360:
A patient has a fracture dislocation of the cervical spine. Which of the following nerve roots must be spared to preserve intact finger extension:
1) C 5
3) C 7
2) C 6
5) T1
4) C 8
Finger extensors are innervated by the C 7 cervical spine nerve root. Motor innervations include:
Shoulder abduction (deltoid) - - C 5 Elbow flexion - - C 5
Wrist extension - - C 6, C 7 Wrist flexion - - C 7
Finger extension - - C 7 Finger flexion - - C 8
Finger abduction/adduction - - T1 Correct Answer: C 7
1) C 5
3) C 7
2) C 6
5) T1
4) C 8
The FDS flexes the proximal interphalangeal joint and is innervated by the C 8 cervical spine nerve root. The FDS is innervated peripherally by the median nerve.
The flexor digitorum profundus flexes the distal interphalangeal joint and is also innervated by the C 8 cervical spine nerve root. The middle and index fingers are supplied by the median nerve, and the ring and little fingers are supplied by the ulnar nerve.Correct Answer: C 8
2537. (1952) Q6-2364:
A patient with a herniated disk has a diminished patellar tendon reflex. Which of the following lumbosacral nerve roots is affected:
1) L1
3) L3
2) L2
5) L5
4) L4
The patellar tendon reflex is primarily transmitted through the L4 lumbosacral nerve root. Although the L4 lumbosacral nerve root is the primary transmitter, the L2 and L3 lumbosacral nerve roots also contribute to the fibers. A weak reflex is present if the L4 lumbosacral nerve root is completely cut and fibers of the L2 and L3 lumbosacral nerve roots are still present. The patellar tendon reflex is seldom completely absent unless a patient has primary muscle or anterior horn lesions.Correct Answer: L4
2538. (1953) Q6-2365:
A patient with radicular pain is experiencing skin numbness on the medial aspect of his leg and great toe. Which of the following nerve roots is effected:
1) L2
3) L4
2) L3
5) S1
4) L5
When examining patients, it is important to remember the sensory dermatomes. The medial aspect of the leg, foot, and great toe are supplied by the L4 lumbosacral nerve root. The tibial crest separates the L4 and L5 dermatomes on the leg.
L4 Medial aspect of leg, foot, and great toe
L5 Lateral aspect of the leg and toes 2 through 4 S1 Lateral aspect of the fifth toe
Correct Answer: L4
roots is compressed:
1) L1
3) L3
2) L2
5) L5
4) L4
The extensor hallucis longus muscle is primarily innervated by the L5 lumbosacral nerve root. The L5 lumbosacral nerve root innervates the following muscles:
Extensor hallucis longus
Extensor digitorum longus and extensor digitorum brevis Gluteus medius
Correct Answer: L5
2540. (1955) Q6-2367:
If the extensor digitorum longus and extensor digitorum brevis muscles are weak in a patient who has radicular back pain, then which of the following lumbosacral nerve roots is compressed:
1) L1
3) L3
2) L2
5) L5
4) L4
The extensor hallucis longus muscle is primarily innervated by the L5 lumbosacral nerve root. The L5 lumbosacral nerve root innervates the following muscles:
Extensor hallucis longus
Extensor digitorum longus and extensor digitorum brevis Gluteus medius
Correct Answer: L5
2541. (1956) Q6-2368:
Testing of the L5 lumbosacral nerve root in a patient who has radicular back pain can be accomplished through which of the following reflexes or tests:
1) Patellar tendon reflex
3) Tibialis posterior reflex
2) Achilles tendon reflex
5) Beevor sign
4) Superficial anal reflex
Although there is not a well-defined reflex arc for the L5 lumbosacral nerve root, the tibialis posterior reflex can be elicited. The tibialis posterior reflex is mediated through the L5 lumbosacral nerve root.
Reflexes and associated nerve roots include: Patellar tendon --- L4
Achilles tendon --- S1
Superficial anal reflex --- S2, S3, S4
Beevor sign refers to asymmetry of the segmental innervation of the rectus abdominus muscles and when performing a sit-up, there is unilateral segmental nerve root loss.
Correct Answer: Tibialis posterior reflex
A patient with radicular pain is experiencing skin numbness on the lateral aspect of the leg and the dorsum of the foot between the second and fourth toes. Which of the following nerve roots is being compressed:
1) L1
3) L3
2) L2
5) L5
4) L4
The L5 dermatome covers the skin on the lateral leg and dorsum of the foot from the lateral border of the great toe to the medial border of the little toe.
L4 Medial aspect of leg, foot, and great toe
L5 Lateral aspect of the leg and toes 2 through 4 S1 Lateral aspect of the fifth toe
Correct Answer: L5
2543. (1958) Q6-2370:
If the peroneus longus and peroneus brevis muscles are weak in a patient who has radicular back pain, then which of the following nerve roots is compressed:
1) L3
3) L5
2) L4
5) S2
4) S1
The peroneus brevis and peroneus longus muscles are principally innervated by the S1 nerve root through the superficial peroneal nerve. Although the nerve is principally innervated by the S1 nerve root, the superficial peroneal nerve is derived from the L5, S1, and S2 nerve roots.
The muscles principally innervated by the S1 nerve root are the: Peroneus longus and peroneus brevis
Gastrocnemius-soleus complex Gluteus maximus
Correct Answer: S1
2544. (1959) Q6-2371:
The left medial and lateral gastrocnemius muscles are weak in a patient after a lumbar spine decompression. Which of the following nerve roots is injured:
1) L3
3) L5
2) L4
5) S2
4) S1
The medial and lateral gastrocnemius muscles are principally innervated by the S1 nerve root through the tibial nerve. Although the nerve is principally innervated by the S1 nerve root, the tibial nerve is derived from the L5, S1, and S2 nerve roots.
The muscles principally innervated by the S1 nerve root are the: Peroneus longus and peroneus brevis
Gastrocnemius-soleus complex Gluteus maximus
Correct Answer: S1
The Achilles tendon reflex (ankle reflex) is absent in a patient who has radicular back pain. Which of the following nerve roots is compressed:
1) L4
3) S1
2) L5
5) S3
4) S2
The Achilles tendon reflex is based on the triceps muscle group (medial and lateral gastrocnemius muscles and soleus muscle) and is transmitted through the S1 nerve root.
Reflexes and associated nerve roots include: Patellar tendon reflex L4
Posterior tibial reflex L5 Achilles tendon reflex S1
Correct Answer: S1
2546. (1961) Q6-2373:
A patient with cauda equina syndrome has decreased perianal sensation. Which of the following groups of nerve roots is involved:
1) L2, L3, and L4
3) L5, S1, and S2
2) L3, L4, and L5
5) S2, S3, S4, and S5
4) S1, S2, and S3
Perianal sensation is derived from the S2, S3, S4, and S5 nerve roots. The sensory distribution is as follows:
S4-S5 - - Innermost perianal ring S3 - - Middle perianal ring
S2 - - Outermost perianal ring Correct Answer: S2, S3, S4, and S5
2547. (1962) Q6-2374:
A patient with a fracture dislocation of the spine has a sensory level at the nipple line. Which of the following nerve root levels indicates this finding:
1) T2
3) T7
2) T4
5) T12
4) T10
In addition to knowing the innervation of selected muscles and the deep tendon reflexes, the clinician should also know the sensory levels to localize pathologic processes.
T4 Nipple line
T7 Xiphoid process T10 Umbilicus
T12 Groin Correct Answer: T4
A patient with a fracture dislocation of the spine has a sensory level at the xiphoid process. Which of the following nerve root levels indicates this finding:
1) T2
3) T7
2) T4
5) T12
4) T10
The skin over the xiphoid process area is innervated by the T7 nerve root.
In addition to knowing the innervation of selected muscles and the deep tendon reflexes, the clinician should also know the sensory levels to localize pathologic processes.
T4 Nipple line
T7 Xiphoid process T10 Umbilicus
T12 Groin Correct Answer: T7
2549. (1964) Q6-2376:
A patient with a fracture dislocation of the spine has a sensory level at the umbilicus. Which of the following nerve root levels indicates this finding:
1) T2
3) T7
2) T4
5) T12
4) T10
The skin of the umbilicus is innervated by the T10 nerve root.br>
In addition to knowing the innervation of selected muscles and the deep tendon reflexes, the clinician should also know the sensory levels to localize pathologic processes.
T4 Nipple line
T7 Xiphoid process T10 Umbilicus
T12 Groin Correct Answer: T10
2550. (1965) Q6-2377:
Which of the following statements regarding the presentation of thoracic disk herniations is false:
1) Pain is the principal symptom.
3) Mechanical axial back pain may be present.
2) Radicular pain may be present.
5) Bowel and bladder symptoms occur in more than 50% of affected patients.
4) Myelopathic pain may be present.
Patients with thoracic disk herniations may present with mechanical axial back pain, radicular pain, or myelopathy, but pain is the principal symptom. Bowel and bladder symptoms occur in 10% to 20% of affected patients.Correct Answer: Bowel and bladder symptoms occur in more than 50% of affected patients.
Which of the following statements regarding the treatment of thoracic disk herniations is true:
1) The majority of patients can be treated nonoperatively.
3) Laminectomy is the surgical procedure of choice.
2) Surgical decompression is necessary in most cases.
5) Costotransversectomy is used for large central calcified herniations.
4) The anterior transthoracic approach is used for T1-T4 lesions.
The majority (75%) of patients with thoracic disk herniations may be managed nonoperatively. Surgical procedures must adequately decompress the involved nerve root. Posterior approach by laminectomy is usually not adequate, and costotransversectomy is not effective for large central calcified herniations (an anterior approach is preferred). The anterior transthoracic approach is effective for T5-T12 lateral and anterior disk herniations.Correct Answer: The majority of patients can be treated nonoperatively.
2552. (2177) Q6-2603:
Which of the following is the only accepted pharmacological agent for the acute treatment of a spinal cord injury:
1) GM-1 ganglioside
3) Nimodipine (calcium channel blocker)
2) Tirilazad (lipid peroxidation inhibitor)
5) Methylprednisolone
4) Naloxone (opoid antagonist)
Methylprednisolone is currently the only accepted pharmacologic agent for the treatment of spinal cord injury. The North American Spinal Cord Injury Studies (NASCIS) found significant motor and sensory improvement in patients who were treated within 8 hours of injury with a methylprednisolone bolus of 30 mg/kg, followed by an infusion of 5.4 mg per hour for 24 hours.
Other agents have been studied in animal experiments but have not been promising in clinical trials.Correct Answer: Methylprednisolone
2553. (2178) Q6-2604:
Slide 1
A 45-year-old man has neck pain following a motor vehicle accident. His neurologic examination is normal. His plain radiographs are shown (Slide). The most likely diagnosis is:
1) Cervical strain (whiplash-type injury)
3) Unilateral facet dislocation
2) Compression fracture of C 5
5) Spinous process fracture
4) Bilateral facet dislocation
The lateral radiograph shows translation and kyphosis at the level of injury. The facets of C 4 do not superimpose on each to create a "double sail" sign. This patient has a unilateral facet dislocation. With unilateral facet dislocations, there is usually 3 mm to 4 mm of forward translation and 5° to 7° of angulation.Correct Answer: Unilateral facet dislocation
Slide 1
A 35-year-old man has neck pain following a motor vehicle accident. His axial computed tomography scan is shown (Slide). The most likely diagnosis is:
1) C 4 compression fracture
3) Bilateral facet dislocation
2) Clay shovelers fracture
5) Pseudosubluxation of C 4 on C 5
4) Unilateral facet dislocation
The axial computed tomography scan of C 4-C 5 shows a unilateral facet dislocation. Notice that the superior facet of C 5 lies posterior to the inferior facet of C 4. This relationship should be the exact opposite. Also, notice that C 4 is rotated on the body of C 5 and translated forward.Correct Answer: Unilateral facet dislocation
2555. (2180) Q6-2607:
Slide 1
A 40-year-old woman has severe neck pain following a motor vehicle accident. Her plain lateral radiograph of the spine is shown (Slide). A sagittal magnetic resonance scan is shown (Slide). The most likely diagnosis is:
1) Pseudosubluxation of C 5 on C 6
3) Unilateral facet dislocation
2) Compression fracture of C 5
5) Degenerative sponylolisthesis C 5 on C 6
4) Bilateral facet dislocation
There is significant subluxation of C 5 on C 6 on the plain radiograph. The facets of C 5 and C 6 have lost their normal relationship. This patient has a bilateral facet dislocation. There is compression and significant changes within the spinal cord. This patient should be treated with reduction and fusion.Correct Answer: Bilateral facet dislocation
Slide 1
A 40-year-old woman has severe neck pain following a motor vehicle accident. Her plain lateral radiograph of the spine is shown (Slide). A sagittal magnetic resonance is shown (Slide). The most appropriate treatment would be:
1) Observation
3) Reduction and collar immobilization
2) Neck collar and physical therapy
5) Reduction and fusion
4) Reduction and halo immobilization
There is significant subluxation of C 5 on C 6 on the plain radiograph. The facets of C 5 and C 6 have lost their normal relationship. This patient has a bilateral facet dislocation. There is compression and significant changes within the spinal cord.
This patient should be treated with reduction and fusion. This is a ligamentous injury so reduction and immobilization will not result in satisfactory healing.Correct Answer: Reduction and fusion
2557. (2182) Q6-2609:
In which of the following nerve roots is compression neuropathy common in cervical spondylosis:
1) C 3 and C 4
3) C 5 and C 6
2) C 4 and C 5
5) C 7 and C 8
4) C 6 and C 7
The nerve roots that are most commonly affected in cervical spondylosis are C 6 and C 7, secondary to degenerative changes in the C 5-C 6 and C 6-C 7 nerve roots. Patients may have specific dermatomal pain or pain that is diffuse and poorly localized.Correct Answer: C 6 and C 7
2558. (2183) Q6-2610:
Which of the following sensory areas is affected by compression of the C 6 nerve root:
1) Lateral forearm into the radial side of the hand
3) Posterior neck, occiput
2) Base of neck, medial shoulder
5) Posterolateral forearm into the middle finger of the hand
4) Ulnar side of the forearm and hand
It is important to remember the sensory dermatome when examining patients who have neck and upper extremity pain: C3 Posterior neck, occiput
C4 Base of neck, medial shoulder
C5 Base of neck to shoulder and upper arm
C6 Lateral forearm into the radial side of the hand
C7 Posterolateral forearm into the middle finger of the hand C8 Ulnar side of the forearm and hand
Correct Answer: Lateral forearm into the radial side of the hand
Which of the following sensory areas is affected by compression of the C 7 nerve root:
1) Base of neck, medial shoulder
3) Lateral forearm into the radial side of the hand
2) Base of neck to shoulder and upper arm
5) Ulnar side of the forearm and hand
4) Posterolateral forearm into the middle finger of the hand
It is important to remember the sensory dermatome when examining patients who have neck and upper extremity pain: C3 Posterior neck, occiput
C4 Base of neck, medial shoulder
C5 Base of neck to shoulder and upper arm
C6 Lateral forearm into the radial side of the hand
C7 Posterolateral forearm into the middle finger of the hand C8 Ulnar side of the forearm and hand
Correct Answer: Posterolateral forearm into the middle finger of the hand 2560. (2185) Q6-2612:
Slide 1
A patient comes into your office with neck and arm pain. The patientâs plain radiograph is shown (Slide). Which of the following signs is most likely to be found on physical examination:
1) Triceps muscle weakness
3) Finger flexion weakness
2) Deltoid muscle weakness
5) Numbness on the ulnar side of the forearm and hand
4) Numbness on the lateral forearm into the radial side of the hand
The lateral radiograph shows narrowing of the C 5-C 6 intervertebral disk space with osteophytes arising anteriorly and posteriorly. This degenerative process results in facet joint hypertrophy, osteophytes in the uncovertebral joints, and hypertrophy of the ligamentum flavum. The C 6 nerve root is compressed resulting in numbness on the lateral forearm into the radial side of the hand. Elbow and wrist extension may be affected, and the biceps tendon reflex may be diminished or absent.
It is important to remember the sensory dermatome when examining patients who have neck and upper extremity pain: C3 Posterior neck, occiput
C4 Base of neck, medial shoulder
C5 Base of neck to shoulder and upper arm
C6 Lateral forearm into the radial side of the hand
C7 Posterolateral forearm into the middle finger of the hand C8 Ulnar side of the forearm and hand
Correct Answer: Numbness on the lateral forearm into the radial side of the hand
Slide 1 Slide 2
A 45-year-old woman has pain in her right upper extremity and neck. The plain film is is presented (Slide 1) as well as an axial post myelogram CT images (Slide 2, A & B). Her pain has not responded to nonsteroidal anti-inflammatory drugs or physical therapy. Which of the following is the most appropriate treatment:
1) Needle aspiration of the C 4-C 5 intervertebral space
3) Needle biopsy of C 4 or C 5
2) 6-week course of antibiotics and bracing
5) Anterior cervical diskectomy
4) Mammography and technetium bone scan
The axial post myelogram CT image shows a disk herniation, and the sagittal view shows prominent osteophytes. There is no evidence of an infection or a neoplasm. This patient is a candidate for anterior disckectomy and fusion.Correct Answer: Anterior cervical diskectomy
2562. (2327) Q6-2781:
Plain radiographs of the lumbosacral spine are useful for:
1) Detecting marrow changes
3) Distinction between disk material and the dural sac
2) Far lateral herniated intervertebral disks
5) Instability patterns
4) Lateral recess stenosis
Plain radiographs are useful for assessing the alignment of the spine, bone destruction by tumors and infections, and instability patterns. The radiographs also will show degenerative intervertebral disks.
Plain radiographs are not sensitive for detecting marrow changes, herniated disks, and neural compression secondary to degenerative changes.
Correct Answer: Instability patterns
2563. (2328) Q6-2782:
Computerized tomography scans are efficacious for detecting which of the following conditions:
1) Marrow changes
3) Intrathecal abnormalities
2) Lytic lesion suspected on Plain L spine film
5) Intervertebral disk hydration
4) Instability patterns
Computerized tomography scans are excellent for assessing bone structure, especially in patients with metastatic bone disease and primary bone tumors of the spine. Computerized tomography is useful for distinguishing between bone and soft tissue compression in neural compressive disorders.Correct Answer: Lytic lesion suspected on Plain L spine film
A 35-year-old construction worker has left leg pain and difficulty walking. His examination is normal except for decreased sensation to the lateral border of the left foot, the inability to walk on the toes of the left foot, and a positive stretch test producing left heel and lateral foot pain. A magnetic resonance image shows a large posterolateral herniated nucleus pulposus on the left side at L5-S1. The gait abnormality is most likely due to:
1) Cauda equina syndrome
3) L5 radiculopathy and extensor hallucis longus weakness
2) L5 radiculopathy and gastrocsoleus muscle complex denervation
5) S2 denervation and extensor hallucis longus weakness
4) S1 radiculopathy and gastrocsoleus muscle complex denervation
In the lumbar spine, direct posterior and posterolateral disk herniations typically compress the traversing nerve root. In this patient, the herniated disk at the L5-S1 level compresses the shoulder of the S1 nerve root as it comes off the dural sac. The S1 nerve root supplies sensation to the posterior calf and lateral border of the foot, and motor chiefly to the gastrocsoleus muscle complex.Correct Answer: S1 radiculopathy and gastrocsoleus muscle complex denervation
2565. (2406) Q6-2865:
Which of the following types of neural dysfunction is present with a cervical fracture-dislocation, resulting in a Brown-Sequard neurological injury:
1) Ipsilateral loss of pain, temperature recognition, and contralateral loss of motor function
3) Bilateral loss of pain/temperature and unilateral loss of motor
2) Ipsilateral loss of motor and contralateral loss of pain/temperature
5) Bilateral upper extremity loss of motor and unilateral lower extremity loss of pain/temperature
4) Bilateral loss of motor and unilateral loss of pain/temperature
A Brown-Sequard injury causes damage to half of the spinal cord. Brown-Sequard injuries produce ipsilateral proprioceptive and motor loss and contralateral loss of sensitivity to pain and temperature. Proprioceptive sensory fibers enter the spinal cord, travel in the dorsal columns and lateral and ventral spinothalmic tracts, and decussate high in the thalamus. Motor efferent nerves cross in the medulla and travel down in the lateral corticospinal tracts.Spinthalamic fibers enter and decussate in the spinal cord. Hence, cord hemi-section produces contralateral pain and temperature (spinothalamic) loss, and ipsilateral motor (corticospinal) and, proprioceptive (dorsal columns) deficit. Often due to penetrating injuries, Brown-Sequard injuries have the best prognosis of the cord injury complexes.Correct Answer: Ipsilateral loss of motor and contralateral loss of pain/temperature
2566. (2407) Q6-2866:
Slide 1
The axial computed tomography scan depicts a patient with spinal stenosis (Slide). The primary source of neural compression is impingement on the traversing nerve root by the:
1) Superior facet of the level below
3) Redundant ligamentum flavum
2) Inferior facet of the level above
5) Herniated nucleus pulposus
4) Overgrown medial pedicle
Spinal stenosis involves narrowing of the spinal canal by a combination of factors. Degeneration of the disk with dehydration allows loss of disk height and bulging posteriorly into the canal. The ligamentum flavum becomes redundant at the segment due to loss of the disk height and buckling of the ligament. Chief among the sources of compression, however, is the overgrowth of the facet joint, which acts to autostabilize the motion segment. The facets are oriented in an oblique plane, depending on the level involved. The superior facet of the subjacent vertebral body lies anterior and lateral to its counterpart from the level above, forming a shingle configuration. The superior articular process, therefore, lies adjacent to the shoulder of the traversing nerve root and is a significant source of lateral recess stenosis.Correct Answer: Superior facet of the level below
Slide 1
The type of disk herniation shown (Slide) at the L5-S1 level is most likely to cause:
1) Quadriceps weakness and numbness of the medial thigh
3) Weakness of the tibialis anterior
2) Diminished sensation to the anteromedial calf
5) Extensor hallucis longus weakness
4) Weakness of the gastrocsoleus complex
This slide shows a posterolateral disk herniation on the right. Posterolateral disk herniations cause compression of the traversing S1 nerve root at this level. Sensation affected is the posterior calf and lateral border of the foot, while motor innervation is to the gastroc soleus complex. With far lateral disk herniations, the exiting nerve root is compressed and symptoms may be seen referred to the level above.Correct Answer: Weakness of the gastrocsoleus complex
2568. (2409) Q6-2868:
Which of the following populations is most at risk for compression fractures of the spine:
1) Middle-aged black women
3) Menopausal females of Inuit (Alaskan native) descent
2) Elderly caucasian men
5) Heavy laborers
4) Elderly fair-skinned women of northern European descent
Osteoporosis is an age-related decrease in bone mass usually associated with a loss of estrogen in postmenopausal women. Sedentary, thin white women of northern European descent are most severely affected. In addition, smoking, heavy drinking, and certain pharmacological agents, such as phenytoin, increase the severity of the disease. Women who breastfed their infants or those with low vitamin D or calcium diets are also at increased risk.Correct Answer: Elderly fair-skinned women of northern European descent
2569. (2410) Q6-2869:
Following an osteoporotic compression fracture, the risk of sustaining another compression fracture at a different level is increased by:
1) 2 times
3) 8 times
2) 5 times
5) 15 times
4) 10 times
Osteoporosis is a systemic disease affecting more than 24 million Americans. Osteoporosis results in progressive bone mineral loss and concurrent changes in bony architecture, which leave the spinal column vulnerable to compression fractures, often after minimal or no trauma. There are an estimated 700,000 osteoporotic vertebral compression fractures (VCFs) in the United States each year, of which more than one third become chronically painful. Approximately 85% of VCFs are due to primary osteoporosis and the remainder due to secondary osteoporosis or malignancies. These VCFs lead to progressive sagittal spine deformity and changes in spinal biomechanics and are believed to contribute to a fivefold increased risk of further fracture by virtue of force transmission to weak vertebrae above or below. Whether the fracture is painful or not, the spinal deformity caused by two or more fractures dramatically impacts health, daily living, and medical costs through loss of lung capacity, loss of appetite, reduced mobility, chronic pain, and/or clinical depression.Correct Answer: 5 times
The following are all purported benefits of percutaneous intraosseous methylmethacrylate injection (vertebroplasty) for the treatment of osteoporotic compression fractures except:
1) Decreased stiffness of the segment, distributing stress at adjacent levels
3) Prevention of further compression fractures by restoring sagittal alignment
2) Rapid pain relief
5) Increased mobility leading to prevention of further osteoporosis
4) Faster return to ambulation
Traditional treatment for patients with osteoporotic vertebral compression fractures (VCFs) includes bed rest, analgesics, and bracing. This type of medical management does not restore spinal alignment, and the lack of mobility increases the rate of demineralization. Because of the inherent risks and invasive nature, surgical treatment of osteoporotic VCFs has been limited to patients with concurrent spinal instability or neurologic compromise. Reconstruction with structural bone graft and instrumentation may be performed from an anterior or posterior approach; however, the success of these techniques is limited by a patientâs poor bone quality and general medical condition.
Augmentation of VCFs with polymethylmethacrylate (PMMA), "vertebroplasty," involves the forced injection of low viscosity PMMA cement into the closed space of the collapsed vertebral body. Although vertebroplasty is currently being used successfully for pain relief in VCFs, this technique does not restore the height of the collapsed vertebral body.Correct Answer: Decreased stiffness of the segment, distributing stress at adjacent levels
2571. (2412) Q6-2871:
Acute thoracic compression fractures should have the following signal characteristics on magnetic resonance imaging:
1) High signal on T1 and T2
3) High signal on T1 and low signal on T2
2) Low signal on T1 and T2
5) Intermediate signal on both T1 and T2
4) Low signal on T1 and high signal on T2
Acute fractures produce local hematomas that displace the adipose tissue normally present in the bone marrow and decrease the signal uptake from fat on T1-weighted sequences. Fractures also produce edema, which is bright on T2-weighted sequences. Acute thoracic compression fractures should be low signal intensity on T1 sequences and high on T2 sequences.Correct Answer: Low signal on T1 and high signal on T2
2572. (2413) Q6-2872:
Slide 1
In this slide of a lumbar burst fracture, which column is disrupted to distinguish it from a compression fracture:
1) Anterior
3) Posterior
2) Lateral
5) Medial
4) Middle
Denis was the first surgeon to include the middle column in his description of thoracolumbar fractures and to accentuate its importance in fracture stability. The defining characteristic of a burst fracture is disruption of the middle column, which distinguishes these fractures from compression fractures. Involvement of the middle column indicates an unstable fracture pattern.Correct Answer: Middle
When evaluating thoracolumbar burst fractures, it is important to remember that the spinal cord ends in the conus medullaris, which typically is present at what level:
1) T10
3) L2
2) L1
5) L4
4) L3
While the conus medullaris can end anywhere from T12 to L3, in the majority of patients it is present at the L1 level. Injury at this level is much different from injury to the spinal cord or the cauda equina.Correct Answer: L1
2574. (2415) Q6-2874:
Slide 1
Canal compromise in burst fractures (Slide) is caused by:
1) Lamina fracture and anterior migration
3) Retropulsion of the posterosuperior vertebral body
2) Migration of the posteroinferior vertebral body
5) Herniated disk material
4) Narrowing of the interpedicular distance
An essential component of burst fractures, as described first by Denis, is the involvement of the middle column. Typically, the posterosuperior vertebral body is separated from the remainder of the body and encroaches into the spinal canal, causing damage to the neural elements. No other part of the middle column is a standard component of the injury.Correct Answer: Retropulsion of the posterosuperior vertebral body
2575. (2416) Q6-2875:
A 16-year-old boy sustains a burst fracture of L2. Radiographs indicate loss of approximately 20% of vertebral height anteriorly and 10° of kyphosis. He is neurologically intact. Management should include:
1) Anterior decompression and strut graft reconstruction
3) Laminectomy, open reduction of the bony intrusion, and fusion
2) Posterior indirect decompression with transpedicular instrumentation and fusion
5) Delayed reconstruction of the spine when the initial injury has been determined
4) An initial period of bed rest, monitoring the patient for ileus, and early mobilization in an extension orthosis or body cast
Burst fractures represent 17% of major spine fractures. Instability and failure occur in the anterior and middle columns. Fifty percent of patients have a neurologic deficit. These fractures are considered unstable if there is more than 50% to 60% anterior compression, 20° to 25° of kyphosis, more than 50% of canal compromise, and posterior injury. Incomplete or progressive neurologic deficits require early decompression and stabilization. Treatment of the stable fracture without neurologic deficit is hyperextension bracing for 3 to 4 months.Correct Answer: An initial period of bed rest, monitoring the patient for ileus, and early mobilization in an extension orthosis or body cast
The sagittal plumb line should fall:
1) Anterior to the C 4 vertebral body
3) Posterior to the anterosuperior corner of S1
2) Through the L3-L4 intervertebral disk
5) Anterior to the S1 body
4) Through the T11 intervertebral disk
Radiographic assessment of the sagittal balance is usually made with a patient standing, with his or her arms flexed forward 90° and supported on a bar. Radiographic assessment is recorded on a 36-inch cassette. Several reference points regarding this technique have appeared in the literature. These points include a plumb line through the center of the C 7 vertebral body, passing 35 mm to 56 mm posterior to the anterosuperior border of S1.Correct Answer: Posterior to the anterosuperior corner of S1
2577. (2418) Q6-2879:
Normal sagittal thoracic alignment is:
1) 5° to 10° of kyphosis due to the adjacent lordotic cervical and lumbar segments
3) Lordotic to support the body weight anteriorly
2) Straight because of the rib cage
5) 60° to 80° of kyphosis between T1 and T10
4) 20° to 50° of kyphosis between T1 and T10
Thoracic kyphosis has contributions from the trapezoidal shapes of the thoracic vertebrae, from the intevertebral disk positions, and from the stiffness of the ribs and sternum. The reported normal values range from 20° to 50°.Correct Answer: 20° to 50Â
° of kyphosis between T1 and T10
2578. (2441) Q6-2902:
A 70-year-old man complains of severe, burning pain in both calves after he ambulates approximately one block. He denies significant back pain. He has long-standing, insulin-dependent diabetes mellitus and a history of coronary artery disease. The patient has smoked two packs of cigarettes each day for more than 30 years. What questions from his history can help differentiate vascular from neurogenic claudication:
1) Distribution of pain
3) Posture changes
2) Pattern of sensory loss
5) Timing of symptom onset
4) Relief of pain with rest
Pain distribution may be similar in vascular and neurogenic claudication. The pattern of patient-reported sensory loss is unlikely to be contributory due to the patientâs history of insulin-dependent diabetes and, presumably, a diabetic peripheral neuropathy. In both syndromes, pain is relieved with rest. Usually, pain relief is quicker in vascular claudication. In neurogenic claudication, standing alone may not relieve patient symptoms; sitting is usually required. Timing of symptom onset is variable in both syndromes. Vascular claudication usually produces less variability in exercise tolerance. Relief of pain with changes in posture (bending over a walker or shopping cart) is found only in neurogenic claudication.
Correct Answer: Posture changes
A 70-year-old man complains of severe, burning pain in both calves after he ambulates approximately one block. He denies significant back pain. He has long-standing, insulin-dependent diabetes mellitus and a history of coronary artery disease. The patient has smoked two packs of cigarettes each day for more than 30 years. The patient is diagnosed with neurogenic claudication. What is the most likely source of his symptoms:
1) Herniated lumbar disk
3) Degenerative spinal stenosis at L3-L4
2) Isthmic spondylolisthesis
5) Metastatic tumor
4) Degenerative spinal stenosis at L4-L5
The most common cause of neurogenic claudication in this patient is degenerative stenosis. L4-L5 is the most commonly affected level. Herniated lumbar disk is less likely. Although a metastatic tumor is possible, especially in light of the patientâs smoking history, the absence of back pain makes this unlikely.
Correct Answer: Degenerative spinal stenosis at L4-L5 2580. (2443) Q6-2905:
Slide 1
A 70-year-old man complains of severe, burning pain in both calves after he ambulates approximately one block. He denies significant back pain. He has long-standing, insulin-dependent diabetes mellitus and a history of coronary artery disease. The patient has smoked two packs of cigarettes each day for more than 30 years. A magnetic resonance image (MRI) of the patient is obtained (Slide). What does the MRI show:
1) Critically severe stenosis at L3-L4 and L4-L5
3) Herniated lumbar disk
2) Moderate lumbar spinal stenosis at L3-L4 and L4-L5
5) Lumbar metastatic disease
4) Moderate lumbar stenosis at L3-L4, L4-L5, and L5-S1
The MRI shows moderately severe lumbar stenosis at L3-L4 and L4-L5. While the degree or severity of stenosis remains subjective, terming this stenosis critical is an exaggeration. The section of the axial images at L5-S1 is not in plane with the disk, hence there appears to be lateral recess stenosis at this level also. The sagittal images, however, do not confirm this diagnosis. There is no evident lumbar disk herniation, and there are no findings indicative of lumbar metastatic disease.
Correct Answer: Moderate lumbar spinal stenosis at L3-L4 and L4-L5
Slide 1
A 70-year-old man complains of severe, burning pain in both calves after he ambulates approximately one block. He denies significant back pain. He has long-standing, insulin-dependent diabetes mellitus and a history of coronary artery disease. The patient has smoked two packs of cigarettes each day for more than 30 years. A magnetic resonance image (MRI) of the patient is obtained, as well as a myelogram (Slide). Conservative options in this patient include:
1) Medication
3) Conditioning exercise
2) Epidural steroids
5) None of the above, proceed to surgery
4) All of the above
The myelogram confirms the diagnosis of lumbar spinal stenosis at L3-L4 and L4-L5. There is no myelographic block, although the stenosis is significant. A trial of conservative therapy is appropriate for this patient.
Correct Answer: All of the above 2582. (2445) Q6-2908:
Slide 1 Slide 2
A 70-year-old man complains of severe, burning pain in both calves after he ambulates approximately one block. He denies significant back pain. He has long-standing, insulin-dependent diabetes mellitus and a history of coronary artery disease. The patient has smoked two packs of cigarettes each day for more than 30 years. Based upon the patientâs history, magnetic resonance image (MRI) (Slide 1), and computed tomography (CT)-myelogram (Slide 2) available for your review, what is the correct diagnosis in this patient:
1) Lumbar metastatic disease
3) Lumbar herniated disk
2) Lumbar spinal stenosis
5) Ankylosing spondylitis
4) Degenerative lumbar spondylolisthesis
There is no evidence of lumbar metastases in this patient. The CT-myelogram shows compression arising posterior to the thecal sac, making a disk herniation less likely. Similarly, there is no evidence of a lumbar disk herniation on MRI. The patientâs sagittal alignment is well maintained, with no spondylolisthesis evident. Ankylosing spondylitis generally presents in younger patients, and the classic radiographic finding of spontaneous arthrodesis is not present.
Correct Answer: Lumbar spinal stenosis
A 75-year-old woman presents with low back pain that is worse with motion and bilateral lower extremity pain that is worse with ambulation. She notes that the pain extends down the posterior aspects of her lower extremities, from the buttocks to the calves. The pain limits her activity â she can only ambulate approximately one block before having to rest. She reports that lumbar flexion, notably leaning over a walker or a shopping cart, considerably diminishes her lower extremity pain. She has a significant past medical history of coronary artery disease, and she has had previous angioplasty of her coronary vessels. On examination, her lower extremity pulses are easily palpable. What would you expect to find on this patientâs neurological examination:
1) No abnormal findings on the neurological examination
3) Severe proximal lower extremity weakness
2) Mild proximal lower extremity weakness
5) Severe distal lower extremity weakness
4) Mild distal lower extremity weakness
Lumbar spinal stenosis is a dynamic process. Patients classically have no deficit until they are physically active. Therefore, this patient may not have a deficit during her clinic examination. It would be unusual for her to present with a fixed lower extremity deficit.
Correct Answer: No abnormal findings on the neurological examination
2584. (2447) Q6-2910:
A 75-year-old woman presents with low back pain that is worse with motion and bilateral lower extremity pain that is worse with ambulation. She notes that the pain extends down the posterior aspects of her lower extremities, from the buttocks to the calves. The pain limits her activity â she can only ambulate approximately one block before having to rest. She reports that lumbar flexion, notably leaning over a walker or a shopping cart, considerably diminishes her lower extremity pain. She has a significant past medical history of coronary artery disease, and she has had previous angioplasty of her coronary vessels. On examination, her lower extremity pulses are easily palpable. What radiographic evaluation would you obtain to best establish her diagnosis:
1) Plain L-spine films
3) MRI of cervical spine
2) Angiogram of lower extremity vessels
5) Magnetic resonance image (MRI) of lumbar spine
4) Computed tomography (CT) of lumbar spine
The most efficacious and least invasive means of evaluation for lumbar spinal stenosis is MRI. A CT scan, while showing bony anatomy well, may not provide adequate information about soft tissue structures. Plain films are nonspecific and, although they are often used as an initial evaluation, may not yield adequate diagnosis. This patientâs symptom complex does not correlate with vascular claudication.
Correct Answer: Magnetic resonance image (MRI) of lumbar spine
2585. (2448) Q6-2913:
A 75-year-old woman presents with low back pain that is worse with motion and bilateral lower extremity pain that is worse with ambulation. She notes that the pain extends down the posterior aspects of her lower extremities, from the buttocks to the calves. The pain limits her activity â she can only ambulate approximately one block before having to rest. She reports that lumbar flexion, notably leaning over a walker or a shopping cart, considerably diminishes her lower extremity pain. She has a significant past medical history of coronary artery disease, and she has had previous angioplasty of her coronary vessels. On examination, her lower extremity pulses are easily palpable. This patient is diagnosed with degenerative spondylolisthesis with significant lateral recess stenosis. Treatment of this patient could include:
1) Epidural steroids
3) Decompression with noninstrumented fusion
2) Decompression alone
5) All of the above
4) Decompression with instrumented fusion
Treatment options for this patient are legion. Considering her significant coronary artery disease, a conservative approach using anti-inflammatory drugs, physical therapy, and epidural steroids might be chosen by some physicians. Surgical options include decompression alone or decompression with fusion. This patientâs significant mechanical low back pain encourages some surgeons to include a fusion with the decompression.
Correct Answer: All of the above
2586. (2493) Q6-2960:
The distinguishing phenotypic feature that differentiates a schwannoma from a neurofibroma is:
1) The consistently hard and irregular surface of a schwannoma tumor
3) The presence of a dissection plane between the tissue and a schwannoma tumor
2) The lack of a capsule around a neurofibroma tumor
5) There is no phenotypic distinguishing characteristic that differentiates a schwannoma from a neurofibroma.
4) Distinctly differing locations of occurrence along the spinal axis
Nerve sheath tumors account for 25% of intradural spinal cord tumors in adults and can be further broken down into either schwannomas or neurofibromas. Schwannomas are more common than neurofibromas, most commonly occurring in patients 30 to 50 years of age, equally between the sexes. Most schwannomas arise in the dorsal nerve root. Neurofibromas, however, have a predilection to the ventral root. Both tumors primarily are intradural but as many as 10% to 15% can escape through the dura to form a dumbbell shape and exist as both an intradural and extradural tumor. Histologically, fibrous tissue and nerve fibers make up a neurofibroma. Grossly, neurofibromas appear as a fusiform enlargement of the nerve, making a clear distinction between tumor and nerve impossible. Macroscopically, schwannomas look like smooth globoid masses sitting on the nerve fiber and a clear resection plane is apparent.Correct Answer: The presence of a dissection plane between the tissue and a schwannoma tumor
2587. (2494) Q6-2962:
What is the advantage of performing a magnetic resonance image (MRI) to evaluate spinal tumors:
1) The post-contrast enhancement pattern is sensitive and specific in diagnosing malignant tumors.
3) Magnetic resonance imaging is more sensitive than computed tomography (CT) myelography.
2) The diagnosis is often made based on imaging location of a tumor, thus avoiding the need for invasive tissue diagnosis.
5) Magnetic resonance imaging is a helpful, adjunct diagnostic tool that can elucidate characteristics of spinal cord lesions and help narrow the differential diagnosis.
4) Magnetic resonance imaging has no advantage as a diagnostic tool in evaluating spinal cord lesions.
Although diagnosis and localization of spinal column tumors depends on a patientâs history and physical examination, differentiation of intramedullary versus extramedullary location of a tumor relies primarily on image findings. The most common imaging modality is MRI. Lesion signal abnormalities, cerebral spinal fluid (CSF) capping, and cord or cauda equina displacement signify extramedullary masses, even without contrast. Gadolinium enhancement increases the sensitivity of the MRI, as almost all spinal cord tumors demonstrate some contrast enhancement. Although more sensitive than MRI, myelography and postmyelography CT are rarely used initially due to their invasive nature.Correct Answer: Magnetic resonance imaging is a helpful, adjunct diagnostic tool that can elucidate characteristics of spinal cord lesions and help narrow the differential diagnosis.
2588. (2495) Q6-2964:
A 54-year-old man presents with low back pain and lower extremity weakness. Imaging shows a solitary lesion located in the conus medullaris with enhancement after administration of gadolinium. The most likely diagnosis is:
1) Epidermoid cyst
3) Meningioma
2) Lymphoma
5) Astrocytoma
4) Ependymoma
Ependymomas are the most common intramedullary tumor in adults and are found with equal prevalence in middle-aged men and women. They are most prevalent in the caudal regions of the spinal cord around the conus medullaris and filum terminale.
Epidermoid cysts and dural-based meningiomas are extramedullary tumors. Astrocytomas are most commonly found in the lower cervical region, and a patient presents with neck pain and upper extremity deficits.Correct Answer: Ependymoma
Which of the following diagnostic modalities is used most often to evaluate suspected malignant astrocytomas of the spinal cord:
1) History and physical examination is often sufficient to make the diagnosis.
3) Malignant astrocytoma of the spinal cord is a clinical diagnosis and is only confirmed after post-mortem tissue evaluation.
2) Imaging characteristics of malignant astrocytomas on magnetic resonance imaging (MRI) are specific enough to make the diagnosis.
5) Computed tomography with a myelogram
4) An open biopsy with tissue evaluation is the only way to make the diagnosis.
Clinically, early symptoms of intramedullary tumors are nonspecific. Almost all intramedullary tumors show contrast uptake. Even though there are specific MRI characteristics to each tumor, enough variability and overlap exists in their radiologic appearance that histological examination is still required for definitive diagnosis.Correct Answer: An open biopsy with tissue evaluation is the only way to make the diagnosis.
2590. (2497) Q6-2966:
The most effective treatment for malignant intramedullary tumors of the spinal cord is:
1) Surgical excision
3) Chemotherapy
2) Radiation therapy
5) Neither a single treatment modality nor a combination of treatment modalities has proven effective in significantly improving mortality.
4) Surgical excision followed by a combination of chemotherapy and radiation therapy.
Despite treatment, a poor prognosis is given to patients with malignant intramedullary tumors. The median survival time for patients with cervical tumors is 3 to 6 months. Surgical excision, radiation, and chemotherapy are not found to significantly improve survival. Treatment is generally supportive.Correct Answer: Neither a single treatment modality nor a combination of treatment modalities has proven effective in significantly improving mortality.
2591. (2498) Q6-2967:
Initially, the most appropriate method to evaluate a patient with suspected peripheral nerve injury involves:
1) An imaging study, preferably magnetic resonance imaging (MRI), of the injured region
3) A doppler ultrasound to study blood flow to the injured area
2) Electromyography and nerve conduction velocity studies
5) A detailed neurologic evaluation noting distal motor function
4) An MRI of the entire spine to evaluate possible spinal cord injury
After a traumatic injury to peripheral nerves, early clinical examination is imperative. The key is to test for motor function in the most distal aspect of the nerve and be able to localize the site of injury. Imaging studies are far less sensitive than clinical examinations. Electromyography and nerve conduction velocity studies are usually performed during the follow-up examination to assess for residual, or recovery of, function.Correct Answer: A detailed neurologic evaluation noting distal motor function
2592. (2499) Q6-2969:
The type of peripheral nerve injury that requires acute repair is:
1) A sharp transection
3) A contusion injury
2) A blunt transection
5) No peripheral nerve injury should be acutely repaired.
4) A stretch injury
The type of traumatic peripheral nerve injury dictates the timing of repair. If a nerve has been sharply transected, then repair should be performed within 72 hours. This can be accomplished during the repair of associated skin, vessels, muscles, and tendons. For a sharply transected but partially cut nerve, early repair is also recommended. Bluntly transected nerves require delayed repair. Contusion and stretch injuries may require delayed surgery.Correct Answer: A sharp transection
The proper treatment plan for contusion and stretch peripheral nerve injuries involves:
1) Immediate surgical exploration and repair
3) Immediate surgical exploration followed by delayed repair weeks to months later
2) Delayed surgical repair 2 weeks after injury
5) Observation followed by delayed surgical exploration if no functional recovery is found
4) Observation only
For contusion and stretch peripheral nerve injuries, delayed surgery is recommended. Patients are followed for several weeks to months and monitored for functional nerve recovery. If there is no evidence of regeneration, then surgical exploration is performed.Correct Answer: Observation followed by delayed surgical exploration if no functional recovery is found
2594. (2501) Q6-2971:
The most common type(s) of peripheral nerve injury is:
1) A sharp laceration injury
3) Contusion and stretch injuries
2) A blunt laceration injury
5) Traumatic peripheral nerve injuries occur with approximately the same frequency.
4) A proximal root avulsion
The most common types of traumatic nerve injuries are contusion and stretch injuries. A severe blow to soft tissues or even a fracture can cause a contusion. Gunshot wounds, for example, may produce contusion injuries. Stretch injuries usually result from extreme movements of the limbs, most commonly the shoulder joint with involvement of the brachial plexus.Correct Answer: Contusion and stretch injuries
2595. (2502) Q6-2972:
Which of the following is the anatomic origin of the Brown-Sequardâs syndrome:
1) Ischemic damage to the periphery of the cord
3) Contusion to the dorsal cord
2) Shear injury to the central cord
5) Complete transection of the cord
4) Traumatic hemisection of the cord
Brown-Sequardâs syndrome often results from penetrating wounds that cause anatomical hemisection of the cord. Shear injury to the central cord usually results in the central cord syndrome. Contusions to the spinal cord lead to level-specific and long-tract findings depending on the location of contusion. Complete transsection leads to complete neurologic loss.Correct Answer: Traumatic hemisection of the cord
2596. (2503) Q6-2973:
Which of the following is the most common cause of and the treatment for conus medullaris syndrome:
1) Traumatic injury treated with steroids
3) Chronic metabolic treated by correcting the underlying cause
2) Ischemic injury treated by medical management
5) Idiopathic, no treatment is needed
4) Compressive lesion treated by surgical decompression
Conus medullaris syndrome is caused by upper and lower motor neuron injury because of a combined spinal cord and nerve root injury caused by thoracolumbar injuries (levels between T-11 and L-1). Causative agents are compressive in nature such as a compression fracture or herniated disk. Treatment is emergent surgical decompression. The prognosis is better for incomplete injuries.Correct Answer: Compressive lesion treated by surgical decompression
Which of the following is the most important prognostic sensory modality during examination of a patient with a spinal cord injury:
1) Pain and temperature sensation carried by the spinothalamic tracts
3) Joint position carried by the spinocerebellar tract
2) Light touch and joint position carried by the dorsal column tracts
5) Sensory examination has no prognostic value in evaluation of spinal cord injury patients
4) All modalities carry the same prognostic value
The most important prognostic sensory modalities are those carried in the lateral spinothalamic tract rather than dorsal columns.Correct Answer: Pain and temperature sensation carried by the spinothalamic tracts
2598. (2505) Q6-2976:
Which of the following is the time window from the time of injury during which treatment of nonpenetrating spinal cord injury with methylprednisolone is indicated:
1) 2 hours
3) 8 hours
2) 4 hours
5) 24 hours
4) 12 hours
Administration of methylprednisolone within 8 hours of injury provides benefit to patients with spinal cord injury. Treatment of patients arriving after 8 hours of treatment has been shown to worsen morbidity. Therefore, patients arriving at trauma centers within this time receive methylprednisolone treatment as part of the standard of care. The exception is the group of patients with penetrating spinal cord injuries where the risk of treatment outweighs the potential benefits.Correct Answer: 8 hours
2599. (2596) Q6-3069:
Pain from a herniated lumbar disk is caused by:
1) Rupture of the thecal sac
3) Ischemia and potential necrosis of the nerve root
2) Local instability due to a ruptured nucleus pulposus
5) Herniated nucleus pulposus and the resulting local inflammation
4) Associated spinal stenosis
The annulus is composed of alternating laminae that are primarily composed of type I and type II collagen. The annulus is thinnest posterolaterally and thickest anteriorly. As the disk is loaded, the nucleus transfers axial loads to the annulus in the form of hoop stresses. With degenerative or traumatic processes, fissures or tears may develop in the annulus and the nucleus can become herniated.
A herniated nucleus pulposus is a foreign material to the surrounding structures. The combination of mechanical pressure on a nerve root and local inflammation can lead to neurologic signs and symptoms.
Correct Answer: Herniated nucleus pulposus and the resulting local inflammation
2600. (2597) Q6-3070:
The term Schmorlâs nodule refers to:
1) A giant synovial cyst
3) A pathologic dorsal root ganglion
2) An enlarged paravertebral lymph node
5) Anterior displacement of an intervertebral disk
4) Superior or inferior displacement of an intervertebral disk
Superior and inferior displacements into the vertebral body are known as Schmorlâs nodules.Correct Answer: Superior or inferior displacement of an intervertebral disk
A far lateral herniated nucleus pulposus at the L4-L5 level would lead to signs and symptoms of which nerve root on the affected side:
1) L3
3) L5
2) L4
5) None of the above
4) S1
A far lateral herniated nucleus pulposus, which is less common, can lead to compression of the nerve root that has already exited the supra-adjacent foramen. Therefore, a far lateral L4-L5 herniated nucleus pulposus leads to L4 nerve root compression.Correct Answer: L4
2602. (2599) Q6-3073:
A sequestered disk herniation refers to:
1) Bulging of the nucleus through a weakened annulus
3) Rupture of the nucleus through the annulus and the posterior longitudinal ligament
2) Rupture of the nucleus through the annulus
5) Separation of a herniated fragment from the disk
4) Rupture of the nucleus through the posterior longitudinal ligament
A sequestered herniation is a separation of a herniated fragment from the disk from which it came.Correct Answer: Separation of a herniated fragment from the disk
2603. (2600) Q6-3074:
Common presentations of cauda equina syndrome include:
1) Severe low back pain with nausea and vomiting
3) Progressive chronic low back pain radiating to the gluteal region
2) Acute onset unilateral foot drop
5) Fever, photophobia, and nuchal rigidity
4) Saddle anesthesia and bowel and/or bladder dysfunction
In patients with suspected central herniated nucleus pulposus, cauda equina syndrome must not be missed as it could cause irreversible neurological damage. Cauda equina syndrome presents with saddle anesthesia and bowel or bladder changes.Correct Answer: Saddle anesthesia and bowel and/or bladder dysfunction
2604. (2601) Q6-3075:
Which of the following statements is true regarding the natural history of a herniated lumbar disk:
1) The natural history of a herniated lumbar disk is usually consistent with approximately 90% spontaneous resolution without intervention by 3 monthsâ follow-up.
3) Despite aggressive surgical correction, permanent neurological deficits are common.
2) Surgical intervention is often required for definitive and long-term treatment.
5) The natural history of lumbar disk herniations has not been studied.
4) Surgical diskectomy is a contraindication in patients with neurologic deficit.
A period of rest is prescribed for 1 to 2 days with supports under the knees and neck to minimize root tension. Also, nonsteroidal anti-inflammatory drugs are used. Prolonged bed rest is no longer advocated because it can lead to deconditioning of compensatory musculature. Ambulation is begun as tolerated after the first few days of an acute event. More than one-half of patients who initially present with low back pain recover within 1 week and more than 90% of patients recover in 1 to 3 months. Physical therapy is started as tolerated.Correct Answer: The natural history of a herniated lumbar disk is usually consistent with approximately 90% spontaneous resolution without intervention by 3 monthsâ follow-up.
Long-term follow-up studies of surgical versus conservative treatment of herniated lumbar disks indicate:
1) Conservative management yields better long-term outcome.
3) No statistically significant difference in outcome is noted despite the type of treatment used.
2) Surgical intervention has better long-term results.
5) None of the above
4) No long-term data are available.
The prognosis of herniated lumbar disks is generally good regardless of treatment. Patients operated on for proven disk herniations improved more rapidly than patients treated nonoperatively. However, within 4 to 5 years, the outcomes begin to approximate each other.Correct Answer: No statistically significant difference in outcome is noted despite the type of treatment used.
2606. (2603) Q6-3078:
The predominant cause of low back pain in the general population, aside from the general sprain and strains of the paraspinal structures, is attributed to:
1) Spondylolisthesis
3) Spinal stenosis
2) Herniated nucleus pulposus
5) Vascular insufficiency
4) Degenerative disk disease
The consequences of normal aging of the spine include progressive disk dehydration, chemical alterations and subsequent mechanical âincompetenceâ of the intervertebral disk, which may be manifested in low back pain, although an exact correlation between disk degeneration and low back pain has not been established. Nevertheless, many believe that the predominant cause of persistent low back pain is degeneration of the disk.Correct Answer: Degenerative disk disease
2607. (2604) Q6-3079:
Initial work-up of an otherwise healthy individual with acute onset low back pain should include:
1) A complete history, physical examination, and follow-up imaging studies only if indicated
3) A magnetic resonance imaging study of the lumbar spine
2) A complete history, physical examination, and plain radiographs
5) No evaluation is needed on initial visit as most low back pain resolves spontaneously
4) A computed tomography of the lumbar spine
All patients presenting with back pain should have a thorough history taken and a complete physical exam including a detailed neurologic exam. In the recently published Agency for Health Care Policy and Research Clinical Practice Guideline on Acute Low Back Pain Problems in Adults, it was concluded that a focused physical exam was sufficient to assess a patient with acute or recurrent low back pain of fewer than 4 weeks duration, unless findings suggested an underlying tumor, or an infectious, a traumatic or a major neurologic syndrome.Correct Answer: A complete history, physical examination, and follow-up imaging studies only if indicated
2608. (2605) Q6-3080:
A 72-year-old man with acute onset low back pain with increased severity during the night should be evaluated by:
1) History and physical examination only
3) History, examination, and urine protein electrophoresis
2) Magnetic resonance imaging of the lumbar spine
5) Plain radiographs of the lumbar spine
4) Computed tomography of the lumbar spine
An elderly patient with unsolicited low back pain is suggestive of a primary malignancy or metastatic disease of the lumbar spine. A thorough history and physical examination are indicated, as well as imaging to evaluate the lumbar spinal axis and the neural elements.Correct Answer: Magnetic resonance imaging of the lumbar spine
An otherwise healthy 56-year-old patient with suspected spinal stenosis after history and physical examination undergoes plain radiography that is unremarkable for spondylolisthesis. The next feasible imaging modality that is indicated in aiding the diagnosis is:
1) No more imaging studies are needed
3) Magnetic resonance imaging
2) A computed tomography myelogram
5) Spinal angiogram
4) Ultrasound
Although a computed tomography myelogram is slightly more specific and sensitive than magnetic resonance imaging (MRI) in evaluating lumbar stenosis, MRI is almost as sensitive and it is noninvasive. Therefore, in an otherwise healthy patient without contraindications, an MRI should be considered as the next imaging modality.Correct Answer: Magnetic resonance imaging
2610. (2607) Q6-3083:
Untreated low back pain most commonly:
1) Improves spontaneously
3) Leads to neurological decompensation
2) Undergoes progressive worsening prompting further medical care
5) Progresses to chronic failed back syndrome
4) No studies have been done to evaluate the natural history of untreated low back pain.
Generally, patients diagnosed with low back pain should undergo 4 weeks of conservative treatment with an accepted prognosis of predominantly spontaneous improvement over a 4-week period, regardless of treatment.Correct Answer: Improves spontaneously
2611. (2608) Q6-3084:
Which of the following is NOT a routinely used imaging modality for evaluation of spinal pathology:
1) Plain radiography
3) Magnetic resonance imaging
2) Computed tomography
5) Myelogram
4) Diskography
Diskography is a diagnostic technique that has been used since the 1950âs. The study involves injection of dye into an intervertebral disk space. A positive study is one in which the injected dye is not contained within the disk space or in which the injection reproduces the characteristic distribution of the patientâs pain. The current role of diskography remains undefined and, at this time, diskography is not a first-line diagnostic study in the evaluation of patients with low back pain.Correct Answer: Diskography
2612. (2609) Q6-3085:
Which of the following is the most common type of spondylolisthesis seen in the adult population:
1) Degenerative
3) Congenital
2) Isthmic
5) Pathologic
4) Traumatic
The prevalence of degenerative spondylolisthesis is 2% to 5%; the prevalence increases with age. Symptomatic patients usually present in the fourth decade of life or later. The disease is five times more common in the female sex. The African American population, diabetics, and patients with sacralization of the L5 vertebrae are also at increased risk for developing symptomatic spondylolisthesis.Correct Answer: Degenerative
Which of the following is the most common location of adult degenerative spondylolisthesis:
1) L1-L2 interspace
3) L3-L4 interspace
2) L2-L3 interspace
5) L5-S1 interspace
4) L4-L5 interspace
The L4-L5 interspace is the most common location of adult degenerative spondylolisthesis.Correct Answer: L4-L5 interspace
2614. (2611) Q6-3088:
Which of the following statements is true regarding the initial diagnostic radiographic evaluation of patients with spondylolisthesis:
1) Initial diagnostic radiographic evaluation includes plain radiographic imaging of lumbar spine with flexion and extension.
3) Spondylolisthesis is a clinical diagnosis; no imaging studies are indicated.
2) Computed tomography of the lumbar spine region is the first order diagnostic imaging study.
5) If spondylolisthesis is suspected, myelogram is necessary for diagnosis.
4) Initial diagnostic radiographic evaluation includes magnetic resonance imaging of the lumbar spine to evaluate spinal stenosis and nerve root compression.
Plain radiographs should be performed in a standing position as some cases of spondylolisthesis can be missed if x-rays are taken in a supine position. Forward displacement of L4 on L5 and more rarely L5 on S1 or L3 on L4, without a pars interarticularis defect is often demonstrated. Other radiologic findings of osteophyte formation, such as disk-space narrowing, endplate sclerosis, vacuum disk sign, facet sclerosis and hypertrophy, are consistent with long-standing degenerative disease. Hemisacralization of L5 may be revealed. Flexion, extension, and lateral bending films often reveal hypermobility.Correct Answer: Initial diagnostic radiographic evaluation includes plain radiographic imaging of lumbar spine with flexion and extension.
2615. (2612) Q6-3089:
Initial nonoperative management of adult degenerative spondylolisthesis includes all of the following except:
1) Physical therapy
3) Modified activity
2) Anti-inflammatory medication
5) Support brace
4) Strict bed rest
Conservative treatment for degenerative spondylolisthesis is consistent with the conservative care of most degenerative spinal disorders. It includes modified activity, physical therapy (conditioning exercises emphasizing lumbar flexion and progression to aerobic conditioning), anti-inflammatory medication, and sometimes spinal support with a corset or light-weight brace.Correct Answer: Strict bed rest
2616. (2613) Q6-3090:
Which of the following statements is true regarding lumbar degenerative scoliosis:
1) Lumbar degenerative scoliosis is most commonly distributed to the left.
3) Lumbar degenerative scoliosis is most commonly evenly distributed between left and right.
2) Lumbar degenerative scoliosis is most commonly distributed to the right.
5) No data are available.
4) The distribution of lumbar degenerative scoliosis depends on age of patient at the time of onset.
Degenerative lumbar scoliosis occurs in approximately the same number of women as men. Lumbar curves are generally smaller than those in idiopathic scoliosis and are more evenly distributed between left and right, also in contrast to idiopathic curves that occur predominantly to the left.Correct Answer: Lumbar degenerative scoliosis is most commonly evenly distributed between left and right.
Which of the following is the most common complaint in patients with degenerative lumbar scoliosis:
1) Radicular pain radiating to one or the other leg
3) Low back pain and reduced tolerance for walking
2) Mechanical pain during motion
5) Sensory changes at the dorsal feet bilaterally
4) Urinary incontinence
Patients with degenerative lumbar scoliosis typically complain of symptoms related to the associated spinal stenosis. These symptoms commonly include (with approximate incidence rates): low back pain (100%), reduced tolerance for standing and walking (85% to 100%), neurogenic claudication (50%), and radicular or pseudoradicular pain radiating into the buttocks or thighs (40% to 60%).Correct Answer: Low back pain and reduced tolerance for walking
2618. (2615) Q6-3095:
Bony contribution to the lumbar lordotic curvature is provided by:
1) Spinous processes
3) Lamina
2) Articulating facets
5) Vertebral body
4) Pars interarticularis
The anterior portion of each body has a slightly increased height that contributes to the sagittal lumbar lordosis. The posterior vertebral arch consists of the paired pedicles, laminae, and a midline dorsal spinal process.Correct Answer: Vertebral body
2619. (2616) Q6-3096:
Superior articulating facets in the lumbosacral spine differ from those in the thoracic spine because facets in the lumbosacral spine:
1) Face posteriorly
3) Have a thicker facet joint capsule
2) Face dorsomedially
5) Are fused and are not true joints
4) Face superolaterally
The paired superior articular facets are directed dorsomedially with their corresponding inferior articular processes directed ventrolaterally. These diarthrodial articulations possess thin, lax joint capsules capable of a limited gliding articulation between adjoining vertebrae. They permit flexion, lateral bending and extension, but resist rotation due to both size and facet orientation. The facets alone can bear up to 18% of the compressive load.Correct Answer: Face dorsomedially
2620. (2617) Q6-3098:
Limitation of hyperextension in the lumbosacral spine is controlled by the:
1) Posterior longitudinal ligament
3) Ligamentum flavum
2) Anterior longitudinal ligament
5) Intertransverse ligament
4) Interspinous ligament
More flexion-extension motion occurs in the caudal segments of the lumbar spine than in the upper and middle levels. The well-developed anterior longitudinal ligament and the anterior portion of the annulus fibrosus are important inhibitors of hyperextension.Correct Answer: Anterior longitudinal ligament
Which of the following is the primitive remnant of the nucleus pulposus:
1) The smoites
3) Midgut
2) Ectoderm
5) Notocord
4) Rhombencephalon
The nucleus pulposus is derived from the primitive notochord. It consists predominantly of hydrated proteoglycans with a minor component of a random network of type I and type II collagen.Correct Answer: Notocord
2622. (2619) Q6-3102:
Which of the following is the most common region of the spine affected by metastatic disease:
1) Craniocervical junction
3) Thoracic region
2) Cervical region
5) Sacral region
4) Lumbar region
The thoracic spine is the most common site of metastatic disease. This has been attributed to the watershed zone being located in the low thoracic region.Correct Answer: Thoracic region
2623. (2620) Q6-3103:
Which of the following is the most common complaint at time of presentation in patients with metastatic spine disease:
1) Difficulty with balance
3) Night sweats
2) Difficulty with urination
5) Numbness in the lower extremities
4) Pain during the night
The most common manifestation of metastatic disease is persistent pain. Pain is most marked at night and aggravated by movement. History of trauma is usually absent. Pain is followed by weakness of the lower extremities, sensory loss, and bladder and bowel changes.Correct Answer: Pain during the night
2624. (2621) Q6-3105:
Which of the following methods is the standard in diagnosing vertebral metastatic disease:
1) Plain radiography demonstrating lytic lesion
3) Magnetic resonance imaging with typical destructive lesion characteristics
2) Computed tomography with bony destruction respecting adjacent vertebral levels
5) There is no standard of care in diagnosing vertebral metastasis.
4) Tissue biopsy
The only definitive method of determining the presence and nature of metastatic tumor is vertebral biopsy. Computed tomography-guided biopsy of the spine provides an accurate access to the lesion. Open biopsy is indicated when image guided biopsy is not feasible or non-diagnostic. Differential diagnosis mainly involves spinal infections, osteoporosis, disk disease, and multiple myeloma.Correct Answer: Tissue biopsy
Which of the following is NOT an indication for surgical intervention in metastatic vertebral disease:
1) Progressive neurologic deficit
3) Instability of the spine
2) Poor prognosis
5) Failure of radiation therapy
4) Uncontrollable pain
In patients with metastatic vertebral disease, indications for surgery include progressive neurologic deficit, instability of the spine, uncontrollable pain, and failure of radiation therapy. Surgical intervention can add significant morbidity while providing marginal improvement in longevity of a patient with an already poor prognosis.Correct Answer: Poor prognosis
2626. (2623) Q6-3107:
Which of the following is the most common cause of lumbar stenosis:
1) Congenital
3) Degenerative
2) Posttraumatic
5) Idiopathic
4) Iatrogenic
Degenerative lumbar stenosis is the most common cause of lumbar stenosis. With normal aging of the disk, the water-binding capacity of the nucleus pulposus is dissipated, diminishing its ability to withstand normal compressive and rotational forces. With progressive degeneration of the disk, collapse occurs. This collapse results in overriding of the facet joints and relative lengthening of adjacent capsular and ligamentous structures.
Continued instability, which may be multidirectional, results in hypertrophic changes about the periphery of the vertebral body at its annular attachments. Radiographically, these are seen as traction osteophytes. Similarly, osteophytes form about the facet joints, which lead to compromise of the neural canal. With disease progression, hypertrophic changes predominate, leading to ankylosis and auto stabilization. In patients with less than optimal canal configurations or dimensions or those with excessive hypertrophic degenerative changes, narrowing of the spinal canal, lateral recesses, and neural foramina may result in neurogenic signs and symptoms.
Correct Answer: Degenerative
2627. (2624) Q6-3108:
Which of the following is the most common presentation of a patient with lumbar stenosis:
1) Foot drop
3) Chronic low back pain with neurogenic claudication
2) Acute onset of pain in bilateral lower extremities
5) Intermittent urinary incontinence
4) Saddle anesthesia
The most common complaint in patients with spinal stenosis is chronic low back pain with worsening and lower extremity weakness after ambulation (claudication). Symptoms are often resolved by rest and/or leaning forward.Correct Answer: Chronic low back pain with neurogenic claudication
2628. (2625) Q6-3110:
Clinical symptoms of lumbar spinal stenosis usually correlate with a canal anteroposterior diameter of less than:
1) 20 mm
3) 10 mm
2) 15 mm
5) There is no correlation.
4) 5 mm
The clinical syndrome of lumbar stenosis correlates with a measured anteroposterior diameter of the dural sac of less than 10 mm.Correct Answer: 10 mm
Which of the following is the most commonly fractured location along the thoracolumbar axis:
1) The cervicothoracic junction
3) The thoracolumbar region
2) The mid-thoracic region
5) The lumbosacral junction
4) The lumbar region
Up to 60% of spinal injuries occur between the T11 and L1 segments. The rigid thoracic rib cage and coronal orientation of the facets permit lateral bending and rotation but little flexion and extension. The facet joints then transition caudally to a more sagittal orientation in the lumbar spine, allowing increased flexion/extension but limiting lateral motion. These factors create a stress concentration at the thoracolumbar junction, which is demonstrated by the high incidence of injury at the T11 to L1 segments.Correct Answer: The thoracolumbar region
2630. (2627) Q6-3113:
A 26-year-old man who was involved in a motor vehicle accident is found to have a T12 compression fracture on plain radiography without evidence of posterior extrusion. The likelihood of finding another fracture in the spinal axis with further evaluation is:
1) Highly remote, these fractures are usually isolated
3) 50% to 75%
2) 10% to 15%
5) The incidence of noncontiguous-associated fractures in the spine is not known.
4) A low thoracic fracture is almost always associated with another fracture in the spine.
A thorough workup in these patients is essential; approximately 10% to 15% of patients will have noncontiguous injuries located elsewhere in the spine.Correct Answer: 10% to 15%
2631. (2628) Q6-3114:
Based on the three-column model of spinal stability, an unstable spinal injury is defined as:
1) An injury that disrupts no less than all three columns
3) Disruption of more than one column
2) Disruption of any of the three columns is considered unstable.
5) The three-column model of injury is not a reliable marker of instability.
4) Disruption of all three columns plus neurological injury
The three-column spine consists of the anterior, middle, and posterior columns. In this widely used classification system, the middle column is the key to instability. If the middle column is disrupted, in addition to either the anterior or posterior columns, then instability results.Correct Answer: Disruption of more than one column
2632. (2629) Q6-3115:
Burst fractures of the vertebral body require prompt evaluation because:
1) Although burst fractures of the vertebral body are stable injuries, neurologic deterioration is likely.
3) Burst fractures of the vertebral body are extremely painful to the patient.
2) Burst fractures of the vertebral body involve two-column injury and are unstable.
5) Burst fractures of the vertebral body often result in spinal shock.
4) Burst fractures of the vertebral body are commonly associated with other noncontiguous fractures.
When the middle column is involved in a compression injury, it is classified as a burst fracture. This involves axial load on the spine, with or without a flexion component, and retropulsion of the posterosuperior vertebral body into the spinal canal, thus requiring prompt medical attention. Neurologic deficit is variable and is related to the severity of the initial injury and location of the fracture, and only loosely related to the percent of canal compromise.Correct Answer: Burst fractures of the vertebral body involve two-column injury and are unstable.
Compression fractures of the spine, although typically considered a one-column injury, can be unstable. Findings at time of presentation suggestive of an unstable fracture include:
1) Pain out of proportion to the physical examination
3) Initial kyphosis greater than 20° to 30°
2) Radiographic findings of more than one compression fracture
4) Loss of less than 50% of anterior vertebral body height
Compression fractures are inherently stable and may be treated with extension bracing or casting. If, however, the flexion injury is severe enough, damage to the posterior ligaments can result and the injury becomes unstable. Criteria for this instability were developed by McAfee and include more than 20° to 30° of initial kyphosis or more than 50% loss of anterior vertebral height, applicable to both compression and burst fractures.Correct Answer: Initial kyphosis greater than 20° to 30°
2634. (2631) Q6-3117:
Which of the following is the most important factor responsible for a decreasing proportion of patients with complete paraplegia after sustaining a spinal cord injury today compared with four decades ago:
1) Improvements in rehabilitative measures
3) New and novel medication therapy
2) Advances in operative techniques and instrumentation
5) Better initial triage, resuscitation, and clinical management of patients
-
Higher patient motivation and participation in therapy and rehabilitation
Improvements in the initial triage, resuscitation, and clinical management of spinal cordâ injured patients are likely responsible for a decreasing proportion of patients with complete paraplegia. Currently, approximately 45% of spinal cordâinjured patients have a complete injury, as opposed to two-thirds four decades ago.Correct Answer: Better initial triage, resuscitation, and clinical management of patients
2635. (2632) Q6-3118:
Based on the current consensus on treatment of acute spinal cord injury, intravenous steroid treatment is considered to have potential benefit if begun within how many hours of original injury:
1) 3
3) 8
2) 5
-
24
4) 12
The results of the National Acute Spinal Cord Injury Study II (NASCIS II) demonstrated significant motor and sensory improvement in patients who were treated within 8 hours of injury with a methylprednisolone bolus of 30 mg/kg, followed by an infusion of 5.4 mg/kg per hour for 24 hours.Correct Answer: 8
2636. (2633) Q6-3119:
A 73-year-old woman with a history of cervical stenosis who sustained a fall at home yesterday is now complaining of âclumsyâ fingers and weakness in her hands. She denies any difficulty with ambulation or bowel and bladder dysfunction. She most likely has:
-
Bilateral cervical radiculopathy
3) Anterior cord syndrome
2) Exacerbation of cervical stenosis
5) Central cord syndrome
4) Posterior cord syndrome
The most common incomplete spinal cord injury syndrome is most likely central cord syndrome. Central cord syndrome often occurs as a result of a pinching of the spinal cord in elderly patients who have a narrowed spinal canal as the result of degenerative spondylosis. It is a pattern of disproportionately severe upper extremity motor and sensory changes as compared to lower extremity findings.Correct Answer: Central cord syndrome
A 27-year-old man was involved in a motor vehicle accident. He was resuscitated at the scene but was noted to have a prolonged hypotensive period. Upon arrival at the medical center, he is noted to be paraplegic but radiographic evaluation does not demonstrate any fracture or soft tissue abnormality. Which of the following is the most likely diagnosis:
1) Occult fracture with retropulsion into the cord
3) Spinal shock
2) Contusion of the cord at a high thoracic level
5) Conversion disorder
4) Spinal cord ischemic injury at the low thoracic watershed zone
A watershed zone refers to an area that is supplied purely by end arteries. Therefore, during periods of hypoperfusion, it is the most likely region to sustain an ischemic injury. In the spinal cord, this region lies in the T7-T9 region as it is a watershed zone between the rostral anterior spinal artery distribution and the caudal dominant lumbar segmental artery.Correct Answer: Spinal cord ischemic injury at the low thoracic watershed zone
2638. (2635) Q6-3121:
The watershed zone of the spinal cord most closely correlates with which region of the spinal cord:
1) C 5-C 7
3) T7-T9
2) T4-T6
5) L3-L5
4) T11-L1
A watershed zone refers to an area that is supplied purely by end arteries. Therefore, during periods of hypoperfusion, it is the most likely region to sustain an ischemic injury. In the spinal cord, this region lies in the T7-T9 region as it is a watershed zone between the rostral anterior spinal artery distribution and the caudal dominant lumbar segmental artery.Correct Answer: T7-T9
2639. (2659) Q6-3152:
Which of the following is the most common source of infection in vertebral osteomyelitis:
1) Trauma
3) Hematogenous spread
2) Iatrogenic
5) Unknown mechanism
4) Spontaneous
Hematogenous seeding from another primary source is the most common causative agent. Hematogenous spread of infections is believed to affect the spine via septic emboli in the endarteriolar circulation of segmental spinal arteries at the vertebral endplates. The majority of cases of pyogenic spondylitis begin in the subchondral, metaphyseal region of the anterior subligamentous portion of the vertebral body â the portion with the greatest arterial supply and the most anastomoses.Correct Answer: Hematogenous spread
2640. (2660) Q6-3153:
Which of the following is the most common location of vertebral osteomyelitis along the spinal axis:
1) Craniocervical junction
3) Lumbar spine
2) Thoracic spine
5) Cervical spine
4) Sacral spine
Lumbar spine is the most common region of the spine affected by hematogenous spread of organisms leading to osteomyelitis followed by the thoracic spine.Correct Answer: Lumbar spine
Which of the following is the most common organism identified in cases of vertebral osteomyelitis:
1) Staphylococcus aureus
3) Haemophilus influenzae
2) Streptococcus pneumoniae
5) Anaerobic gram-negative rods
4) Escherichia coli
Staphylococcus aureus remains the most common causative organism, but an increasing proportion of cases are due to gram-negative and anaerobic organisms such as Proteus, Escherichia coli and Pseudomonas.Correct Answer: Staphylococcus aureus
2642. (2662) Q6-3155:
Which of the following is the most common presentation of vertebral osteomyelitis:
1) Fever of unknown origin
3) Unrelenting back pain not relieved by rest
2) Lower extremity pain and weakness
5) None of the above. It is usually an incidental finding during an unrelated work-up.
4) Urinary incontinence
The most common presenting sign of vertebral osteomyelitis is back pain and malaise, often of 3 monthsâ duration or greater. It is often well localized to the affected level and the nature is not unlike most degenerative spinal conditions. A high index of suspicion is essential to make a timely diagnosis. Back pain that awakens a patient at night is a hallmark of infection or tumor.
Pain associated with infection tends to be relentless and not related to activity level. Most patients have percussion tenderness over the involved segments. Fevers are noted in fewer than half of patients.Correct Answer: Unrelenting back pain not relieved by rest
2643. (2663) Q6-3156:
Which of the following is the hallmark distinguishing feature of vertebral osteomyelitis when compared to a neoplastic process on imaging:
1) Uniform enhancement after administration of gadolinium in the neoplasia
3) Lack of endplate involvement in the neoplastic process
2) Destruction of disk space and encroachment of adjacent vertebral body in vertebral osteomyelitis
5) There are no distinguishing radiographic features between vertebral osteomyelitis and a tumor.
4) Evidence of a compression fracture in vertebral osteomyelitis
The crossing of the infectious process along the disk space to involve adjacent vertebrae is a hallmark feature of osteomyelitis used to differentiate it from a neoplastic process.Correct Answer: Destruction of disk space and encroachment of adjacent vertebral body in vertebral osteomyelitis
2644. (2664) Q6-3157:
Which of the following is the imaging modality of choice with the highest relative sensitivity and specificity in patients with suspected vertebral osteomyelitis:
1) Plain radiography
3) Magnetic resonance imaging with contrast administration
2) Computed tomography with contrast administration
5) Vertebral osteomyelitis is primarily a clinical diagnosis
4) Post myelogram computed tomography
Magnetic resonance imaging (MRI) is the modality of choice for spinal infections. An MRI study provides excellent visualization of the neural elements and can determine whether the inflammatory process extends beyond the margins of disk and bone. MRI also provides excellent regional anatomic information. Scans performed with and without intravenous gadolinium are diagnostic in 90% to 95% of cases.Correct Answer: Magnetic resonance imaging with contrast administration
Neurogenic shock is defined as:
1) Decreased cardiac output due to increased parasympathetic tone
3) Widespread gram-negative septicemia with hypoperfusion
2) Severe volume depletion leading to hypotension
5) Increased cardiac output due to decreased parasympathetic tone
4) Loss of sympathetic tone and widespread vasodilation
Neurogenic shock is a unique hemodynamic alteration in patients with spinal cord injuries who have their sympathetic outflow disrupted in addition to the interruption of the motor and sensory pathways. The loss of sympathetic tone to the heart and peripheral vasculature leads to bradycardia and hypotension.Correct Answer: Loss of sympathetic tone and widespread vasodilation
2646. (2694) Q6-3188:
The normal range of thoracic kyphosis is:
1) 0° to 10°
3) 20° to 50°
2) 5° to 20°
5) 40° to 60°
4) 35° to 50°
The normal range of thoracic kyphosis is 20° to 50°. The mean in normal adults is 35°. The normal range of lumbar lordosis is 40° to 80°. The mean in normal adults is approximately 60°.
The spine is usually straight in the sagittal plane between T10 and L2. The majority of lumbar lordosis occurs between L4 and S1. Correct Answer: 20° to 50°
2647. (2695) Q6-3189:
The normal range of lumbar lordosis is:
1) 0° to 10°
3) 20° to 50°
2) 10° to 20°
5) 60° to 90°
4) 40° to 80°
The normal range of thoracic kyphosis is 20° to 50°. The mean in normal adults is 35°. The normal range of lumbar lordosis is 40° to 80°. The mean in normal adults is approximately 60°.
The spine is usually straight in the sagittal plane between T10 and L2. The majority of lumbar lordosis occurs between L4 and S1. Correct Answer: 40° to 80°
2648. (2696) Q6-3190:
Which of the following regions of the spine is normally straight:
1) T1 to T6
3) T10 to L2
2) T7 to T12
5) T12 to S1
4) L1 to L4
The normal range of thoracic kyphosis is 20° to 50°. The mean in normal adults is 35°. The normal range of lumbar lordosis is 40° to 80°. The mean in normal adults is approximately 60°.
The spine is usually straight in the sagittal plane between T10 and L2. The majority of lumbar lordosis occurs between L4 and S1. Correct Answer: T10 to L2
2649. (2697) Q6-3191:
The endplates and pedicles of which of the following vertebra are normally parallel to the ground in a standing individual:
1) L1
3) S1
2) L3
5) T12
4) T1
The alignment of the spine is important in normal upright posture. There is a normal degree of lordosis in the cervical and lumbar spines and a moderate degree of kyphosis in the thoracic spine. The head, spine, and pelvis are connected and balanced. If the spine is out of balance, then a deformity can develop causing fatigue of the paraspinal muscles.
The normal sagittal alignment in the upright patient is as follows: Plumb line
The sagittal plumb line falls from the odontoid process through the C 7-T1 intervertebral disk and then anterior to the thoracic spine. The plumb line then crosses the spine at the T12-L1 intervertebral disk, and then travels posterior to the spine. The plumb line crosses at the posterior corner of the S1 vertebra.
The endplates and pedicles of the L3 vertebra are normally parallel to the ground. Correct Answer: L3
2650. (2698) Q6-3192:
Which of the following is true regarding the alignment of the spine with aging:
1) Thoracic kyphosis decreases; lumbar lordosis increases
3) Thoracic kyphosis increases; lumbar lordosis decreases
2) Thoracic kyphosis decreases; lumbar lordosis decreases
5) The alignment of the spine undergoes no significant changes with aging.
4) Thoracic kyphosis increases; lumbar lordosis increases
The normal range of thoracic kyphosis is 20° to 50°. The mean in normal adults is 35°. The normal range of lumbar lordosis is 40° to 80°. The mean in normal adults is approximately 60°.
The spine is usually straight in the sagittal plane between T10 and L2. The majority of lumbar lordosis occurs between L4 and S1.
With aging, due to changes in the intervertebral disks, thoracic kyphosis increases and lumbar lordosis increases. There is loss of height of the intervertebral disks.
Correct Answer: Thoracic kyphosis increases; lumbar lordosis increases
In reference to the normal sagittal vertical axis (sagittal plumb line), the axis normally falls from the odontoid process through the C 7-T1 intervertebral disk and anterior to the thoracic vertebra. This normal axis crosses the spinal column at which of the following levels before crossing the spinal column at the posterior superior border of the S1 vertebral body:
1) T3-T4 intervertebral disk
3) T8-T10 intervertebral disk
2) T6-T7 intervertebral disk
5) L3-L4 intervertebral disk
4) T12-L1 intervertebral disk
The alignment of the spine is important in normal upright posture. There is a normal degree of lordosis in the cervical and lumbar spines and a moderate degree of kyphosis in the thoracic spine. The head, spine, and pelvis are connected and balanced. If the spine is out of balance, then a deformity can develop causing fatigue of the paraspinal muscles.
The normal sagittal alignment in an upright patient is as follows: Plumb line
The sagittal plumb line falls from the odontoid process through the C 7-T1 intervertebral disk and then anterior to the thoracic spine. The plumb line then crosses the spine at the T12-L1 intervertebral disk, and then travels posterior to the spine. The plumb line crosses at the posterior corner of the S1 vertebra
The endplates and pedicles of the L3 vertebra are normally parallel to the ground. Correct Answer: T12-L1 intervertebral disk
2652. (2700) Q6-3194:
The vertebral artery on the right side of the body arises from the subclavian artery and enters the lateral mass foramen of which of the following cervical vertebra (the first one it enters) before ascending to the brain:
1) C 3
3) C 5
2) C 4
5) C 7
4) C 6
The vertebral artery arises from the subclavian artery on the right side of the body and the aortic arch on the left side. The vertebral artery enters the lateral mass foramen of the sixth cervical vertebra before ascending to the brain.Correct Answer: C 6
2653. (2701) Q6-3195:
To avoid damages to the vertebral arteries when exposing the posterior aspect of the first cervical vertebra, dissection should be limited to mm from the midline on the superior aspect of C 1 and mm from the midline on the posterior aspect of C 1.
1) 8 mm; 12 mm
3) 12 mm; 16 mm
2) 10 mm; 14 mm
5) 16 mm; 22 mm
4) 14 mm; 20 mm
One must be careful not to damage the vertebral artery when exposing the posterior and superior aspect of the C 1 vertebra. It is especially important when using a Cobb elevator or an electrocautery not to dissect too far from the midline.
The vertebral artery lies close to the midline. On the superior aspect, the groove for the vertebral artery lies 8 mm to12 mm from the midline. On the posterior aspect of the vertebral body, the vertebral artery lies 12 mm to 23 mm from the midline.
Correct Answer: 8 mm; 12 mm
Which of the following levels most significantly contributes to the blood supply of the cervical spinal cord:
1) C 2 (accompanying the left C 2 spinal nerve)
3) C 6 (accompanying the left C 6 spinal nerve)
2) C 4 (accompanying the right C 4 spinal nerve)
5) T1 (accompanying the right T1 spinal nerve)
4) C 7 (accompanying the right C 7 spinal nerve)
The major blood supply to the cervical spinal cord comes from the anterior spinal artery, which arises from the deep cervical artery. This vessel most commonly accompanies the left C 6 spinal nerve.Correct Answer: C 6 (accompanying the left C 6 spinal nerve)
2655. (2931) Q6-3434:
Patients with Brown-Séquard syndrome usually presents with:
1) Ipsilateral paralysis, loss of contralateral vibration and touch sensation, and loss of ipsilateral pain and temperature sensation
3) Ipsilateral paralysis, loss of ipsilateral vibration and touch sensation, and loss of contralateral pain and temperature sensation
2) Ipsilateral paralysis and loss of contralateral vibration, and touch, pain, and temperature sensation
5) Contralateral paralysis, loss of contralateral vibration and touch sensation, and loss of ipsilateral pain and temperature sensation
4) Contralateral paralysis, loss of ipsilateral vibration and touch sensation, and loss of contralateral pain and temperature sensation
Brown-Séquard Syndrome usually results from hemisection of the spinal cord, which is often a result of trauma (eg, penetrating stab wounds). Clinical presentation usually consists of:
Ipsilateral paralysis
Loss of ipsilateral vibration and touch sensation
Loss of contralateral pain and temperature sensation
Correct Answer: Ipsilateral paralysis, loss of ipsilateral vibration and touch sensation, and loss of contralateral pain and temperature sensation
2656. (2932) Q6-3435:
Central cord syndrome is typically due to:
1) An axial compression injury with resultant injury to the central gray matter
3) A hyperextension injury with compression of the cord by osteophytes anteriorly and infolded ligamentum flavum posteriorly
2) A hyperextension injury with compression of the cord by herniated disk material anteriorly
5) A hyperflexion injury compression of the cord by the anterior longitudinal ligament anteriorly and osteophytes posteriorly
4) A hyperflexion injury with compression of the cord by herniated disk material anteriorly
Central cord syndrome is the most common incomplete spinal cord lesion and is usually seen in patients with preexisting cervical spondylosis who then sustain a hyperextension injury to the cervical spine. This mechanism causes compression of the cord by osteophytes anteriorly and the infolded ligamentum flavum posteriorly with resulting injury to the central gray matter. The clinical presentation is variable but usually consists of:
Greater loss of motor neurons to the upper extremities than the lower extremities often resulting in profound weakness in the arms and hands, and some weakness in the legs and feet
Variable sensory loss
Patients with central cord syndrome have variable return of function but are usually left with some degree of residual deficit and spasticity.
Correct Answer: A hyperextension injury with compression of the cord by osteophytes anteriorly and infolded ligamentum flavum posteriorly
Patients with anterior cord syndrome usually presents with:
1) Preservation of motor function, preservation of pain and temperature sensation, and loss of vibration and touch sensation
3) Motor paralysis, loss of pain, temperature, vibration, and touch sensation
2) Preservation of motor function, with loss of pain, temperature, vibration, and touch sensation
5) Motor paralysis, loss of vibration and touch sensation, and preservation of pain and temperature sensation
4) Motor paralysis, loss of pain and temperature sensation, and preservation of vibration and touch sensation
Anterior cord syndrome is due to injury of the anterior elements of the spinal cord, which is usually due to a space-occupying lesion anterior to the cord such as vertebral body fracture fragments, a herniated disk, or a hematoma. The clinical presentation consists of:
Complete motor paralysis (loss of anterior corticospinal tract)
Loss of pain/temperature sensation (loss of lateral and anterior spinothalamic tracts)
Preservation of vibration sensation/proprioception and light touch sensation (preservation of dorsal columns)
In less severe cases, some motor function is preserved through the lateral corticospinal pathways. Prognosis is generally poor and in patients with absence of sacral sensation (pin prick/temperature) after 24 hours following injury, recovery is seen in 10% of patients.
Correct Answer: Motor paralysis, loss of pain and temperature sensation, and preservation of vibration and touch sensation
2658. (2934) Q6-3437:
A patient with cauda equina syndrome and the full spectrum of symptons presents with:
1) Severe low back pain, sciatica, saddle anesthesia, and preservation of bladder vesicular control
3) Severe low back pain, saddle anesthesia, loss of motor and sensation in the lower extremities, and preservation of bulbocavernosus reflex.
2) Severe low back pain, sciatica, urinary retention, and preservation of perianal sensation
5) Severe low back pain, sciatica, saddle anesthesia, urinary retention, and loss of bulbocavernosus reflex
4) Severe low back pain, loss of motor and sensation in the lower extremities, and preservation of bladder vesicular control
Cauda equina syndrome is a severe neurologic disorder that results from an injury to the neural elements within the thecal sac between the conus medullaris and the lumbosacral nerve roots (ie, cauda equina or "horse's tail"). Cauda equina syndrome usually occurs as a result of lumbar disk herniation with compression of the cauda equina and requires urgent surgical decompression. Clinical presentation includes:
Severe low back pain Bilateral or unilateral sciatica Saddle anesthesia
Motor or sensory deficit
Bladder and bowel vesicular involvement (classically leading to urinary retention)
With a complete lesion, a loss of bulbocavernosus reflex, anal wink, and reflexes in the lower extremities
Correct Answer: Severe low back pain, sciatica, saddle anesthesia, urinary retention, and loss of bulbocavernosus reflex
2659. (2951) Q6-3454:
Which of the following descriptions applies to the sacroiliac joint:
1) The sacroiliac joint accounts for 15% of lower back pain.
3) Focal pain over the sacral sulcus is rare.
2) Pain is referred most commonly to the groin.
5) Provocative tests (Patrick and Gaenslens) are useful predictors of joint pathology.
4) Focal neurological deficits are common.
Sacroiliac joint pathology accounts for 15% of lower back pain, and the sacroiliac joint is one of the most common sites of referred pain. Patients with sacroiliac joint pathology commonly experience pain above the posterior buttock and seldom have focal neurological deficits. Physical examination tests are poor predictors of sacroiliac joint pathology.Correct Answer: The sacroiliac joint accounts for 15% of lower back pain.
Which of the following statements is true regarding the sacroiliac joint:
1) The anterior supporting structures are stronger than the posterior supporting structures.
3) The sacroiliac joint withstands medially directed forces better than the lumbosacral spine.
2) Sectioning of the sacrotuberous and sacrospinous ligaments results in increased motion.
5) The posterior interosseous ligaments are weak.
4) Counter-nutation (forward rotation of the ilium on the sacrum) is the most common motion.
The sacroiliac joint is the largest axial joint in the body. The anterior capsule is thin and weaker than the posterior capsule. The posterior supporting structures are strong and are comprised of a tough interosseous ligament, a long posterior sacroiliac ligament, and strong sacrotuberous, sacrospinous ligaments. Joint innervation usually occurs anteriorly in the S2 ventral rami. Compared with the lumbosacral spine, the sacroiliac joint can better withstand medial forces, but is weaker in axial compression and in axial torsion. Nutation (backward rotation of less than 4° and 1.6 mm rotation of the ilium on the sacrum) is the most common motion in the sacroiliac joint. Increased motion of the sacroiliac joint occurs only with sectioning of the interosseous ligaments.Correct Answer: The sacroiliac joint withstands medially directed forces better than the lumbosacral spine.
2661. (2966) Q6-3472:
Osteochondromatosis is a hereditary genetic disorder that is:
1) Autosomal recessive with incomplete penetrance
3) Autosomal dominant with incomplete penetrance
2) Autosomal recessive with complete penetrance
5) Sex-linked recessive
4) Sex-linked dominant
Osteochondromatosis (also known as hereditary multiple exostoses) is a genetic disorder that is autosomal dominant with incomplete penetrance in women. The genetic defect occurs on the EXT1, EXT2, and EXT 3 genes located on chromosome 8q24.Correct Answer: Autosomal dominant with incomplete penetrance
2662. (2967) Q6-3473:
Osteochondromatosis is a hereditary genetic disorder that is caused by:
1) Mutation in the fibrillin-1 gene
3) Mutation in the g-fos gene
2) Translocation between chromosomes 9 and 22
5) Mutation in the EXT1, EXT2, and/or EXT3 genes
4) Translocation between chromosomes 11 and 22
Osteochondromatosis (also known as hereditary multiple exostoses) is a genetic disorder that is autosomal dominant with incomplete penetrance in women. The genetic defect occurs on the EXT1, EXT2, and EXT 3 genes located on chromosome 8q24. Mutation in the fibrillin-1 gene is seen in patients with Marfan syndrome. Translocation between chromosomes 9 and 22 is seen in myxoid chondrosarcoma. Mutation in the g-fos gene is seen in patients with Ollierâs disease. Translocation between chromosomes 11 and 22 is present in patientâs with Ewingâs tumor.Correct Answer: Mutation in the EXT1, EXT2, and/or EXT3 genes
2663. (2968) Q6-3474:
Osteochondromas in the spine most commonly occur in:
1) Posterior elements of the cervical spine
3) Posterior elements of the lumbar spine
2) Posterior elements of the thoracic spine
5) Vertebral body of the thoracic spine
4) Posterior elements of the sacral spine
Osteochondromas most commonly occur in the appendicular skeleton but can also occur in the spine (<5% of cases). When present in the spine, solitary osteochondromas have a predilection for the cervical spine. They can, however, also occur in the thoracic and lumbar spine. Sacral involvement is rare.Correct Answer: Posterior elements of the cervical spine
When an osteoid osteoma occurs in the spine, it can involve all of the following except:
1) Facets
3) Pedicles
2) Transverse processes
5) Vertebral body
4) Rib heads adjacent to thoracic vertebrae
When an osteoid osteoma occurs in the spine, involvement of the posterior elements of the vertebra is typical and includes: Lamina
Pedicles
Transverse processes Facets
Rib heads adjacent to thoracic vertebrae Correct Answer: Vertebral body
2665. (2970) Q6-3476:
Typical histologic features of an osteoid osteoma include all of the following except:
1) Chondrocytes in an arrangement similar to that of a physis
3) Varying degrees of mineralization with greatest mineralization in the center of the lesion
2) Nidus composed of haphazardly arranged network of osteoid trabeculae
5) Vascularized spindle cell stroma
4) Osteoblasts rimming the trabeculae
The histologic features of an osteoid osteoma include the following:
Nidus composed of haphazardly arranged network of osteoid trabeculae
Varying degrees of mineralization with greatest mineralization in the center of the lesion Loose fibrovascular connective tissue between trabeculae
Osteoblasts rimming the trabeculae Vascularized spindle cell stroma
Correct Answer: Chondrocytes in an arrangement similar to that of a physis
2666. (2971) Q6-3477:
Treatment of a vertebral osteoid osteoma includes all of the following except:
1) Surgical excision/curettage of the nidus
3) Observation if symptoms are mild
2) En-bloc resection
5) Radiofrequency ablation
4) Aspirin/salicylates/nonsteroidal anti-inflammatory drugs (NSAIDs)
Treatment of osteoid osteomas in the spine include the following: Aspirin/salicylates/NSAIDs
Administered for up to 2 years Successful in up to 50% cases
Radiofrequency ablation (RFA)
Usually computed tomography-guided
Clinical success rates as high as 97% have been reported with 1 to 2 treatments Surgical excision of the nidus/curettage
Necessary when aspirin/salicylates/NSAIDs cannot be tolerated for long periods of time and RFA is not possible or unsuccessful
Can usually be accomplished through a posterior approach
En-bloc resection or a more radical procedure play no role in management Correct Answer: En-bloc resection
1) Torticollis
3) Stiffness
2) Painful scoliosis
5) Radicular symptoms
4) Diskogenic pain
The most common symptoms of spinal osteoblastomas include: Pain
Usually the first and most common presenting symptom Night pain is not as common as it is with osteoid osteomas
Night pain is not as common as it is with osteoid osteomas Painful scoliosis
Torticollis Stiffness
Radicular symptoms usually due to mass effect Correct Answer: Diskogenic pain
2668. (2973) Q6-3479:
When an osteoblastoma occurs in the spine, it can involve all of the following except:
1) Facets
3) Pedicles
2) Transverse processes
5) Vertebral body
4) Lamina
When an osteoblastoma occurs in the spine, involvement of the posterior elements of the vertebra is typical and includes: Lamina
Pedicles
Transverse processes Facets
Rib heads adjacent to thoracic vertebrae Correct Answer: Vertebral body
2669. (2974) Q6-3480:
The proper treatment of a vertebral osteoblastoma includes:
1) Chemotherapy
3) Marginal excision/curettage of the tumor
2) En-bloc resection
5) Radiofrequency ablation
4) Radiation
Treatment of spinal osteoblastomas usually consists of marginal excision or curettage of the tumor. Local recurrence rates of up to 10% have been observed from some osteoblastomas, however, malignant degeneration is rare. There is no role for radiation or chemotherapy. Radiofrequency ablation has been used successfully for the treatment of osteoid osteomas, but not osteoblastomas.Correct Answer: Marginal excision/curettage of the tumor
1) Vascularized spindle cell stroma
3) Occasional areas of aneurysmal bone cyst formation
2) Nidus composed of haphazardly arranged network of osteoid trabeculae
5) Chondrocytes arranged in a zonal pattern
4) Osteoblasts rimming the trabeculae
Histologically osteoblastoma is similar to an osteoid osteoma; its features include:
Irregular osteoid arranged haphazardly with rimming by round osteoblasts Loose fibrovascular connective tissue between trabeculae
Osteoblasts rimming the trabeculae Vascularized spindle cell stroma
Areas of aneurysmal bone cyst formation can be seen Correct Answer: Chondrocytes arranged in a zonal pattern
2671. (2976) Q6-3482:
What percentage of osteoblastomas occur in the spine:
1) 20% to 30%
3) 40% to 50%
2) 30% to 40%
5) 60% to 70%
4) 50% to 60%
Osteoblastomas are:
Osteoblastic bone-forming lesions measuring more than 2 cm in size characterized by marked growth potential Similar in histology and presentation to osteoid osteoma with the main difference being the size of the tumor Most common in the 2nd and 3rd decades of life
Twice as common in men than in women Common in the spine:
Spinal osteoblastomas account for 40% to 45% of all osteoblastomas Over half of spinal osteoblastomas occur in the lumbar spine
Correct Answer: 40% to 50%
2672. (2979) Q6-3485:
Primary spinal tumors account for:
1) 0.4% of all tumors and 1% of all bone tumors
3) 0.4 % of all tumors and 25% of all bone tumors
2) 0.4 % of all tumors and 10% of all bone tumors
5) 0.04 % of all tumors and 25% of all bone tumors
-
0.04% of all tumors and 10% of all bone tumors
Neoplasms of the spine can be broadly categorized into metastatic tumors and primary tumors. Primary spinal tumors are rare and account for 0.04% of all tumors and 10% of all bone tumors.Correct Answer: 0.04% of all tumors and 10% of all bone tumors
patients:
1) 55%
3) 75%
2) 65%
-
95%
4) 85%
-
Pain is the most common complaint in patients presenting with a primary spine tumor
-
Present in up to 85% of patients
-
Typically localized to the site of lesion but can be radicular
-
Characterized as:
-
Progressive
-
Gradual in onset
-
Worse at night
-
Non-mechanical
-
-
Loosely associated with trauma
-
-
Weakness can be seen in up to 42% of patients
-
Mass is evident in up to 16% of patients
-
Three percent of patients are asymptomatic
-
Other symptoms can include:
-
Sensory loss
-
Loss of sphincter control Correct Answer: 85%
-
-
2674. (2981) Q6-3487:
Patients presenting with a primary spine tumor most often characterize their pain as:
1) Constant, sudden in onset, worse at night, mechanical, and loosely associated with trauma
3) Progressive, gradual in onset, worse at night, non-mechanical, and loosely associated with trauma
2) Constant, gradual in onset, worse at night, non-mechanical, and loosely associated with trauma
5) Progressive, gradual in onset, worse at night, mechanical, and loosely associated with trauma
4) Progressive, sudden in onset, worse at night, mechanical, and loosely associated with trauma
-
Pain is the most common complaint in patients presenting with a primary spine tumor
-
Present in up to 85% of patients
-
Typically localized to the site of lesion but can be radicular
-
Characterized as:
-
Progressive
-
Gradual in onset
-
Worse at night
-
Non-mechanical
-
Loosely associated with trauma
-
-
-
Weakness can be seen in up to 42% of patients
-
Mass is evident in up to 16% of patients
-
Three percent of patients are asymptomatic
-
Other symptoms can include:
-
Sensory loss
-
Loss of sphincter control
-
-
Correct Answer: Progressive, gradual in onset, worse at night, non-mechanical, and loosely associated with trauma
1) 20% to 40%
3) 40% to 60%
2) 30% to 50%
5) 70% to 90%
4) 60% to 80%
-
Imaging studies used most frequently in the diagnosis of primary spine tumors include:
-
Plain radiographs
-
Initial imaging study
-
Recommended for any patient with prolonged back pain (>6 weeks)
-
Identify 30% to 70% of spine tumors at presentation
-
Early lesions difficult to detect because 30% to 50% of trabecular bone must be destroyed before changes can be seen
-
Absence of the pedicle is usually the earliest radiographic sign of vertebral
-
Cortical bone loss easier to detect than destruction of trabecular bone
-
"Winking owl" sign
-
Disk space generally preserved
-
Geographic lesions with well-circumscribed borders suggest a benign tumor
-
Permeative lesions suggest a malignant tumor
-
-
Bone scan
-
Technetium (Tc)-99m
-
Can identify lesions 3 to 18 months before plain radiographs
-
Sensitivity 74%, specificity 81%
-
False negative in up to 60% of patients with multiple myeloma
-
Single photon emission computerized tomography scan can improve both sensitivity (87%) and specificity (91%)
-
When used in combination with gallium scanning, Tc-99 bone scan can help to differentiate between tumors and infections
-
-
Computed tomography/myelography
-
Best test to determine extent of bony destruction
-
Important in surgical planning
-
Myelography usually used only when magnetic resonance imaging (MRI) not possible (danger of complete myelographic block)
-
-
MRI
-
-
Modality of choice in evaluating tumors of the spine
-
Noninvasive
-
Allows direct visualization of entire spinal cord
-
Visualization of soft tissues
-
Sensitivity 92%, specificity 90%
-
Additional lesions in 20% to 24%, and 10% will have multiple levels of cord compression
Correct Answer: 30% to 50%
1) Osteoblastoma
3) Giant cell tumor
2) Eosinophilic granuloma
5) Aneurysmal bone cyst
4) Chordoma
-
Primary benign tumors of the spine are:
-
Slow-growing
-
Well-circumscribed
-
Usually occur in patients younger than 21 years of age
-
Involve the vertebral body and posterior elements of the spine
-
Overall slight predilection for the posterior elements
-
Location of tumor is an important factor in determining the type of tumor
-
-
Examples include:
-
Osteochondroma
-
Osteoid osteoma
-
Osteoblastoma
-
Aneurysmal bone cyst
-
Giant cell tumor
-
Eosinophilic granuloma
-
-
-
Primary malignant tumors of the spine are:
-
Fast-growing
-
Permeative
-
Usually occur in patients older than 21 years of age
-
Examples include:
-
Multiple myeloma/solitary plasmacytoma
-
Osteosarcoma
-
Chondrosarcoma
-
Ewing's sarcoma/primitive neuroectodermal tumor
-
Chordoma
-
Lymphoma Correct Answer: Chordoma
-
-
1) Osteosarcoma
3) Multiple myeloma
2) Chordoma
5) Lymphoma
4) Osteoblastoma
-
Primary benign tumors of the spine are:
-
Slow-growing
-
Well-circumscribed
-
Usually occur in patients younger than 21 years of age
-
Involve the vertebral body and posterior elements of the spine
-
Overall slight predilection for the posterior elements
-
Location of tumor is an important factor in determining the type of tumor
-
-
Examples include:
-
Osteochondroma
-
Osteoid osteoma
-
Osteoblastoma
-
Aneurysmal bone cyst
-
Giant cell tumor
-
Eosinophilic granuloma
-
-
-
Primary malignant tumors of the spine are:
-
Fast-growing
-
Permeative
-
Usually occur in patients older than 21 years of age
-
Examples include:
-
Multiple myeloma/solitary plasmacytoma
-
Osteosarcoma
-
Chondrosarcoma
-
Ewing's sarcoma/primitive neuroectodermal tumor
-
Chordoma
-
Lymphoma Correct Answer: Osteoblastoma
-
-
1) Slow-growing, well-circumscribed, and usually occur in patients older than 21 years of age
3) Fast-growing, permeative, and usually occur in patients older than 21 years of age
2) Slow-growing, well-circumscribed, and usually occur in patients younger than 21 years of age
5) Fast-growing, permeative, and usually occur in patients older than 40 years of age
4) Fast-growing, permeative, and usually occur in patients younger than 21 years of age
-
Primary benign tumors of the spine are:
-
Slow-growing
-
Well-circumscribed
-
Usually occur in patients younger than 21 years of age
-
Involve the vertebral body and posterior elements of the spine
-
Overall slight predilection for the posterior elements
-
Location of tumor is an important factor in determining the type of tumor
-
-
Examples include:
-
Osteochondroma
-
Osteoid osteoma
-
Osteoblastoma
-
Aneurysmal bone cyst
-
Giant cell tumor
-
Eosinophilic granuloma
-
-
-
Primary malignant tumors of the spine are:
-
Fast-growing
-
Permeative
-
Usually occur in patients older than 21 years of age
-
Examples include:
-
Multiple myeloma/solitary plasmacytoma
-
Osteosarcoma
-
Chondrosarcoma
-
Ewing's sarcoma/primitive neuroectodermal tumor
-
Chordoma
-
Lymphoma
-
-
Correct Answer: Fast-growing, permeative, and usually occur in patients older than 21 years of age
2679. (3069) Q6-3577:
All of the following are elements of the lateral mass of cervical spinal segments except:
1) Inferior articulating process
3) Spinous process
2) Superior articulating process
5) Transverse foramen
4) Transverse process
The lateral mass of the cervical spinal segments includes the inferior and superior articulating processes, the transverse foramen, and the transverse process. The spinous process is not an element of the lateral mass.Correct Answer: Spinous process
2680. (3070) Q6-3578:
Advantages of minimally invasive lumbar interbody fusion over traditional open interbody fusion include:
1) Minimal muscle dissection and trauma
3) Better fusion rates
2) Wider surgical exposure
4) Lowered risk of nerve root injury
Minimally invasive lumbar interbody fusion involves less muscle dissection and trauma than traditional open approaches. The surgical exposure is more limited, though, and there is no evidence to date of minimally invasive techniques providing better fusion rates or lowered risk of nerve root injury.Correct Answer: Minimal muscle dissection and trauma
1) Minimally invasive fusion may only be safely performed with the assistance of endoscopy.
3) Internal fixation with pedicle screws is not possible via the minimally invasive approach.
2) Minimally invasive fusion has increased risk of nerve root injury.
4) Intraoperative fluoroscopy if of great value in minimally invasive fusion.
Intraoperative fluoroscopy or radiography is vital for the proper identification of lumbar level and vertebral structures in minimally invasive posterior lumbar interbody fusions. While endoscopic assistance has been well described as a method of minimally invasive fusion, it is not vital to this technique. There is no evidence of increased risk of nerve root injury with minimally invasive techniques, and it is possible to internally fixate the lumbar segment with pedicle screws through minimally invasive techniques.Correct Answer: Intraoperative fluoroscopy if of great value in minimally invasive fusion.
2682. (3137) Q6-3855:
A 21-year-old man presented to the emergency department after sustaining a low-velocity gunshot wound to his midback resulting in grade 0 (out of 5) weakness in his quadriceps and tibialis anterior muscles. His extensor hallucis longus and gastrocnemius/soleus muscles were grade 3 (out of 5) bilaterally. His sensation remained intact. An intradural bullet fragment was seen at T12. No fracture was seen on computed tomography (CT) scan. Management should consist of:
1) Administration of methylprednisolone 30 mg/kg bolus followed by an infusion of 5.4 mg/kg for 24 hours.
3) Administration of broad-spectrum antibiotics for 14 days.
2) Application of a thoracolumbosacral orthosis (TLSO).
5) Removal of the bullet fragment and instrumented fusion from T10 to L2.
4) Removal of the bullet fragment.
In complete and incomplete lesions from T12 to L4, removal of the bullet fragment from the canal has been associated with significant motor recovery. This improvement is not seen in other regions of the spine.
High-dose steroids have not been shown to offer improvement in patients with spinal cord injury after a gunshot wound, and the complications of high-dose steroids have been documented in this population. The majority of gunshot wounds to the spine are stable injuries. This patientâs CT scan does not demonstrate any instability. Therefore, neither nonoperative (eg, TLSO bracing) nor operative (instrumented fusion) stabilization is indicated. While infection after transalimentary bullet wounds to the spine is a well-documented complication, this patientâs injury was sustained from the back, thereby avoiding the alimentary canal and obviating the need for intravenous antibiotics.
Correct Answer: Removal of the bullet fragment.
2683. (3155) Q6-3974:
In relation to the lumbar pedicle, the exiting nerve root is found:
1) Immediately superior to the pedicle
3) At the midpoint between the superior and inferior level pedicles
2) Immediately inferior to the pedicle
5) None of the above
4) Nerve root has no anatomic relationship to the pedicle
The exiting nerve root is found traversing immediately inferior to the pedicle.Correct Answer: Immediately inferior to the pedicle
2684. (3156) Q6-3975:
Regarding the anatomy of the lumbar pedicle, which of the following statements is true:
1) The pedicle is located at the origin of the transverse process.
3) The pedicle is located at the base of the superior facet, at the origin of the transverse process.
2) The exiting nerve root is found immediately superior to the lumbar pedicle.
5) There is no relationship between the pedicle and the superior facet.
4) The pedicle joins the vertebral body at its inferior border.
The lumbar pedicle is the bony bridge that connects the posterior vertebral elements to the anterior body. The pedicle is located at the base of the superior facet, at the origin of the transverse process. The exiting nerve root traverses immediately inferior to the superior segment pedicle, and the pedicle joins the vertebral body at its superior half.Correct Answer: The pedicle is located at the base of the superior facet, at the origin of the transverse process.